1st series [1] [2] [3] [4] [5] [6] [7] [8] [9] [10] [11] [12] [13] [14] [15] [16] [17] [18] [19] [20] [21] [22] [23] [24]  2nd series [1] [2] [3] [4] [5] [6] [7] [8] [9] [10] [11] [12] [13] [14] [15] [16] [17] [18] [19] [20] [21] [22] [23] [24] [25] [26] [27] [28] [29] [30] [31] [32] [33] [34] [35] [36] [37] [38] [39] [40] [41] [42] [43] [44] [45] [46] [47] [48] [49]

  View the latest questions and answers at askaphilosopher.org

Ask a Philosopher: Questions and Answers 17 (1st series)

Here are some of the questions that you asked a philosopher from May 2002 — July 2002:

  1. What philosophy is
  2. Paradoxes, logic and the problem of evil
  3. Difference between chance and a miracle
  4. Athens and Jerusalem
  5. Can a photograph engender understanding?
  6. Difference between murder and killing
  7. The telos of a human being
  8. Entropy and the 2nd law of thermodynamics
  9. Belief, knowledge and faith
  10. God and morality
  11. How can a material thing think?
  12. Defining what is valuable about a 'person'
  13. How a good Father could allow his son to be crucified
  14. Is economics politics in disguise?
  15. Hume vs. Descartes on the idea of a perfect being
  16. Objections to Kant's categorical imperative
  17. Why a falling object cannot reach the ground
  18. Nietzsche, social darwinism, nazism and eugenics
  19. What makes one definition more correct than another
  20. Why there can't be two Gods
  21. Happiness, joy and pleasure
  22. Something philosophical about Shakespeare
  23. "I will, therefore I am"
  24. Lacan on Hegel's master and slave
  25. Philosophy of education
  26. Can non-human animals be moral?
  27. Natural law theory and divine command theory
  28. Why "to exist is to be free"
  29. 'Good' and 'happiness' as ends
  30. Is the idea of God ingrained in our minds?
  31. Cures for depression
  32. Why it is hard to be a philosopher
  33. Why there can't be a Utopian society
  34. Chaos, randomness and the butterfly effect
  35. Information on tattoos and body piercing
  36. Are we just asking the same questions over and over?
  37. Existence of extra-terrestrial life
  38. Definitions of 'rationalist' and 'empiricist'
  39. Human belief that we are the dominant species
  40. Fate, destiny and free will
  41. Where concepts of good and evil come from
  42. "What it is like to be me" and "what it is like to be a bat"
  43. Empiricist argument for scepticism
  44. Why we take risks in pursuit of sex
  45. Definition of the word "word"
  46. Can entropy be explained by probability?
  47. Who was the "father of Greek science"?
  48. How "other minds" can exist for the solipsist
  49. Discipline, democracy, freedom and punishment
  50. Individuality and the just society
  51. Kant on synthetic a priori knowledge
  52. Mind-body problem and psychology
  53. Is Buddhism a philosophy or a religion?
  54. "The limits of my language are the limits of my world"
  55. I have dreams in which I am a boy
  56. Should we help people who've made wrong decisions in life
  57. How life can be objectively meaningless but have subjective meaning
  58. Information about the 'Human Project'
  59. Overcoming the sense of futility in the face of death
  60. Freud on religious belief
  61. Popper on 'conjecture' and 'refutation'
  62. "What I tell you three times is true"
  63. Is this a quote from Wittgenstein?
  64. Einstein's special theory of relativity
  65. Why we need history of philosophy
  66. Different types of philosophical inquiry
  67. How photographs can depict abstract objects
  68. "The rebel can never find peace..."
  69. Can philosophy be taught?
  70. Kierkegaard on aesthetic, ethical and religious stages
  71. Was slavery immoral in the past or only now?
  72. Jaspers on 'ciphers'
  73. How can I become a professional philosopher?
  74. Value of martyrdom
  75. Did the universe have a beginning according to Spinoza?
  76. Check my ontological proof of God's existence
  77. Why a perfect copy of me would not have "my" consciousness
  78. How can we best live, while still searching for the truth?
  79. 'Modal' argument for mind-body dualism
  80. What Heidegger means by 'feeling'
  81. Spirit, ego, mind, soul, consciousness
  82. Further reflections on Clinton and Lewinski
  83. Christianity in a nutshell
  84. Idea of an existentialist 'logic'
  85. Philosophy and the art of M.C. Escher
  86. Human emotions and the philosophy of film
  87. Everyone is a philosopher
  88. Sartre on how we choose values
  89. Evaluating good intentions which lead to bad consequences
  90. Information about Norman Malcolm
  91. Postmodernism and traditional philosophy
  92. How to be a better general through philosophy
  93. "What I tell you three times is true" a second time
  94. Point of philosophy
  95. Age, race and disability and the idea of equality
  96. Is marriage doomed?
  97. Is evil just misperception of the good?
  98. God's knowledge of the world before Creation
  99. Applying Descartes' doubt to human testimony
  100. How do we know we exist?
  101. On the alleged African origin of Greek philosophy
  102. Naturalism and spiritualism
  103. Thoughts on God for a reluctant atheist
  104. Is it wrong to kill animals for food?
  105. Hard determinist view of the death penalty
  106. Something about Confucius
  107. Puzzles about memory
  108. Kant's advice about suicide
  109. Idea of 'false knowledge'
  110. Topics in Plato's Republic
  111. How philosophy can bring about peace
  112. Churchland's argument for eliminative materialism
  113. Did God create logic?
  114. Why western religions are against sex and drugs
  115. Explaining visual illusions
  116. Philosophy as part of our daily life
  117. "If you want to improve, be content to be thought foolish and stupid..."
  118. Plato on permanence, change and the simile of the line
  119. How fallacies can be persuasive
  120. I'm looking for philosophical issues to take a shot at
  121. Unanswerable questions and pseudo questions
  122. Value judgements and factual judgements
  123. Is science the new religion?
  124. Popper, Quine and Kuhn on how theories are rejected
  125. Morality, rationality and utilitarianism
  126. Simple advice on how to increase one's knowledge and one's willpower
  127. Wittgenstein on 'family resemblances'
  128. How to study both philosophy and law
  129. Logical positivism on the meaninglessness of 'God' statements
  130. What's the latest view on theism vs. agnosticism vs. atheism?
  131. John Locke on revelation as a source of knowledge
  132. Can an Act of Parliament make people better?
  133. Plato on kosmos noetos and aeithetos
  134. The most important ethical issue

Desmond asked:

What is philosophy and where does it come from?

What would Kant say to that question?

Every introductory text on philosophy starts with some comments on your question. So look it up.

The first general answer is: "Philosophy is about questions." But then science and theology — and "common sense" and superstition and "mom and dad and your friends" — give answers to questions too. So what's special with philosophical answers?

The first divide is between philosophy and religion: Religion states for instance: "In the beginning God created heaven and earth." And then the philosopher unsatisfied asks: "How do the priests and the Holy Scriptures came to know that — have they been there?" and then "What do they mean by 'God'?" And next he may think them to be liars who try to scare the people to make them well behaved and obedient to the priests and elders and the king.

The second divide is between philosophy on the one hand and experience and "common sense" on the other: Where do people get their "knowledge"? That was the concern of Socrates. He said "Nobody knows anything for sure. I don't know anything too. But at least I know this one thing and I am not boasting I know anything for sure or going to sell it." Now that is an extreme position from which the sceptics started. Plato was not that modest and he invented the metaphysical concept of "ideas". How do we know that some way of acting or thinking is "better" than another way if we had not some inborn idea of what acting and thinking should be "ideally". Aristotle thought this to be an unnecessary and unjustified conclusion: By everyday experience we know that all things can be done in a stupid and clumsy way and in a masterly way. But to know that one doesn't need the concept of "ideas". So this was one of the first struggles between two first-class philosophers.

But what is "philosophical" about the arguments of Plato and Aristotle in this case? They argue not over objects but over arguments and their justification. That is one of the greatest themes of philosophy: How to distinguish a justified and "valid" argument from a "sloppy" and invalid one. This "critical" approach to philosophy characterized the style of thinking of Descartes, Kant and Wittgenstein.

Philosophers are arguing as we all do, but the question of the philosopher is: "How can I be sure that my arguments are valid? Or as Wittgenstein had it: "What are we doing with our words and sentences, how do we use them in a correct and meaningful way?"

Just as you can build imaginary worlds in dreams and in the movies and in the arts, so you can build imaginary, fictitious worlds in texts with words and arguments. If you do that in a novel, that's no problem, since a novel is openly sold and bought as fiction, as a written dream. But if you sell some political or religious or scientific fiction as truth and reality, then the true and concerned philosopher is getting nervous. As the late Nelson Goodman once stated: "There may indeed be more things between heaven and earth than our wisdom dreams of, as Hamlet said, but it is my duty to see to it, that there are not too much more things dreamt of than do exist between heaven and earth." (That was the drift of what he said, I cite from memory.) You should read the short and wonderful book Ways of Worldmaking by Nelson Goodman. It's not about "making" different worlds like a movie-director or a novel-writer does, but it's about different ways of seeing the world: The Antiquity "saw" another world than the Middle Ages or the Modern Age. And as Goodman puts it in the foreword to the book cited: "Kant exchanged the structure of the world for the structure of the mind. C.I. Lewis exchanged the structure of the mind for the structure of concepts. And that now proceeds to the exchange of the structure of concepts for the structure of the several symbol systems of the sciences, philosophy, the arts, perception, and everyday discourse."

Of course the study of different ways of seeing the world goes back to Vico and Hegel, and the study of causes for this different ways of seeing the world is indebted to Feuerbach, Marx, Nietzsche, Freud — to name but some of the most important thinkers on that question.

So philosophy is a really great undertaking of some of the best human minds. An undertaking to achieve what? To clear our understanding and our arguments concerning the world around us — and our way of arguing — from false pretensions and false presumptions, from misleading concepts and misleading lines of thought to get at a true picture of the world we live in.

"A" true picture is not "the" true picture. There are many true pictures of the world as there are many true pictures of a person or a landscape or of anything else. But there are many false and distorted and misleading pictures too. There's no contradiction in this.

What did Kant say to that question "what philosophy is all about"? I am no specialist on Kant. There are some Kant dictionaries and concordances to look up for citations. But then his stance was not too different from Goodman's.

And then: Where does philosophy come from? It comes from doubt in the knowledge, wisdom and sincerity of the elders — and of oneself. Doubt comes from contradicting answers and experiences. The first great philosophers we know of — Socrates, Plato, Aristotle, but Confucius and Buddha and some other great Asian thinkers too — appeared in a time (ca. 600-400 BC) when the first great cities drew people from all countries, and international trade and colonization built a network of contacts between cultures as today. Then what was valid and valued in one region was not valid and valued in another region and people had to think how this had to be explained. That was the birth of philosophy. Look up Herodotus for this. And: This doubting in the words of the priests was called "Adams Fall" — the fall from the grace of innocence, of living in "no-doubt". Philosophy set man against nature, since he became aware of being a thinking animal, a doubting animal, an inquiring animal, not part of the whole of nature anymore, not able to speak to plants and animals or to share their world by transformation as in fairy tales.

So philosophy is the great eye-opener of mankind and the great destroyer of trust and naivety. A famous student of the presocratic philosophers in Greek Antiquity gave his book the title "From Mythos to Logos" — both words meaning "speech", but in a different sense. The same change took place in India and China and in some other countries at about the same time in another way. That was the beginning of philosophy.

Hubertus Fremerey

back

Bruce asked:

I have a number of questions that I am interested in pursuing. I am interested in studying philosophy and hopefully eventually teaching it. I am a Christian.

Some of the philosophical problems I am currently wrestling with:

Solutions to paradoxes (e.g. paradox of vagueness, paradox of the heap)

Solutions to the problem of evil

The implications/ relevance of Godel's incompleteness theorem to philosophy

Finding a good introduction book to logic (i.e. including argument construction, logical fallacies, symbolic logic etc...)

Here are some works and authors that might interest you on the topics you mention:

Solutions to Paradoxes: anything by Roy Sorenson. Especially Vagueness and Contradiction. I've just finished reading it and his views on vagueness generally struck me as quite sensible. His analysis of the paradox of the heap was very clear and accessible.

Solutions to the problem of evil: My advice to you as a Christian interested in philosophy would be to read Alvin Plantinga's "Advice to Christian Philosophers" (if you haven't already read it). This was a landmark event for Christian philosophy in the English speaking world and includes sections on both the logical and probabilistic versions of the argument from evil. I believe that the article was published in the journal Faith and Philosophy. Plantinga has a list of publications on his web page at the University of Notre Dame site.

Godel: On this topic I would recommend Michael Potter's book Reason's Nearest Kin.

Logic: The best text book on logic that I know of (and lots of people disagree with me about this....) is Quine's Methods of Logic. It probably doesn't qualify as introductory.

Lance Flowerree

back

Joseph asked:

Please describe the difference between a chance and a miracle (except that the latter is based on supernatural influence).

In terms of of ordinary language a miracle is an event or thing we cannot explain and chance only exists when we are non-determinists. As we humans seem to want to explain everything and also make everything predictable, and there are many different ways to do this, a more profound answer to this question will very much depend on your worldview.

If you for example asked a pantheist, he/she would simply answer that God is identical with everything, that therefore there are no supernatural events and hence it doesn't make sense to talk about miracles. If you add strong determinism to this view, then also nothing will happen by chance.

In the worldview of "classical physics" events are determined by cause and effect. Every event can be described by laws of nature. A miracle, then is a violation of a law of nature and there are two ways to explain away this violence:

a. Whenever we encounter a miracle we are hallucinating.
b. Every miracle can be explained, when our understanding of the laws of nature is sufficient.

Development psychology supports view b: To a new born baby everything is a miracle and happens by chance. Step by step he/she learns to explain things or at least to develop a model how things could be. As we grow older the number of miracles decreases while our knowledge increases. Some anthropologists apply this view of development on the whole mankind.

Many people feel that this more or less mechanical assumption is wrong and there really are miracles. To evade the strong opposition between describable event and miracle, between certainty and possibility/ chance, we nowadays tend to give the term miracle a weaker definition with a dash of chance. For example, Merriam Webster defines miracle as "an extremely outstanding or unusual event", which is similar to saying something happens by chance.

In terms of quantum physics strange things happen for the same reason that anything else happens — everything follows the same rules. Due to quantum mechanical effects, there is an inherent randomness, "chance", in everything. Everything has a certain probability of happening. When you roll a die, there is approximately a one in six chance of rolling any particular number between 1 and 6 and there is a very little chance for it to stop on an edge. Air molecules move around a room randomly. There is a small chance, that all of the air molecules move to the one side of the room. If this happened, it would not be good for anyone in the room at the time: the effect would be that people in the room suddenly exploded! As this would have happened for no obvious reason we would talk of a miracle. Hopefully this drastic example never will happen!

Simone Klein
http://www.sophiasworld.at

back

Rani asked:

Could you please explain the two main traditions: Jerusalem and Athens that prevailed until the Enlightenment Era? I can't find anything on the Jerusalem tradition but it was based on faith and was a response to questions such as "why are we here".

That's a great question. Start reading the first chapter of Erich Auerbach's Book Mimesis (ISBN: 0691012695), "The Scar of Odysseus". The Greek ("Athens") are "eye-people", they are seeing the world. Theo-ria means "seeing the gods of the township". One of the greatest inventions of the Greeks has been geometry. They posted statues on every corner, they advised their youth to shine in public, to present themselves, to be proud and ambitious. That all appeals to the eye. They were real show-masters and practically invented tragedy and comedy as we know them. And so the cosmos appeared as a wonderful order of moving parts in a necessary equilibrium like a wonderful great machinery. That's why Plato wrote over the entry of his academy: "Maedeis a-geometraitos eis-ito" — "No-one not loving geometry shall enter here!" The world was order to be looked at with the eyes of the body and with the eyes of the spirit and in-sight (!) of reason likewise. For the Greek beauty and truth and the good were but three aspects of reality and could not be inconsistent.

The jewish ("Jerusalem") understanding of the world is totally different. The fundamental experience of the Jew had been the Pharaoh of Egypt and the God-King of Babylon. People had not to argue, they had to obey. It didn't matter what they thought the world to be like if only they obeyed to the God-King. In German the word "Gehorsam" (obedience) is derived from "hoeren" (to hear). The whole Bible ist strewn on almost every page with "So says the Lord" and "Now hearken Israel!" and similar sentences. So the Jews didn't study nature, they studied texts, they studied "The Law" (The Torah) and they became great jurists, because jurists have to understand sentences and arguments and not geometrical figures. And "sin" is not a deviance from a timeless order of a "cosmos", but it's a deviance from a contract with God, it is a violation not of laws of nature or reason but it is a violation of love and trust and mutual respect between two contracting parties — God and Man or God and the Jewish People.

And then the Jew had, what the Greek had not: The concept of personal responsibility to a responsive person — God. The latin word "respondere" means "to answer". The Greek knew of nobody to answer to save their own reason and sense of beauty and what is proper and fitting.

Now combine these two strings starting from "Athens" and "Jerusalem" with the ability of the Romans to govern an empire that spanned the world from the Indus to the border of Scotland (Hadrian's wall). Then you see what the "History of the Occident" is all about. And you see why this singular combination of the Greek sense for rational order (to be seen in nature and mathematics) and the Jewish sense for interpersonal relations of love and trust and mutual respect surpassed the Orient with his eternal traditions. Of course this doesn't mean that the people of the Orient have been or are less bright than those of the Occident. But if you don't learn mathematics, you simply don't know mathematics, and if you don't look into the mechanics of nature, you don't know the mechanics of nature either, be as bright as you may. And if you know of no god you feel personally responsible to, then you behave in another way than when you do know one. That has nothing to do with intelligence or with personal superiority of any sort, and so any racists claim of western superiority is pure nonsense. But the combined forces of "Athens", "Jerusalem" and "Rome" have created — unintentionally of course — a singular form of culture that is now taken over by the rest of the world for it's efficiency.

Don't be scared by the word "efficiency" here: As Toynbee said in a Darwinian mood "History is the study of challenge and response of cultures to their environments". But then of course some people are saying that mankind is about to founder altogether in the "Titanic of Western Culture" on the iceberg of natural and human constraints. That you may think over for yourself.

Hubertus Fremerey

back

Jodi asked:

I have a interest in photography and I have an interest for learning more about philosophy. Through examining the ask a philosopher website I wanted to take a closer look into photography and philosophy.

My question to you is: Do you feel that photographs can or can not engender understanding? I mean many people debate whether photographs can be understood or have meaning like text can. Like Susan Sontag stated in her book On Photography that photographs can't engender understanding and by looking at a picture you can not get a complete understanding of the subjects involved like you can with a text. I was wondering how you felt about this subject?

I know this is not a simple question. But what true philosophy question is?

I have formed many opinions concerning this topic and just wanted someone else's point of view.

As you have no doubt seen on the Ask a Philosopher website, I lay no claim to being an expert on photography, though I enjoy taking photographs. Your question is rather more complex than it appears at face value. Can photographs engender understanding? In some cases the answer is yes, in others the answer is no. Consider a picture deliberately produced to instruct us to do something, say, scissors on a dotted line; this requires no language, the instruction is self evident. Historic pictures, if we know our history, can show us how things were in the past, we could work out the period and the event, or we could make a very good guess at what is going on. However, the more complex the picture, the more things happening, and the more in need we may be of words to describe the total scene. Therefore, the picture might give us a general, overall, view, but may fall short in coming to terms with specific conclusions. Hence, we are, in general, using our pre-conceived knowledge to interpret overall views, but for specific events within the overall view we often need some assistance from word description.

There is no doubt that a photographer by skillful use of the medium can compose pictures that say a great deal, and often a photograph without a caption can press home the point and leave no doubt in the mind what the photograph is telling us. but, again, there can be specifics within the picture which leave us wondering, e.g. Is the man on the sidewalk involved in what is going on? Why is the little girl running away? etc.. Unless the photographer fully describes what is going on we are bound to be dependent on imagination and guesswork in some instances.

So, to deal with your question of whether one can get a complete understanding of the subjects involved in a picture, I would say that it depends upon the simplicity of the subject content, the more complex the subject matter the more vague our understanding. I am, of course, referring to pictures which set out to describe an event, or, in other words, to make a statement about reality. However, as I have said elsewhere, a photographer is an artist in his/her own right, quite capable of constructing a photograph with form and meaning which is a genuine work of art, such a photograph would be open to interpretation ; whether we could properly interpret the photographer's subjective impressions would depend on confirmation by the artist. However, works of art are there to give pleasure or to encourage the observer to think. Photographs of natural scenery, photographs of gardens, could, without captions, say something to us, but if we were curious to know the location we would need a caption.

During the Second World War two of the finest illustrated magazines were Picture Post and War Illustrated. The photography was superb, and some would claim that the impact they produced required no captions. However, it was war, and in wars ships catch fire, bombs explode, guns fire missiles, soldiers are killed, aircraft crash in flames, people lose their homes, etc.. All these things are self evident, the question to be asked is, Where is it happening? During the war we would have been forced to ask, Is it in Warsaw? Is it in Paris? Is it in the Atlantic? Is it in the Pacific? and so on. This I believe usefully describes what I mean by the seeming vague answer to your question — Can photographs engender understanding? In some cases the answer is yes, in others the answer is no.

John Brandon


I do not know very much about photography as a craft; but I do have an opinion about the statement you attribute to Sontag, above. Now, I must also offer the disclaimer that I do not have Sontag's book, so I cannot check the context of that statement; thus, I will be responding only to you, and not, perhaps, to Sontag's argument... unless that does happen to correspond to what you say above.

Ok. How does text have meaning? A good question, and one which has occupied people for centuries. "Deconstruction" is only the latest aspect of the hermeneutics tradition. That tradition started, more or less, in Medieval times as the analysis of the interpretation of religious texts. Hermeneutics has been broadened to include anything termed "text" these days, which, if you are Derrida, at least, does indeed include paintings and photographs (e.g., see his Memoirs of the Blind). The literature on hermeneutics is quite extensive, and it's a field I'm only peripherally interested in... but given all my disclaimers, here's my position on your question: text, as the written word, is just as metaphorical (and I'm using the term "metaphorical" in the general sense that Lakoff et al — see below — use it), undefinable, vague, and infinitely dense as are photographs. My position comes from consideration of Lakoff, Mark Johnson, Mark Turner, and Fauconnier's writings, and others (including Derrida). Now, as far as formal languages go, that text is about as non-metaphorical, i.e., as literal and interpretable as text can be. That is, formal logic, mathematics, computer programming languages, and similar formal languages are indeed completely understandable (or as understandable as we can get... Lakoff actually disagrees with this, and I think he may be right). That's all very fine, but in natural languages like English, we are immersed in an enormous web of metaphorical words, phrases, meanings, and so forth. The result of this is that we must to some extent agree with Quine, at least, as to the web of relationships which define linguistic terms (and the people above go much further than he does). Given that, we do not in any sense understand text more "completely" than we do, say, photography.

There are many counterarguments I could bring up to my own points, e.g., photography is not a language as natural languages are, and thus has no general description (grammar and syntax)... etc, etc... but I could argue against that one pretty easily in terms of visual gestalts and cultural assumptions. And on and on. The upshot is that to defend a position such as that which you attribute to Sontag requires a particular understanding of language which is at this point quite controversial, and which has been strongly argued against.

Steven Ravett Brown

back

Cathi asked:

What is morally objectionable about murder? and is killing the same as murder?

The crucial difference between murder and killing seems to be that killing is socially approved but murder is not. Killing of animals or killing of enemies or killing under death penalty are culturally approved actions. Therefore most people would not agree that "soldiers are murderers". Even "The Sixth Commandment" in the recent translations does not state "Thou shalt not kill" but "Thou shalt not murder" — consistent with the fact, that there is lots of killing even of children, women and elderly under God's approval in the Old Testament. And even the killing of Jesus has been ordered by the High Priest with approval from the Synhedrion (Council of Priests).

Murder is socially objectionable because it is a private deed without consent of the society and it's institutions by law. That cannot be accepted. It's the same difference as between a sworn in surgeon who is maiming you in your hopefully best interest and anybody else maiming you with bad intentions.

There is left a very deep question, posed by Sophocles in his drama Antigone and by many dramatists and writers — and philosophers — afterwards: Is there a difference between what is "morally" objectionable and what is "socially" objectionable" — and if that is the case (as it surely is), then where's the line to be drawn, and why. In part, this marks the division between "law" and "custom" — as in blood-feud or in duelling or in bride-kidnapping, which all have been socially accepted somewhere sometimes, as have been terrible forms of death-penalty. But that leads into the history and philosophy of law and is another question.

Hubertus Fremerey


Murder is, not simply killing, not even unjustified intentional killing, but the unjustified intentional killing of a person. To wantonly kill a cat may be morally objectionable, but one cannot murder a cat because a cat is not a person. Murder is morally objectionable because (a) objectionability varies with the value of that which may be harmed or destroyed by a prospective action (i.e., the greater the value, the more objectionable is any prospective action taken against it) and (b) persons are the highest moral values there are.

Tony Flood


In terms of moral principle, we object to murder on the ground that if it were to be found acceptable, society would be a dangerous place. Murder and killing do differ. Legally, murder is defined as having a mens rea which is the possession of an intention to kill. Without this intention, where killing is in some way accidental, an act of causing the death of another is not murder. In times of war, there is an intention to kill, but this intention is brought about by instructions of a body which is legally empowered to order killing whether we like it or not. But it is not (arguably) as abhorrent as the evil which is present in an individual murder. David Hume thought that there is nothing that we can point to in a murder which makes it objectionable. We just don't like it. I think this is wrong. There was evil in the mind of the person and even if we can't see into the mind of the person, evil was there. And really, it is not just a moral principle regulating a safe society that makes murder wrong.

We might say that manslaughter, or accidental death, and killing in war are "evils", as many things that occur in the world are, such as death caused by volcanic eruption. These are events or features of the world. This is a way of describing the facts in the world. When we say a murder is evil, we are describing a fact, but also the state of mind of a person. It is not just that the fact is evil, but that the man embodies evil which makes it morally abhorrent.

Rachel Browne

back

Richard asked:

Is it possible to articulate an object or rationale for the telos of being a human?

You seem to be asking this in the context of Aristotle and Plato... if that's your approach, you have, I assume, read Heidegger and his followers on this (if not, he's the one to go on for a modern treatment of this). There's also the strictly religious point of view: de Chardin, for example. But you might also try Dewey; he has some interesting things to say on this subject from a rather different perspective. Modern positions on naturalized ethics also attempt to answer this question... I'm not too familiar with this area, however. There's Johnson's Moral Imagination, but that's only one of many.

So in answer to your question, there are very many people who believe that it is possible to articulate the telos of being human, ranging from the religious to the phenomenological to the postmodern to the naturalized; just pick your poison. I'm certainly not going to try to summarize all those positions here.

Steven Ravett Brown


First ask yourself if you want to be called a robot or an "intelligent animal". Then think of what may be the difference to being "a human". The idea of Aristotle's was to bring out the best possibilities of any being, and that he called "virtue". So his question was: what are the virtues that make the difference between "man-kind" and (for instance) "ape-kind" or "horse-kind". But of course, nobody can hinder you if you are satisfied being a happy ape. You must find out for yourself if that is "a rationale for the telos of being a human".

The next question would be, what exactly does it come to "being a human". That is one of the deepest questions of philosophy, that neither Aristotle nor Aquinas nor Kant could answer convincingly. It's one of the great open questions of the philosophia perennis. The greatest danger in our time is to mistake man for an "intelligent ape" or a "smart robot". Perhaps you should read F.R. Leavis The Great Tradition or C.S. Lewis, but all the best philosophy and poetry has been and is concerned with this question.

Hubertus Fremerey

back

Arthur asked:

A problem arose to my mind while reading about the 2nd law of thermodynamics, it says that "the overall disturbance in a closed system must increase or at least remain constant but never decrease" but the problem here is that the "disturbance" is something to humans, WE think that scattered billiard balls are disturbance and think that when they take the shape of a triangle (at the beginning of the game) they are not disturbed, so how can something related to humans take place in an impersonal physical law?

Entropy measures the disorder of a system. This term is not a term expressing approval or disapproval it just refers to the fact that e.g. molecules in ice are tighter packed than in liquid water, which in turn has a higher level of organisation compared to gas molecules in steam. This difference in the organisation of matter can be objectively measured (e.g. via the energy levels and the properties of the system) i.e. is not dependent on human beings. The 2nd law simply says that the entropy (disorder) of the universe overall increases. [If the volume and energy of a system are constant, then every change to the system increases the energy].Things get broken, living beings die and decay ... Whenever you want to create order out of disorder you need to put energy into it (e.g. it is easy to break a cup but difficult to put it back together), and part of that energy is irretrievably lost as heat, thus increasing the overall disorder of the system.

Helene Dumitriu


This is actually sort of an interesting point. The term "disturbance" is indeed an odd one, to me at least. I would have said something like "smoothness", "randomness", or "uniformity". But aside from that, one could still object that we perceive certain patterns or types of patterns as order, and certain other patterns as disorder, and so this distinction is a purely human one.

There are a couple of ways around this, however. First, there are only a small number of ways that billiard balls can be put into a triangle, and many hundreds or thousands of ways they can be scattered over the table. One could say, then, that what we are actually comparing is the probability of them being in the relatively small number of distributions we term "ordered" versus the huge number we term "disordered", and so that probability governs the frequency of those distributions actually seen. Not unreasonable, but we can still ask why some subset of the "disordered" set is not perceived as a distinct subset, as the "ordered" sets are, i.e., whether there is any real difference between what we perceive as ordered and what we perceive as disordered.

I wondered this myself for quite a while, until I saw the results of the work of G. Chaitin. What he did was to show that there is a difference in the minimum length of formulas which describe various states of affairs, mostly in formal systems. But this is an interesting result in this context, because we can see that symmetrical groupings will have smaller descriptions (because of the identity of rotations) than asymmetrical and chaotic groupings. Those latter will have to be described by larger formulations, because we can't abbreviate for the symmetry in them. Now, no matter who is doing the describing: us, aliens, mother nature, or whatever, that's going to be true. So that is, I think, a nice way to relate more and less random distributions in some reasonably absolute way, and leads to the same types of classifications that our intuitions do, which is rather heartening metaphysically, when you think about it. The interesting implication is that description has to do with composition, a very strange result for me, at any rate... but consistent with the Second Law, it seems.

Steven Ravett Brown


I am not a physicist, but it seems to me that you are too bothered by the term "disturbance" which may just be used in a technical way by physicists.

Ken Stern

back

Jeremy asked:

What is faith, in the context of our life-view? If our actions are based on what we believe (about God, our nature, the afterlife, etc.), then what is the basis of our belief? It seems that faith goes beyond knowledge. To know and to believe are quite different, although they affect each other. Is faith merely empirical, or is it based on more than evidence?

If you sit down on a chair — is it by "faith" or is it by "knowledge"? What then is knowledge and evidence? You cannot in earnest "suppose" God to be, like you cannot in earnest "suppose" the chair to be — that would make you crazy. Of course you can say, as Laplace said to Napoleon when asked why God didn't show up in his great work on astronomy: "I didn't need this hypothesis". That's what most people in practice think on the existence of God — they don't need this hypothesis. But you cannot build cathedrals and you cannot roast martyrs on hypotheses. For the true believer God is not a hypothesis but an experience (as I myself know "by experience") like trusting in your mother or in your best friend. "If you cannot trust then you have to bust" — there's no choice.

Note a twist logic in this: You cannot "have to" believe. If you "have to" trust, then you are not trusting anymore. If you "have to" believe, then you don't believe anymore. You never "have to" believe that the chair you are sitting on is real. If you cannot accept the reality of God in the same way as the reality of the chair, you are not believing in God. That's the content of "you cannot build cathedrals on hypotheses." The whole of Christian culture could not have been built on a God taken merely to be a hypothesis. But of course that doesn't exclude the possibility that God is just that — a hypothesis and not more. For Othello to strangle his wife it is not relevant if she is indeed deceiving him. It suffices that he thinks so. In this sense "faith goes beyond knowledge".

But what then is "knowledge"? Ask Sir Karl Popper.

Hubertus Fremerey


Philosophically (that is, theology aside), faith is the fundamental belief underlying and underpinning the formation of all of one's other beliefs. It is a belief about the ultimate intelligibility of the world, which is always more unknown than known, such that our effort to understand it is not an exercise in futility. Even to ask seriously whether it is such an exercise is already to presuppose that it is not. Faith that the world is ultimately intelligible is not a blind faith: it simply has no coherent alternative. Having no coherent alternative, it is rational, but not empirical: it is not discovered empirically, but all empirical inquiry presupposes it. It is known a priori. On the basis of this faith, we judge as reliable sources of information that we do not, cannot, verify for ourselves (e.g., a newspaper, or a calculator). We can, for example, personally verify that we read the newspaper this morning, but not what it reports. The latter we judge to be probably true because we made a prior judgment about the newspaper's reliability. We act on the basis of such judgments, sometimes verifying what we had only believed, and we report our experience to others, who do not verify it but regard us as a reliable source of information. The progress of human knowledge rests on this collaboration, the mutual "affecting" of knowing and believing that Jeremy refers to, the support that value judgments and factual judgments provide each other.

Tony Flood

back

Jim asked:

Does the existence or non-existence of a god have an effect on our morality? That is, if today god exists, and I do wrong, and bad, and then tomorrow god does not exist, and I do wrong, and bad...do the actions of today become more wrong/ right with god existing; and do the actions of tomorrow become less wrong/ right with god not existing?

The law has an answer. As the Romans said "nulla poene sine lege" — if it is not forbidden by law it cannot justifiably be punished. And it should not either, because then the world would be a world of arbitrariness and despotism. But there's another principle stating that "Ignorance of the law does not prevent punishment (if you have violated the law)".

The very core of your question is: What do you mean by "doing wrong"? Are you acting against written law — then see above. Are you acting against your conscience, then the existence of God is irrelevant here, because your conscience is yours, you must come to terms with it. If you are bleeding, then stop it, but don't ask if God is looking on. That's simple. But if it really matters if God exists or doesn't for to do what you think is right, then you have a bad character and are trapped in a false logic.

What the better among the theologians mean (there are bigots too) is not that you should act out of fear and trembling, but that you should act out of love and a sense of honour as one of God's children — if ever God means anything to you. Then some ways of acting, that are not forbidden by law or custom may be forbidden by love and honour with respect to God or to the very fact of being a thinking human and not a dumb animal — and not a coward.

Hubertus Fremerey


It should first be noted that Jim did not ask the traditional question, "Does God's command or prohibition make an action right or wrong, or is its being right or wrong the reason for God's command or prohibition?" I'm not sure what Jim means by "today god exists . . . and then tomorrow god does not exist", but I do know what it means for someone to affirm God's existence and deny it the next. If one believes, for example, that adultery is wrong, he will probably still think it's wrong whether he went from theism to atheism or from atheism to theism. The answer to Jim's question depends how reflective a given person chooses to be. Any worldview, theistic or atheistic, will ultimately have implications for how we think about morality, and vice versa. Our particular moral judgments may survive our conversion from one worldview to another; they may even be the motivating reason for converting. Many people seem more tenacious about their moral judgments (e.g., for or against capital punishment, for or against abortion) than about the set of consciously held judgments that is their worldview. The more reflective among them will therefore be vitally interested in ascertaining how well any proposed worldview supports their moral intuitions. But the latter tend to survive whether or not a suitable worldview home is found for them.

Tony Flood

back

Reg asked:

I try to keep any philosophical thinking I do as simple as possible. So I'm not convinced that the brain 'thinks'. Matter or extension or any physical object can do but one thing viz. move; all the rest is done by a mind which of course is non-physical. I'm not convinced that the relationship can be solved by 'going and looking' which, on analysis, is what scientists do is it not?

Simplicity is certainly a virtue, but not when it is too simple for the facts.

Your argument that the mind is non-physical seems to be to beg the question in that it assumes what needs to be proved, namely that "matter or extension or any physical objects can do but one thing viz. move. For instance, here is a counter-example: the mind is a physical object and it can do more than move, for it can think. For as the philosopher Thomas Hobbes asked Descartes, "Why cannot a physical thing think?"

You are simply intuiting a certain concept of mind, and then arguing, on the basis of that concept, that it is not a physical object. That is an instance of what the philosopher Antony Flew calls "an ostensibly counter-evidential intuition" for all our evidence is that the mind is a physical thing.

Ken Stern


You are right in identifying one of the great puzzles of philosophy — how mind and matter interact. However there seems to be an inconsistency: You want to keep it 'simple' yet you assume (if I understand correctly) two 'substances' i.e. 'mind' and 'matter' (the physical brain). That is less simple than idealism (everything is 'mind stuff') or materialism (there is only matter). Also if you really are a dualist you have a problem: how to explain the interaction between mind and brain/ body and vice versa? In other words how can something of an immaterial nature interact with something material? Also how to explain that various types of physical brain damage can rob you e.g. of your sense of smell, your memory, your emotions etc.? How to explain that drugs can make you happy, sad, indifferent, psychotic etc.? How to explain that when you have certain thoughts certain brain areas are active? How to explain that you can learn to relax by watching the wave patterns of your brain on screen?

Many philosophers today think that the 'inside' or first-person view of 'mind' and the 'outside' or third-person view of 'brain-functions' are like two stories we tell about the same thing.

Helene Dumitriu

back

Suyen asked:

What is personhood? What is the definition of human life? What is the value of life? Do the neonate and infant have personhood? How about the disabled neonate and infant, are they not as valuable as the normal neonate and infant?

What do you mean by "as valuable as the normal neonate and infant"? We should be loyal to all humankind by the Golden Rule. But for an employer the value of an employee is in his abilities. In our output-driven culture there is an conflict then: People should follow the image of being strong and fit and active and youthful, they should be winners. But then what to do with those 90% of mankind, that definitely are not winners? Since there never will be a clear cut definition of what is a real winner, one always has this relation of 10%: 90% winners — or maybe 1%: 99% — as you like. Therefore socialism tried to get rid of this altogether. That's the core of stressing equality.

This conflict translates to the neonates: What do you expect them to be — lovable humans or failed winners from the start? You can't evade that question. That's the core of the quarrel over PID (pre-implantation-diagnostics), the question, if foetuses should be screened after conception to select and remove those with handicaps.

There is one more large aspect to this: We live in the epoch of "reflected enlightenment". Reflected, since the naive enlightenment has been destroyed by people like Hitler or Stalin. But even the "reflected enlightenment" clings to the idea of improving all things using science and technology. But why not improve men? Every teacher and coach claims to do just that. So why not improve men using genetics and selection? There you are again at "not as valuable as the normal neonate and infant". You never can't evade that aspect of life as a contest if you speak of "improvements".

Now you understand why the churches have been against "Enlightenment": If men don't accept good and evil as coming from god, disabled neonate and infant as valuable as the normal ones, if everything is looked at as "to be improved by science and technology", then the disabled are in a lost position. You have to make up your mind on that, no philosopher will free you from that dilemma.

The other questions are (1) What is personhood? (2) What is the definition of human life? (3) What is the value of life? (4) Do the neonate and infant have personhood?

They all have spawned meters of bookshelves already and continue doing so. Maybe I will answer to those questions another time — or somebody else will.

Hubertus Fremerey

back

Ronald asked:

Assuming God is omniscient and omnipotent, what did he sacrifice by allowing his son to be crucified?

God's having a son, etc., seems to be as much an assumption as is God's being omniscient and omnipotent, but Ronald's question treats it as a fact. The assumption that God is omniscient and omnipotent is purely conceptual and can be considered apart from theological interpretations of alleged historical events. That is not the case for the assumption that God had a son and sacrificed him. The philosopher, as such, has no special competence to ascertain historical events or assess their theological significance.

Tony Flood

back

Bill asked:

I am perplexed by the field of economics, because the conclusions drawn by economists seem to depend on whether they have an "R" or a "D" after their names (i.e., what political party they belong to).

Is economics really "science" — or is it politics in disguise?

As individuals, economists can favor one economic policy over another without compromising their scientific credentials if they keep their public and private roles distinct. Economists will say that if you want economic result ER, then you ought to implement (or dismantle) economic policy EP. As economists, however, they have nothing at all to say about the short- or long-term desirability of ER. When, for example, Milton Friedman says, "We ought to have a more libertarian society," or if John Kenneth Galbraith says, "We ought to have a more egalitarian society," neither is wearing his economist cap.

This also holds for the political scientist with regard to political result PR or for the military scientist with regard to military result MR. (There is, for example, no basis for imputing pro-Nazi sentiment to a military game enthusiast simply because he can show how Germany might have won World War II had its military forces implemented a different strategy or different tactics.) All three sciences presuppose that human beings use means in order to achieve goals, but they prescind from evaluating those goals.

Taking early-20th-century physics as the paradigm of science, however, the dominant philosophy of science regards economics, military science, political science and other "social studies" as sciences only by courtesy. According to that paradigm, "real" science presupposes materialism, mechanism, and determinism and, as we all know, human action notoriously resists deterministic, mechanistic, or materialistic interpretation. Sciences of human action can therefore be, at best, "soft," the insinuation being that they're not really sciences. It may be, however, that some degree of self-determination occurs, not only at the human level, but also at the organic, chemical, molecular, and even quantum levels. It may be that the apparently deterministic laws of one level are in fact habit-like expressions of a lower level's indeterministic, nonmechanistic, nonmaterial processes. If those things are so, and there is evidence that they are, then that paradigm's days are numbered. It has been decomposing over most of the last century, and one of the consequences may be the revelation of the so-called "soft" sciences as holding the key to the unification of science.

Anthony Flood

back

Matthew asked:

Would Hume agree with Descartes that God must be the cause of my idea of a perfect being? why or why not?

No. Because Hume would not have agreed that anything must be the cause of anything. (In fact Hume wrote that so far as we can know a priori (without any evidence) anything might be the cause of anything else.) And, of course, all of our evidence tells us that it is man who is the cause of all his ideas including that of God.

Ken Stern

back

Damon asked:

What are the main objections the Kant's categorical imperative?

I don't know how to answer this. "The" objections? From whom? Well... I'll give you a couple of my own. First, as Kant acknowledged, the general rule, "act as if you would will the maxim of your action to be a universal law", does not, purposely, relate to any specific situation, i.e., has no "content" (a technical term in Kant). But then all you're left with is a general rule... now what? In other words, how do you apply the categorical imperative to, say, the assembly line in Ford Motors? Whoops... nothing said about that in Kant.

Second, Kant arrives at this rule by explicitly accepting societal norms as the basis for ethics, and then abstracting from them. Well, why do that? He says that since they're widely accepted, they must have some truth to them. Ugh. I hate that kind of reasoning. The world had to be flat, by that reasoning... the sun went around the earth, etc. So that's an even more general objection than the above. I mean, for someone who called, early in his career, for "reformation" based on rationality to then use the above reasoning as a principle for deriving an ethical rule just seems very strange to me... not to mention being basically, as I see it, unjustifiable.

So, even given that you can get around the first objection, the second seems much more severe anyway, and I don't see a way around that one, except for the obvious: "gee, you have to start somewhere...". Well, sorry, but I think there are better ways to start. I'm in favor of naturalized ethics, myself.

You might take a look at M. Johnson's Moral Imagination for more on this.

Steven Ravett Brown

back

Simon asked:

Hello, I'm 15 and there is an unanswerable question on which I would like a philosopher's view. If an object were to fall to the ground, where the object started would be "1", and where the ground is would be "0". Starting at "1", the object falls half of the way... then one fourth of the way... then one eighth of the way... then one sixteenth of the way, and so on as the number doubles. The question comes down to, when is the fraction so small to call "0"? if as learned in school there is an infinite number of fractions, if this is true the object never becomes "0" and never hits the ground. Please explain this.

Simon's (and, before him, Zeno's) puzzle falsely presupposes that the finite distance to be traversed is a continuum of extensionless points. The potential to double the denominator of a fraction may be infinite — you can double that number forever — but the number of places to be traversed by a moving object is not infinite, and so the falling object does hit the ground. To put it another way, no falling object can reach the ground if it must traverse an infinite number of places to get to its resting place. It can't even leave the place it is supposed to be falling from. Fortunately, a moving object traverses a finite number of discrete (separate) places, not an infinity of points on a continuum.

Tony Flood


Well this is a neato version of Zeno's paradox; look that up. There is no fraction so small to call "0", that is the formulation of the paradox. As you can see, when you drop an object it can never reach the ground. It's the same for reaching any destination, and that's why when you shoot an arrow or a bullet at someone, you never can quite hit them.

Whoops... there's a problem here, isn't there.

So should I tell you the answer? No... that would spoil the fun. Just go look up Zeno's paradox, and you'll find tons of stuff written about this. But I'll say this; there are different types of series of fractions: convergent and divergent. If you add 1/2 + 1/4 + 1/8... etc., you get, after an infinite number of additions, the sum of 1. But if you add 1/2 + 1/3 + 1/4... you get, after an infinite number, the sum of infinity. Something else for you to play with.

Steven Ravett Brown

back

Rick asked:

What is the connection between Nietzsche and Social Darwinism, Nazism, and eugenics? What did Nietzsche have to say about the principles these topics embody?

Nietzsche's working life spans the 15 years from 1873-1888. This was a time of a great boom in Germany, after German troops had won the wars against Austria (1866) and France (1870) and Bismarck had founded "The Reich" (1871). So Nietzsche could have been contend with those achievements as were most of his fellow Germans. But Nietzsche despised this gold rush in a similar mood as did Marcuse 90 years later: There was much gold around, but this was devils-gold, since the soul of men has not been freed but hardened and disabled for the true life of love and joy. This was the core of Nietzsche's critique.

Nietzsche was no Social Darwinist, though — and no Nazi either. The Nazis even understood that. Nietzsche admired Schopenhauer for his daring thesis, preceding Freud, that man is driven by his voluptuous desires of all sorts, mainly sexual. Schopenhauer was one of the first philosophers in Europe allowing sex and the theory of the Buddha of the human suffering from greed and longing to enter his philosophy. But while Schopenhauer's world-view got pessimistic, Nietzsche thought, there could be an optimistic equivalent: Where Schopenhauer following the Buddha said "no" to life and advised renunciation from all clinging to its cheating beauties, Nietzsche said (tried to say) "yes", even "a great yes" to life and to "the will to live". That was his programme.

And now you understand why Nietzsche didn't think much of socialisms of all sorts — including the Marxian and christian versions of his day — and would have despised the nationalistic versions of socialism, as Mussolini and Hitler propagated them, likewise. His aim was a society of free men and not a society of happy sheep, as he took the socialists ideal to be. But "free men" for Nietzsche did not mean so much business people and entrepreneurs but a new sort of the Renaissance "uomo universale" exemplified in Leonardo da Vinci. Nietzsche was in Basel as a professor befriended to his colleague Jacob Burckhardt, who had published an up to this day famous book on the Renaissance in Italy, and Nietzsche himself was professor of Greek Antiquity. So Nietzsche's idea of the free man had nothing to do with free entrepreneurship and nearly nothing with Marcusean communes or with Hesse's "Siddharta" either, but with a free and able personality as was the ideal of the Greek and the Renaissance.

I don't think he openly would have spoken in favor of "active" eugenics, but he would have favored "passive" eugenics saying "let the disabled and weak die." There always has been eugenics from the oldest times and with all peoples and mostly passive. But you see that one could understand and defend Nietzsche's position without favouring active eugenics.

Hubertus Fremerey

back

Roy asked:

What makes a definition correct? Is the contemporary definition (what most people assume the word to mean) a higher priority than a contextual definition? Some people are clueless that words can be defined by the context they are written in. The same people argue that speakers of the language make up the contemporary definitions of a word and that is the final definition.

For example, to most native New Yorkers (where I am from) an 'Argument' is simply a disagreement. When I point out that arguments should be followed by reasons and a conclusion I get criticised — who am I to correct them?

Another example in a college environment. My former speech professor defined and distinguished an 'art' from a 'sport' to his class; he stated that in a sport the consequences are paramount, whereas form is paramount in an art. For example gymnastics is not a sport but an art since form is part of a gymnast's score; a person who falls off of the balance beam scores lower than the one who does not fall. In a sport like basketball form is irrelevant to scoring a basket; one can perform the sloppiest lay up and fall on his face, but if the ball goes in the basket 2 points are scored. Meanwhile, another player might perform a perfect layup or the world's greatest slam dunk in history and also receive 2 points. The distinction makes sense, but is still rejected by many people — at least in New York City — who know what the Olympics are. Many people believe that all Olympic events are definitely sports. When told 'Gymnastics is not a sport' people get upset and argue that whatever the current authorities say is a sport is a sport. How does one solve this kind of problem?

I favor intellectual definitions (extensional, intensional, lexical, range, operational, theoretical, etc); whereas other people appeal to authority — whatever the boss says is right to me.

Going to a dictionary is a good first step to discovering what a term means, and dictionaries are places for authoritative definitions which are based on how native or fluent speakers of a language use a term. It is a "first step" (but still a good one) because there may need to be further explanations refinements and further distinctions not covered by the dictionary but needed for philosophical purposes. The philosopher may, for instance, be interested in distinguishing between believing and knowing is a more elaborate way than could be done by any dictionary. The case you give of your speech professor's distinction between art and sport which had the consequence of gymnastics not being classified as a sport is another illustration of this refinement of ordinary meanings for philosophical purposes. When your professor did this he was pointing out the similarities of gymnastics to something the performance arts like ballet and playing a musical instrument, and suggesting that it was more like those actions than it is like baseball or like football. And this seems true to me, although for other purposes gymnastics is thought of as a sport since it is something which involves a formal competition. The world is more complex than our classifications, and some things can be classified in many different ways. Why not simply say that although for the purposes of the Olympics gymnastics is a sport because it is competitive, it also has stronger similarities to art than most other sports?

You should remember that words may (and often do) have more than just one meaning. Your argument over the word "argument" illustrates this. In most logic texts you will find some mention that the term "argument" as used in that text (and in the context of logic) is not a synonym for "dispute" or "altercation". (I don't think a disagreement is any sort of argument, although disagreements may give rise to arguments in the sense of "altercation.")

Ken Stern

back

Charlene asked:

How can a great philosopher like Descartes, believe that existence is shared when we don't share the same god in the C20th? He even believes there can only be one god!

Descartes defines God as the supreme Being which possesses all perfections, a Being which cannot be conceived to be more perfect. Now if there was more than one god it would obviously be possible to imagine these gods combined, rolled into one — this one God would be 'more', more perfect than the two gods. Hence Descartes would argue that multiple gods are not God in the sense of the supreme Being. So it seems that if you want to believe in God you have to either give up the claim 'God is perfect' or the claim 'there can be more than one God', you cannot have both since they contradict each other.

Helene Dumitriu

back

Narendra asked:

What is happiness, joy, pleasure in life how one can get it? as this life is full of sorrow, unhappiness and painful in all ways.

I think you are aware that there are lots of books in your next bookshop or library selling just what you want: "happiness, joy, pleasure in life". One of the bestselling authors is Joe Murphy, but there are really a lot of them.

But then: What did the Buddha offer or what did Jesus offer? They offered the joy of freedom and love, the end of all sorrow, but they didn't talk of "happiness" or of "pleasure in life". So think for yourself what makes the difference. And why do some people vow "poverty, chastity, and obedience" without being forced to do so? There seems to be more to life than just "happiness and pleasure". Why do some people sail around the world or climb the Mount Everest — risking their lives? Why do some people engage in charity and work for others like Florence Nightingale?

Much of sorrow, unhappiness and painful feelings come from all sorts of ego-centrism, of too much interest in ones self and to much neglect of the world and the people and the tasks around. And that was, what the Buddha and Jesus were teaching: open your heart and mind to the world and to the things "here and now" and to your fellow men and to all flowers and animal and stars around you.

That even has a world-historic aspect: First came St. Francis (ca. 1181-1226), preaching to the birds and singing the light of the sun. Then came the gothic cathedrals of Paris and its environment of the 13th century with their new light and grandeur. Then came the Renaissance with a new look on the beauty of landscapes and animals and plants and women and men (Jan van Eyck, Durer, Leonardo). Get a good history of art with many pictures and look for the chapters on Gothic and Renaissance.

"Open heart and mind" — that was the idea behind Athens in the time of Socrates and Pericles, that was the idea behind Rome in the time of Virgil and Augustus, that was the idea behind Paris in the time of the Impressionists and that was the idea behind the USA in the time of the Marcusean Revolt.

Try to revolutionize your life too by having a fresh look on the world.

Hubertus Fremerey

back

Trevor asked:

I'm afraid that despite all attempts to the contrary, I'm doomed to be a writer, and am currently embroiled neck-deep in a project concerning William Shakespeare. If you have a moment, any and all philosophical insights and advice relating to the man and/or his works would be most welcome and greatly appreciated.

I'm not sure if this counts as a philosophical insight, but it might be interesting to read Hamlet while concentrating on the fact that the play is set during the protestant reformation and specific mention is made of the fact that Hamlet has just returned from studying in Wittenberg. There seems to me to be quite a bit of tension in the play that arises from the Roman Catholic/ Protestant disagreement over the concept of mortal sin. I don't remember ever seeing a commentary that makes mention of this.

Lance Flowerree

back

Greg asked:

I am currently writing a paper on implications of comparing "cogito ergo sum" and "vollo ergo sum" (i.e. I think therefore I am and I will therefore I am).

There are obvious implications for the concept of Cartesian dualism in that "will" does not allow for separation of mind and body.

Is this right? Are there other possible implications I should take into account?

Focusing on Descartes two comments: The statements 'I think' and 'I will' or 'I desire' both entail 'I am'. In other words any state that I am in entails that I must exist in the first place. For Descartes there is a difference between the two in that my intellectual grasp, my understanding of things is limited since my knowledge is limited (since I am a finite being and not God), but my will is unlimited. In the 'Meditations' he gives examples where the will is strongly inclined and the self assents to a proposition e.g. the cogito, or in the absence of perceiving things clearly and distinctly i.e. in the absence of proper understanding or knowledge there is 'indifference'. Considering his scenario of the self as a 'thinking thing' assenting to propositions or refraining from judgement one could argue that these are instances of 'will' in action that do not necessitate a body.

On the other hand one could argue: Show me the will to sit down without actually sitting down. Also Descartes' concept of will has been questioned e.g. on the grounds that there seem to be cases where we are not free to 'will' to believe something: Can a wife 'will' to believe her husband is faithful, if there is overwhelming evidence to the contrary? Could Descartes' thinker choose not to believe the cogito?

Helene Dumitriu

back

Tom asked:

What right has Lacan to apply the Hegelian Master/ Slave transcendental pattern to the empirical therapeutic situation?

Peter Dews in Logics of Disintegration says that Lacan was influenced by Alexandre Kojeve's Introduction to Reading Hegel in which Kojeve interprets Hegel's Phenomenology of Mind as being based on the core position that self-consciousness cannot emerge without a relation to another desiring subject and this led Lacan to the idea that what was wrong in Freud's thinking is that he was a victim of psycho-physical parallelism and that his theory of mind characterised man as a solipsistic being who could become self-consciousness alone. Given this, I suppose that in the therapeutic situation the part played by the therapist, who is another determining subject of the patient's consciousness, becomes emphasised. Consciousness is recognised as precarious, there is more to reality than speech, and within this reality there are relationships of dominance, and alienated desire. This is not a matter of "right", but of speculative theory being influential in the empirical situation, which is how psychoanalysis works.

Dews also finds that Lacan was influenced by Hegel's relation between consciousness and history and social conditions and Hegel's view that beliefs do not reveal truths about ourselves. If Hegel is correct, and Lacan is purported to have believed him to be so, then there is justification in bringing philosophy into empirical situations. However, I notice in The Four Fundamental Concepts of Psycho-analysis at the end of the paper on alienation, that Lacan appears to deny this influence of Hegel. It is not the thesis that the subject is constituted by that which is external to him whereby Lacan was influenced by Hegel, but in the alienated consciousness of the self, the duality of consciousness and unconsciousness in which Lacan claims Hegel influenced him.

And the nature of an emerged self-consciousness, with the alienation of the self behind a veil, probably doesn't have so much part to play in the empirical therapeutic situation.

Rachel Browne

back

Luiz asked:

I am teaching a course on Education in a postgraduate program in Music.

My question is: would you please advise me on some texts (and authors) on the subject of Philosophy of education?

As this branch of philosophy overlaps other main branches of philosophy, especially ethics and epistemology, but also logic and even metaphysics, the selection of suitable literature will depend on what special aspects of philosophy of education you are interested. Here are some suggestions:

If A.N. Whitehead is right saying, Western Philosophy consists of a series of footnotes to Plato, then Philosophy of Education maybe described as a special case of a series of footnotes to Plato's dialogue Meno. There you will find all the fundamental issues such as whether virtue can be taught, what virtue is, what knowledge is, what the relation between teaching and knowledge is and how and whether teaching is possible.

Whitehead himself wrote The Aims of Education and Other Essays (originally published by Macmillan, 1929). An e-text of the first chapter is available at: http://www.realuofc.org/libed/white/aims.html.

Perhaps one of the most influential contributions to the development of educational thinking in the twentieth century was made by John Dewey in his Democracy and Education. An introduction to the philosophy of education.

A more visionary attempt was done by John Amos Comenius about 1630 in his Didactica Magna, usually called The Great Didactic. Perhaps a more meaningful translation would be "The Whole Art of Teaching." It explored how people learn and how they should be taught from infancy through the university and beyond. Comenius addressed such topics as education for everyone, career preparation and lifelong learning (not for nothing one of the educational programmes of the European Union is called Comenius!).

One of the most popular books on Philosophy of Education probably is Jean Jacques Rousseau's Emile. Rousseau's ideas about education have profoundly influenced modern educational theory. He minimizes the importance of book learning, and recommends that a child's emotions should be educated before his reason. He placed a special emphasis on learning by experience.

As a teacher in postgraduate programmes in music you might especially be interested in this one: Music Matters: A New Philosophy of Music Education by David J. Elliot (view details with excerpts at http://www.amazon.com).

If you wish to dive deeper into this subject you might find some of the links at Research Guide: Philosophy of education useful.

Simone Klein
http://www.sophiasworld.at


Begin by looking up Aristotle, Rousseau, Pestalozzi, Froebel, Montessori, Dewey, Makarenko, Tagore, A.S. Neill and Goodman for a start at the UNESCO link Thinkers on education (the texts are in PDF requiring the Acrobat Reader).

And then there's lots of literature! But I think you should be acquainted with the names indicated above before entering any "Philosophy of Education" book. Those books give the grand picture, but you should have some hold on those great thinkers on education to get a feeling of what this philosophy is all about. And then you have to connect it to your own past experience as a child having a home and as a pupil going to school.

Think of what has been good and what has been not so good in your experience — and why. What would you suggest to improve — why? Then you are bringing in your part of "philosophy of education".

What is philosophy of education about? It's like medicine and psychotherapy about coaching and mentoring children and pupils to become "as good human beings as possible". But what does this mean? If you have to coach a tennis player or a piano player, you have an idea what the outcome should be. The utmost aim would be to make her/ him a star. But what is the aim when coaching somebody to become "as good human being as possible"? That's a very deep question and the core of all philosophy of education that already perplexed Socrates and Plato.

And once more: If you had to educate your mom and dad or your teachers or your friends — what would you want to improve in them, why? And what would you want to improve in yourself — why? This "why" is the starting point of all "philosophy of education".

Hubertus Fremerey

back

Dina asked:

Is there morality among the animal kingdom?
Do animals have morals?

Roger Scruton says in Animals Rights and Wrongs that animals don't have morals in the sense that we have in that they don't belong to our moral community. We invite pets into our moral community, treating them as humans, but they can't reciprocate. For sure, animals don't recognise duty and principle, they don't feel remorse nor (though I'm not so sure about this one) empathy. We treat our pets like persons who have rights, requirements and needs, but they don't treat us like that. On the other hand, I think that we love and honour our pets, and they do reciprocate in this respect. But this is just pets, not animals in general.

The question is, what is fundamental to morality? It could be belonging to a community with a code of behaviour which would mean all animals have a moral code (albeit not explicitly known to them perhaps), or the foundation to morality could be love and honour, and in this case pets might be said to have a basic ethical attitude as well as a code of behaviour proper to their species.

The philosopher, Martin Buber (Between Man and Man) believed that the ethical relation, which he called the "I-Thou" relation could be entered to with all animals, and not just pets. He claimed to have entered into an I-Thou relation with a horse, recognising the otherness of the horse and feeling the horses's approval as he let Buber approach to stroke his mane. Buber also felt in the glance of a cat "the language of anxiety". He said "the eyes of an animal have the capacity of a great language". Communication with animals, the recognition of their approval or anxiety is a reason for us to treat them morally and it is not just a matter of sentimentality about pets. That this communication exists indicates to me that were greater communication possible, i.e. greater understanding on the part of the animal of the nature of humans, we might expect a developed ethical attitude of concern to develop. But for sure we can say that animals do have the fundamental capacity for the ethical attitude towards humans.

A lot of animals care about one another, feeding their young, sticking together in packs, sleeping together for comfort, licking one another's wounds. If the attitude of the mother and the healer, and the ability to live communally indicate a moral kingdom, then yes, in this other sense there is morality within the animal kingdom.

Rachel Browne


What is the meaning of "morals"? What makes the difference between moral and im-moral? Since animals cannot see that difference (why?), their behavior cannot be "moral" — and by the same argument cannot be "im-moral" either. Do babies have "morals"? Do madmen have? In the Middle Ages even an animal could be sentenced for "false behaviour", since it had violated the order of God and nature. But even this we would not call "im-moral" but "im-proper" behaviour in an objective sense. The dragon has to be slain even if he is only displaying his nature as a dragon and could not be in any way "im-moral". Some people look upon mass murderers or upon people raping children as upon dragons or beasts to be slain.

To be moral means to have options to behave this way or that and to be able to justify your choice by some "moral" value you adhere to. So if a child decides to have the lolly offered to him because it tastes sweet, this is no moral decision, but if it deliberates to steal the lolly from a shelf or from another child then it is. The first is only a "preference" by taste, the second is a (false) preference by moral standards. But a standard is something somebody holds it to be. So who is "somebody" in this case? That is "the moral community" or your own conscience, which is in part (not all!) a copy of the former. Your conscience is not completely a copy of the standards of "the moral community", since you may (and should) have your own thoughts on some values. Rebels are rebels because they do not conform in some important points to the standards of the moral community they live in. Sometimes — as in the cases of Socrates and Jesus and Antigone and Morus and Bonhoeffer and many more — this is good for the moral community, but often — as in the case of a murderer or a liar or a thief — it is not.

There is some sort of "moral intelligence". Little children think "bad is everything forbidden" or "good is what mom and dad said". Later on they learn that mom and dad may be wrong by thinking or by doing. And still later on they may be thinking that even the teacher and the priest and the Prime Minister may be wrong by thinking or by doing. But look up Lawrence Kohlberg on this. Kohlberg was a pupil of the child-psychologist Piaget of Geneva, Switzerland. Kohlberg is famous for his theory of "Six stages of moral development". You get at all relevant information on the life and theory of Kohlberg in the Kohlberg Tutorial

Hubertus Fremerey

back

Brett asked:

Does the theory of natural law offer something to ethics which the divine command theory does not or cannot?

If it could be demonstrated that there are natural moral laws such laws could rationally convince everybody (everybody who is thinking rationally that is), not just followers of one particular belief. Also any theory of divine command has to answer the following questions: Is something morally good because God says so, or does God say so because it is morally good? In the first case this would mean that God could theoretically decide any action to be morally good, so it becomes really a question of power? In the second case the action is morally good whether God agrees or not. So where does this moral goodness come from?

Helene Dumitriu

back

Damon asked:

In existentialism a main concept is, 'to exist is to be free'. I need confirmation on this. Does this mean to exist in a rational way by combining thought, feeling, and action? This I assume would leave out infants.

Yes of course, infants cannot be free in this "Sartrean" sense to decide where to go from here and now. In every moment you are "here and now" and not determined by your past. If you are wealthy — you can this instant give your wealth to the poor and follow Jesus as he suggested in Matthew19,21. Or you can this moment go to kill somebody. The idea is: Don't ask what has been or what is usual, or what is expected of you, don't be determined by your past or by your habits or by the people surrounding you, but realize that this very moment is a new platform to jump into any possible direction like a flea.

A termination is a stop, a de-termination is a start. But you cannot expect such a de-termination from a child, since a determination includes a decision and not only a childish mood. You could even take this as a means to "define" the point of transition from childhood to manhood or womanhood. A child is called "grown up" if it is able to decide "where to jump or to go" and to accept "responsibility" for this decision.

Most people decide "for the usual" and "the expected", since there is little risk in doing so. But existentialists stress the fact that this decision may be "usual" and "to be expected", but not in any case compelling. They try to remind people that life is always a personal venture and that to decide to behave "as expected" remains a free decision that can be revoked any time.

But then existentialism has been mainly a short-lived philosophical fad after the two World-Wars. Why? Because in principle you can jump in any direction like a flea, but in practice nearly everybody does and plans what is usual and approved by time and reason. It was the "media vita in morte sumus" (In the middle of life we are surrounded by death) experience of the wars that generated this surrealistic "you can do what you want — it doesn't matter" feeling. That explanation fits to the fact that indeed surrealism and existentialism generally come and go together. It is this "we are thrown into this absurd world" rather than a "we are placed into this world of God" that describes the true existentialist feeling. But people can't live in such a state of mind for long — not even philosophers. One student of Heidegger's got sarcastic characterizing a Heidegger lecture of the twenties by the words: "We are wildly determined — if only we knew determined what to do!"

Hubertus Fremerey

back

James asked:

I've read both that "good" is the final end and the end in itself but I've also read the same for "happiness" as well. So my question is, which of these two is the deeper end?

Well, I may be going off the deep end here, but many philosophers including Plato, Aristotle, and John Stuart Mill have identified good and happiness as being the same things or at least they have said that the only finally good thing is happiness. So your question assumes good and happiness are different and they may not be.

But some philosophers, for instance Immanuel Kant, have held that although happiness may be a good thing, it is certainly not the unconditionally good thing (which Kant held was "the good will" or the motive of duty). So Kant believed that the good was a "deeper end" than happiness.

Ken Stern

back

Rakamatootoo asked:

Do you believe god is a idea engrained in all of our minds (in some way)? Or is god as John Lennon sang, "A concept by which we measure our pain"? Or do you believe neither? (This really isn't a problem for me, I just want your opinion.)

Why do you think the reality around you is "real" — or don't you think so? Do you think the reality around you is only a dream — like Tschuang-Tzu who woke up after dreaming to be a butterfly — and then asked himself if he was a butterfly dreaming to be a human?

Or if you think somebody loves you — or if you think you are loving somebody: how do you "know" this to be the fact? Maybe you err on both assumptions?

So what does it mean to be "an idea engrained in all of our minds"? Some people simply have the "experience of God" — but they cannot "prove" God to be real like you cannot "prove" your feelings to be "real" — or your surroundings at that. Is "reality" "an idea engrained in all of our minds" — or is love?

Some people have tried to disprove God by experiment. Galton asked a reasonable question: If God is responding to prayers of true believers, there should be significantly more miraculous recoveries from bad illness among those true believers and their relatives. But the statistical data available on the illness and death of ministers and their families didn't show any significant difference from other statistical samples. But then the true believers would say "The ways of the Lord are strange and unsearchable by us dumb humans!" (compare Romans 11:33). Popper calls this the "ultra stability" of an opinion: The argument is formulated in such a way that no counterargument will destroy it. Once a sect predicted the doomsday for a definite day and hour — and nothing happened. One could have thought that the faith of the true believers would be shattered to pieces by this outcome. But they said: "Since we were praying all the day, God spared mankind for another time." So maybe the idea of God is simply the great stabilizer of souls, the one great hope left to make lifes burden bareable like somebody clings to the hope that his true love is really returning his own feelings. Nobody can disprove that. The philosophers Feuerbach and Marx therefore called the belief in God "the opium of the people". But if this opium enabled people to build the great orders and monasteries and cathedrals and to explore nature like Kepler and Newton did then it's a really strong stuff and much better than the sort Hitler or Stalin offered — or maybe even "the American Creed", "the pursuit of happiness". We all are caught in some dream or living under a delusion. May it be a good one!

And then compare what I answered to Jeremy. (Jeremy asked: What is faith, in the context of our life-view? If our actions are based on what we believe (about God, our nature, the afterlife, etc.), then what is the basis of our belief? ...)

Hubertus Fremerey

back

Mark asked:

If I have been depressed for years and all treatment has resulted only in my being able to perform the most basic functions, i.e. I am unable to have relationships of any consequence; steady employment is out of the question; I have, over the years, tried the following: university education, traditional "therapy", three years at the Carl Jung Institute, rebirthing, Swami Muktananda, ten years at a cloistered Benedictine monastery, ordination to the Roman Catholic priesthood, EST, every anti-depressant known to medical science: the list is really too long for such a forum as this — if all of these things have not worked, nothing is working, I'm just at the end of my rope, don't I have the right, even the responsibility or obligation perhaps, to use suicide as a merciful option? I just think this is a perfectly rational solution. Help me.

Well as you no doubt realize, this is a philosophy forum and not a clinical psychology forum. But I'll say something about this problem anyway. Chronic depression is an extremely bad and sometimes insoluble problem. I see your attempts at solutions as basically going into two categories: religious, and pharmacological. There are other categories of possible solutions.

Category one: physical
Drugs: usually work reasonably well; if they don't for you, then that's not a great sign, but there are always new ones coming out.
Shock therapy, i.e., electroconvulsive therapy. Well, usually I highly disapprove of this, because I think it causes some brain damage. There are recent varieties which are not supposed to; I find that hard to believe, but that's what is claimed. As an absolute last resort, before suicide, I'd try it.
Exercise: good for depression. Take up some intense form of exercise: running, weightlifting (preferably both).

Category two: emotional
Religion: you tried it and it didn't work. However, you might give Buddhism a try... find an old and reputable monastery, NOT some guru.
Traditional therapy: no way this works for chronic depression. Same for EST.
Cognitive and/or philosophical therapy: doesn't sound like a good bet for you, but you might try it.
Group therapy: I'd say this was a reasonably good candidate for you, to get you into something new, and see that there are others in your situation, maybe even to find a relationship. If you can, possibly, get into a good relationship; that goes a long way to help depression.

Category three: situational
Drastically change your location and living situation: i.e., move to another country and get a job. Given the above, it looks like you've tried the equivalent and it didn't work... but I'd be depressed by British winters, myself.
Suicide: always an option, in my very strong opinion. Everyone should have the right to end their life, if they wish. However, this is something I disapprove of, not so much on ethical grounds as on pragmatic ones. That is, the world is vast, possibilities are enormous, and time keeps moving. Things change. Unless you are in truly enormous pain and there is really no other way out (i.e., you're a terminal cancer patient, you're being tortured, etc...), I believe it is better to wait things out.

So where are we? Group therapy; exercise; cognitive therapy; Buddhism; shock therapy; and maybe, hopefully, there's a new drug out that might help. In addition, there is a reasonably good possibility that as you age, you will improve. Wait for that. The fact that you have been able to attempt these solutions, and that you are writing to this forum, indicates that you can take action, and want to change. Go with that.

Here is a book just out which might also help:

Depression is a Choice: Winning the Battle Without Drugs by A. B. Curtiss Hyperion, New York. 2001

Steven Ravett Brown


Mark has my deepest sympathy, but I'm afraid that as a philosopher I am not competent to help him. I wonder, however, if he would consider a relevant philosophical question. He is attempting to find a means to an end, which is relief from depression. He emphatically does not want to try one more thing that fails. My question is: How does Mark know that suicide is not one more such inadequate means to his chosen end? My hope is that this question will induce in him enough agnosticism about "what comes next" to stave off his fateful decision, at least one more day. For after the first suicide, another is not an option, as might be another religious order or another antidepressant. Suicide is the unrepeatable option. (Should one ever find oneself saying, "Well, suicide didn't work. What shall I try now?," one is already in a state beyond any help that I know of.) So to the degree that rationality still commands the allegiance of one who is deeply depressed, to that degree it is rational for him to ensure that he is not about to go from the frying pan into the fire.

Tony Flood


You did all those things! And you haven't given up hope? This is very inspiring. If only more people were driven to find happiness with such perseverance. I suggest you read the philosopher Seneca for wisdom, and Plato's Early Socratic Dialogues for amusement.

Rachel Browne

back

Krystine asked:

What exactly is a philosopher and is it hard to be one?

The most important virtue of a philosopher is to be serious. And to be serious can be very hard, as at least Socrates and Bruno and Galileo and Spinoza and Kierkegaard and Nietzsche and Bonhoeffer had to learn — but many others too. When the Elector of the Palatine invited Spinoza to a professorship at Heidelberg University, Spinoza declined, saying that even the authority of the Elector would not protect him from the furor of Lutheran theologians there.

A true philosopher is someone who in ernest wants to know the truth — and then he has everybody against him who thinks to have the truth found already. For the true believer the search for truth is a provocation since it presupposes that truth has not been found. He cannot accept that. And in this sense most "normal" people are true believers of the "common sense truth" too — which may be simply common nonsense. To ask for the truth is to be a freak like Socrates was. Why ask for the truth if mom and pop and the teacher and the pastor know it? That's insulting and a sort of arrogance. Thus Luther has been chided heavily "How do you little monk dare to oppose the most eminent theologians!" But Luther answered like Socrates: "Since it is not good acting against ones conscience, therefore I will not revoke. God help me, Amen!"

A philosopher is like a judge, hearing all arguments while seeking the truth, the whole truth and only the truth — sometimes drawing heat or hate for that from all sides.

In his "glasshouse note" of 25th September 1999 Geoffrey Klempner cites Wittgenstein saying: "Philosophy should be hard. The philosopher should not seek comfort or stimulus in this or that" (Glass House Philosopher page 17).

Hubertus Fremerey

back

Marie-Michelle asked:

Is it possible to ever have a Utopian Society?
Why are we all searching for a Utopian Society?

Strictly speaking you can never find or build a Utopian Society: Utopia literally means 'there is no such place', so to find one or build one would be a contradiction in terms. But that is probably not what you mean. If you mean is it possible to realize concepts for a better society the answer would have to be yes and no, depending on your viewpoint. People have tried to put into practice ideas like for example communism, but the results differed from those expected by the 'believers' in that some good things failed to materialize and negative aspects failed to be eradicated or entirely new problems surfaced etc. I guess whether you are hoping that change will be for the better largely depends on the starting conditions i.e. whether you hope to gain more than you stand to loose. It also depends on personality — it seems that there usually will be some good the conservative character (rightly) wants to retain and some good the progressive character (just as rightly) wants to achieve. Who is right? The only certainty is that as long as there is life there will be change ....

Helene Dumitriu

back

Malini asked:

I am not a philosophy student but am very fascinated with the chaos theory. It is said that a chaotic system is not a random system. Also, it explains that a small change in the input could produce a huge change in the output, hence it being called the 'butterfly effect'. Correct me if I am wrong. My question is: Every event or action, however small, can produce a specific outcome. Although we're not able to predict it precisely, there is still a cause and effect situation happening. If that's the case, does that mean that there are no such things as coincidences?

Chaotic system not random: that is correct. Butterfly effect: that's correct.

However, when you say "specific outcome", you are not being clear. Specific in what sense? Definite? Definitely known? Observable? Predictable? Strictly speaking, it is not accurate, on the quantum level, at any rate, to say that every action will certainly produce a reaction, because of quantum uncertainty... zero-point energy, if nothing else. But you are going further than that when you say that an action produces a "specific" outcome, if by that you mean one unique to an action. Why should that be true? So when you talk about "cause and effect" situations, you have to be careful. That being said, yes, chaos theory is a deterministic theory, within the purview of Newtonian mechanics (although that doesn't have to be the case), and so is a causal one. But, so what? Newtonian mechanics isn't the last word, right? In other words, chaos theory, a branch of non-linear dynamics, is a subset of mechanics which is concerned with very complex systems, i.e., recursive systems. You can find non-linear, recursive, systems anywhere, and you don't have to be Newtonian to do that, although that's pretty much where it's applied now, as far as I know.

So what does all this have to do with coincidence? Well, I don't have the slightest idea. Perhaps when you can come up with a definition of "coincidence" which is a mathematical treatment applicable to physics you might be able to answer this question. But the normal sense of the term "coincidence" just means something vaguely like a surprising repetition or similarity. Chaos theory has to do with very hard-core mathematical models, not with feelings of surprise. So your question is mixing categories of ideas or concepts, like apples and oranges. Chaos theory is simply not relevant to coincidence, as that latter term is normally employed.

Steven Ravett Brown


That's a brain-twister. Toss a dime or a dice. The outcome of tossing a fair dime is with necessity equal probability of heads and tails. But you cannot tell what the outcome will be in a single toss. So what is it — is it necessity or is it coincidence? That's explains the "butterfly effect": Two or more very large developments are of "nearly exact" equal probability as is a knife standing on it's point: There must be any one outcome, but neither before nor afterwards can anybody prove what outcome there will be, since by sheer logic two or more or an infinity of outcomes are likewise probable, neither having any provable priority. What one has to accept then, is the necessity "by definition" of equal chances of "head or tail", which has nothing to do with the "uncertainty principle" or any physical effect but with "the logic of symmetry". By definition "symmetry of form" includes "symmetry of behaviour and outcomes". You cannot evade that conclusion. So the paradox is: Statistical outcomes are by necessity unpredictable.

Hubertus Fremerey

back

Erin asked:

I am trying to find information regarding the philosophy of in the area of body art (tattoos and piercings of today). I have been unsuccessful in finding any information and wanted to know if you were aware of any references.

You might seek out the following

The Body Aesthetic from Fine Art to Body Modification edited by Tobin Siebers

Pictures on the Human Skin by Gambier Bolton

The Decorated Body by Robert Brain

Rachel Browne

back

Mynam asked:

When I study the history of philosophy and try to understand the world I'm thinking and thus living in now, I seem always (no matter which philosopher or which time period I study) to come to the conclusion that we humans are asking the same questions over and over; just in new forms and in new aspects of reality (or non-reality). I do not mean that great thinkers have all been thinking about the same issues; rather that they (and we) have been in the same path just in different points in space-time. In the beginning I felt that we all are just improving and trying to ennoble the same questions that the the ancient Greeks and even human thinkers long before them asked about their surrounding world.

I'm not open minded enough to imagine what the questions complementary to today's (and yesterday's) questions would be, or if those questions even could be defined as "questions" but I do know that this reality we have created through our questions, i.e. our perception, cannot be the whole truth (if there is a "whole" or a "truth").

Why are we not creating instead of recreating?

That's a great question you posed! Perhaps you start looking at the history of art. Maybe that sheds some light on the history of philosophy as you see it.

The Greeks seem to have had a similar concept of art as we had at least up to the middle of the 19th century. But this is not exactly true, since there has been the art of the Middle Ages in between which is totally different from Greek and Roman "classical" art. But from about 1350 to about 1850 the classical ideal prevailed again. And this fading and reappearance of a special way to look at the world had it's parallel in the history of philosophy too: The selection of philosophical problems and the style of arguing has been very different in the Middle Ages from that in "classical" Antiquity and in Renaissance, and Modern philosophical thought.

And then there has been another great revolution likewise in the arts and in philosophical arguments starting from about the 1850s together with the rise of Industrial Society. You can mark two symbolic dates: The first "World Exposition" 1851 in London and the appearance of the "Communist Manifesto" in 1848 opening a year of revolutions swamping Europe. In those years, Turner and Corot and some other painters began to change the traditional way of painting, and this led ultimately to the wave of "impressionism" dominating the art-scene of Paris after the German-French War of 1870/71.

The main trait of Impressionism is a new "subjectivism" replacing the old "objectivism" of academic art. The academic art of the time was still mainly concerned with mythical and historical themes and with landscapes and townscapes and seascapes in a naturalistic fashion. The paintings look like remakes of the Netherlandic art of the 17th century or as remakes of renaissance art or even gothic. Not so the Impressionists: They ignored history and the past as subjects and the traditional way of painting altogether and tried to have a fresh look at the world around them. They painted coffee-shops and backyards with dancing people and "genre" and in this they still resembled the Netherlandish paintings of Hals and Vermeer in a modernized way. It was "painting of the people for the people" — not for the cultivated upper classes with their knowledge of history and mythology. But that still only paved the way for the great revolution that was to come with the work of Cezanne, Van Gogh, Gauguin and the Fauves and Cubists.

The revolution consisted in not only giving up the classical "great themes" of history and mythology but the values of "realism" and "naturalism" altogether. A picture is not a copy from nature, is not illustrating anything recent or past, be it important or unimportant, but a picture is a "work", consisting of colours and forms and of nothing else. The world may be "cited" in such an art-work, but it is not "represented" anymore. It simply is not the aim or the task of the artist, to illustrate or to represent anything but his own ideas. The whole concept of "representative" art dominating European art from Antiquity has been abandoned together with the traditional ideal of "beauty". The idea of beauty has changed from Rafael to Luigi Fontana, from naked goddesses arranged in a mythical scene to naked areas of colour arranged in a rectangle.

Now you ask what this has to do with your question concerning a new way of seeing the world with philosophical eyes. The answer is, that exactly what has changed in art has changed in the same years in philosophy. Cezanne said "Let us paint as if nobody ever has painted before!" In the same mood Husserl said "let us think as if nobody ever has thought before!". Cezanne said "Forget all themes, don't copy, don't illustrate, but construct a new world of artificial reality of it's own right!" Husserl and Wittgenstein said "Forget all classical problems and arguments, go back to what people experience and see how they use words to express those experiences!"

The years from about 1890 to about 1970 of European cultural history have been one of the greatest cultural epochs in the history of mankind, comparable only to the Golden Era of Pericles or to that of Florentian Renaissance. In this epoch the whole way we are looking at the world has changed fundamentally, not only in the arts, but in literature and music and in mathematics and physics and in the social sciences too — and in philosophy. And that explains in part the trouble some philosophy-students have today with a strange modern "analytic" way of asking questions that seem to ignore all traditional questions and answers concerning God and morals and mankind and what is beauty and what is truth.

Your question is: "Why are we not creating instead of recreating?" As you may have got from the arguments above, we are trying hard to "create". But then people are conservative and resisting change. This must not be bad. The churches have resisted freedom of speech and thought with the argument, that it would lead people astray into the wilderness of strange ideas and far from the truth. The communists said likewise and the Nazis too. There are not too many people seeing in a liberal and multicultural society much more than a social and moral chaos. You have to respect that for a moment and think about it. Ask yourself how much real freedom you can accept and endure. One alway needs a minimum of rule and regularity in life, of reliability, trustability and predictability.

If you change the world by "creating" a new one, this new world too will have it's problems of reliability, trustability and predictability. You have to convince people that your new world is so much more lovable and livable that they dare to give up what they are used to. But then people have been cheated many times by false prophets like Hitler and Stalin and some more of that bunch. Why do you think people are shunning genetic engineering and PID (pre-implantation-diagnostics)? It's not that simple to create a new mankind! What will become of the family? How will we earn a living and pay our rent or get our pensions and social security? All those questions are natural and have to be answered if you want to create a new world.

Jesus has announced a new heaven and a new earth — but what do you see in history? You see the churches and much superstition and bigotry and hypocrisy and moral repression. That was what Voltaire and Nietzsche and Marcuse fought against.

A new society and a new philosophy is like a new house. You can try to build that, but you must be sure that people will like to live there and that it does not collapse.

The philosophical house we are living in today has in part been built by Kant and Hegel on older fundaments laid by Plato and Aristotle and Augustine, but has been modified by Marx and Nietzsche and Freud, by Husserl, Heidegger and Wittgenstein and some more great philosophers of recent time. If you want to build a new philosophical house for mankind, you are not free to build anything fantastical. You always have to see the people living there. Try to see yourself and all those dear to you, and then some more until everybody has a room to live in — and not a cell like a monk or a prisoner or a bee. To devise such a house is no small task but requires the greatest of architects.

What is your time-scale? The pyramids of Gizah were built at about 2600 BC, and the Parthenon on the Acropolis of Athens was built during the lifetime of Socrates some 2400 years ago. The gothic cathedrals are some 800 years old and the Eiffel Tower is from 1889. Is that pace of human history fast or is it slow? I think it is explosive! Think back to the year 1902: Who would have thought then of how we are going today? So you can't predict what will be in 2102, one hundred years from now. A new sort of cyborgs having replaced mankind? And you asked for "creating instead of recreating" and you think "that they — the great thinkers — (and we) have been in the same path just in different points in space-time!" Where should they have been? Maybe mankind will swamp the universe during the next 1000 years. There must be some traditions of thought and feeling holding mankind together, some language of mutual understanding and caring and respect. Don't be too impatient then. Things may get out of control otherwise. Be nice to humankind.

Hubertus Fremerey

back

Sean asked:

What are your philosophical views on the existence of extraterrestrial life? I see it as being both physically possible and highly probable, which makes the odds high that life does exist "out there". I've done a lot of thinking about the origins of life and the theories I've come up with allow, and could possibly prove, the probability of alien life. The thing I can't yet understand is whether or not consciousness would exist among these life forms, mainly because I don't even fully understand consciousness in myself. Maybe I could get some other points of view on consciousness and extraterrestrial life, I mean your own views. This isn't sci-fi, this is the philosophy of existence. Peace.

Sean also asked:

Do I exist? If so, how do you know? Do you exist?

From a philosophical point of view one would have to say at least that the logical possibility of extraterrestrial life cannot be excluded. Scientific thinking today is that it is highly probable that life has existed or exists or will exist other than on planet Earth. Also it seems that consciousness and self-consciousness naturally arise with a certain level of organisation and interaction, so it would seem likely that given favourable conditions and enough time, this would evolve on other planets as well ... If you have a theory that, as you say, could prove the probability of alien life — why not let us know (in short) your argument?

For the question of existence Descartes famously concluded from the fact 'I am thinking' that to think one must exist. In other words — from any state or action the existence of the subject follows. Thus we can conclude from your E-mail to 'ask a philosopher' that you exist (but not who or what or how many you are), and you may conclude that the 'ask a philosopher' list and web site exists ... (i.e. there is at least one X such that X has sent the E-mail signed 'Sean' and at least one Y such that Y has received it and posted it on the website, note that based on this evidence it cannot be excluded that X and Y are the same)

Helene Dumitriu

back

Kathy asked:

Can you please give me a basic definition for rationalists and empiricists?

A "basic" definition is no problem — but not helpful either. In our context, the terms "rationalist" and "empiricist" are used in relation to philosophers only and not in relation to an everyday approach to life. The typical "rationalists" in the history of philosophy are Descartes, Spinoza, Leibniz. Their style of philosophizing is what has later been dubbed "armchair-philosophy": You sit in your chair and think on "first principles" and "necessary conclusions" holding the world together. Then comes the empiricist down the lane and asks, if you ever checked your splendid ideas against the brute facts. How could Newton's "law of gravitation" of 1678 ever have made such an enormous impact on the whole of European thinking, if he had not shown that from his simple formula of gravitational forces sprang the elliptical form of the orbit of planets that Kepler had derived in his famous laws some 70 years previously. And who forced Kepler to formulate those orbits? That was Tycho Brahe with his extremely exact observations of those orbits — which was of course empirical. Since Antiquity the idea has been that the orbits are circles, but Brahe's data excluded this possibility for the orbit of Mars definitely in the same way as Michelson some 300 years later excluded with his extremely exact measurements the possibility of the 'ether'.

Popper's concept of "falsification" sprang from his insight that Einstein's theory of General Relativity could be disproved by the measurements of Eddington in the 1920s, but the theories of the Marxists or the Freudians or the Christians could not. Many people do not even understand the concept of "falsification". It does not mean to show that a proposition or a theory is false, but that there could be devised an experiment (empirical!) that by its outcome conclusively "approves" or disproves the proposition or the theory. Even the word "approves" is correct here, since no experiment can "prove" a theory. Einstein's theories have up to now resisted all attempts to disprove or "falsify" them, but that does not prove them to be right.

In the late 50s Heisenberg devised a "world-formula" for particle-physics from theoretical considerations ("rationalist"), but then came the "quantum-chromo-dynamic" with its "quarks", and to get there one had to build the large colliders costing billions of dollars ("empiricist"). But of course one needs convincing theoretical concepts to make the investment of billions of dollars for the colliders plausible. Even Columbus needed his (false) convictions on the distance to "India" in the west to find support for his endeavour to get there. And the steam-engine of James Watt has not been the outcome of "practical engineering" either: He had to improve a demonstration model of some older type of steam-engine for the physics department of the University of Glasgow — then one of the best universities of the world. There he chatted during the coffee-breaks with some of the most famous physicists and chemists of the time and made use of their hints and suggestions on latent heat etc. for his design of the new steam engine.

Locke and Hume are known as "empiricists" ("nothing is in the mind, what not has been in the senses"), but Kant — who was well acquainted with the work of Descartes and Leibniz — got to his decisive new insights by combining "rationalism" with "empiricism". It was Hume who got him up from his "dogmatic slumber" by the notion, that "consequence" is not and cannot be an "empirical" concept but must be a "postulate" of our thinking that transforms a mere empirical "sequence" into a theoretical "con-sequence". As the examples above with Einstein, Heisenberg, Kepler and Newton have shown, one always has to combine a theory and empirical data to get at useful results.

The rationalist approach had much to do with the remnants of christian Neo-Platonism in Occidental thought. If human intelligence and the laws of nature are both expressions of the creative wisdom of God, then by this common origin the workings of nature and human insight should fit by some sort of inspiration. But Kant didn't need such an assumption. For his christian contemporaries that was shocking, since then the old assumption of a unity of mankind and nature under god was not assured anymore. Nature became a mere object for human technical and economical interests, it changed from a world to live in to a mere resource to be explored and exploited. Since then the principle of "R & D" — research and development — prevails: Theory, experiment and application combined to the modern "instrument of progress" and the antagonism between "rationalists" and "empiricists" of the time from about 1630 to 1780 has become historical.

Rationalism was no sort of stupidity though! The great rationalists have been great mathematicians and logicians. They hoped to get at the core of the machinery of the universe by mere logical and mathematical thinking since they held God himself to be the great "watchmaker" and mathematician of the world. This was a great idea in the neo-platonic tradition. And then they were convinced in the same line of argument of a deep reaching unity of truth, beauty and goodness respectively of reason, sensitiveness and virtue. This unity has been the guiding idea of Spinoza (1632-77) and of Lord Shaftesbury (1671-1713) and their time. This explains why one expected from rationalism much more for the progress of mankind than we can imagine today. Even the slight contempt, with which continental philosophers look on the anglo-saxon "empirical", "pragmatist" and "analytical" tradition of thought springs from this "neo-platonic" and "rationalistic" concept of "the great order of things" (or The Great Chain of Being as Arthur O. Lovejoy has called it in his famous book of 1933, which is still available).

One last remark on the sort of "experience" the English "empiricists" are talking of. This experience is restricted to "sensual data" from which "concepts" and "theories" are derived. It's a strict methodological sort of problem — to explain how "concept-formation from experience" takes place in the human mind. Therefore the sneering remark of a French writer, those tea-sipping English philosophers could not have the slightest idea of "real" experiences with deep love and deep faith and deep mourning, with doubt, despair and commitment, totally misses the point. Philosophy is about arguments, not about feelings.

Hubertus Fremerey

back

Stone asked:

To what extent is the human perception of dominion over the world similar to that of an animal's natural belief that they are the dominant species, so as to ensure the survival of that species?

Hmm. Is there (as your question seems to suggest) really a 'human perception of dominion over the world'? I would suggest that while this belief may still be held by individuals (and as part of some religions) it is no longer the dominant view (at least not in the Western hemisphere). Also what is meant by dominion over the world? That we are the dominant species? Surely not, there are better candidates e.g. in insects and bacteria, who vastly outnumber human beings. That we have a God-given right to exploit other beings? I have yet to see this properly argued in a philosophical way ... Regarding beliefs it seems to me that most philosophers would claim that animals do not have beliefs. But even if we point to animals forming habits and call those a kind of (very simple) belief (e.g. regarding where an animal expects to find food based on recent experience), we do not have any grounds that animals can think about whether they are a dominant species or worry about survival of the species. They strive for survival, procreation etc. through instinct, as do we by the way.

You might have a point that against the backdrop of what we would do anyway because our instinct drives us, human beings are additionally able to 'rationalise' their actions ascribing them to other motives i.e. for the greater good, because it is God's will etc. etc. However a philosophical approach would be to recognize our instinctive tendencies as well as the need to 'look good' and to think for oneself on which basis rational behaviour of oneself/ human beings should be based, and how to argue for that basis in such a way that it is convincing and acceptable for all rational persons.

Helene Dumitriu


That's an interesting idea — why should mankind not be as egoistic as any sort of plant or animal is by it's very nature? The answer is: Man is as egoistic, but then he is so terribly strong with his smart technology that he is endangering the very precondition of his existence (at least up to now) — nature itself.

Maybe mankind won't need nature any more in another 100 years. Maybe the earth will then look like the moon or the mars today, but with men dwelling in big towers with every comfort of living, the food being generated in biochemical processes and plants and animals being hatched and brought up in glasshouses.

The problem up to most recent times was to protect mankind from nature. Now the problem seems to be how to protect nature from mankind.

Nature is a very big system of open feedback-loops of the wolves-sheep type: If there are too many wolves, then the number of sheep goes back and more wolves starve. Then the number of sheep goes up again and the number of wolves likewise. In principle such could be — and sometimes has been locally or regionally — the fate of men too: If there are too many people in some area and some flood or drought or illness destroys part of the plants and animals men are living on, then people starve like the hungry wolves. So in principle there can be the case that the whole system of nature crashes before mankind got to be independent of it and then the whole of mankind has to starve. But nobody knows for sure what it takes to crash nature. Will the glasshouse-effect do? Will overpopulation do? We don't know.

Hubertus Fremerey

back

Richard asked:

Q1. Does destiny control or decide one`s future?
If yes does it imply that man has no free will to decide his future?

Q2. Fate and a curse do they coexist to destine man?

Unless proof is provided whether our lifes/ futures are controlled by destiny, to believe in destiny is just a belief, not philosophy. Free will according to some is not incompatible with destiny whereas others see it as a contradiction. If you believe in fate then again it would depend on your belief whether that is compatible with the concept of blessing and cursing someone having effect on that person's life. Some would say that it is incompatible with the concept of an all-good all-powerful God to allow curses to work.

Helene Dumitriu

back

Joao asked:

What is the explanation of moral behaviour, and our ideas of good and evil?

Seen from an objective point of view our ideas of good and evil are simply regulatory forces. How do you imagine any society where cheating, stealing, destroying and rape are the common behaviour of the people? Such a society could not last. Therefore even little children are advised not to lie, not to steal, not to destroy. That's simple.

Animals cannot be "rational" in this way, but they don't live in anarchy either. They have by nature restrictions to their behavior that are studied by behaviorists and ethologists. Look up any good book on "sociobiology" or on "ethology". But since man has (nearly) no inborn instincts and restrictions any more, culture has to replace nature in this case.

And in the same way a human language is infinitely much more flexible and expressive than any "language" of signs and gestures among animals, so is cultural regulation of human behaviour infinitely more flexible and expressive than any naturally enforced behaviour could be. This includes the fact that culture often leads to "un-natural" behaviour. But this "un-natural" behaviour is nearly the opposite of what some religious fanatics think it to be: Polygamy and homosexuality and suppression of the weak by the strong are very common with apes and other animals. Even murder of relatives has been observed in populations of free living chimpanzees in their natural African habitat (see the books of Jane van Lawick-Goodall). So that all is not "un-natural". But no animal has ever been observed in "mortifying the flesh" or fasting or committing suicide. There are sometimes dogs and cats starving with grief if somebody dear to them is dead or missing. But that is not comparable to "seppuku" ("hara-kiri") or to shooting oneself.

And in the same way as the ability to speech is an outcome of the physical and cognitive abilities of man by his genetic equipment, so is the ability to be moral and to understand and to follow moral rules. This is important! In part the ability to understand and to follow moral rules is a special aspect of "intelligence" (look up Lawrence Kohlberg: Kohlberg Tutorial). But there is a phase mainly in the first year of an infant, when the behavioral foundations on which the ability to understand and to follow moral rules will grow later in life are prepared. When these foundations cannot be set up in the proper way, a growing human will never be able to behave according to moral standards. And that's a great tragedy.

Part of your question was: "What is the explanation of ... our ideas of good and evil? I have been describing what their use and effect is. To explain where they come from in their cultural context is far bigger task! Where do the christian ideas of "charity" and of "brother/ sisterhood" and of humility with respect to god come from? That transcends all notions of "cheating, stealing, destroying and rape" or even the "Ten Commandments". The same is true for the Jewish, the Islamic, the Buddhist, the Hindu, the Confucian, the socialist, the liberal and other ethics. They all are defining and defending values in the context of a special world-view. So "the explanation of .. our ideas of good and evil" has to explain the origin and development of those world-views supporting special values. To do that by sociological arguments was one of the great tasks that Max Weber defined for his profession. From a more historical point of view Arnold Toynbee tried to do the same and from an economical one Marx and Schumpeter did likewise. They all and the scholars of "cultural anthropology" followed in this the pioneering ideas of Vico, Montesquieu, Herder and Hegel.

So I gave you some names and headwords you should look up in your dictionary or online using "google.com", "m.galaxy.com" and other search engines and online dictionaries.

Hubertus Fremerey

back

Amber asked:

Is "What it is like to be you" an essential part of you (i.e. your mind)?

Is it possible for a scientist or psychologist to understand who you are? The mind and body problem is difficult to understand. I read Thomas Nagel's 'What is it like to be a Bat?' but I still do not know why, 'what it is like to be me' is an essential part of me.

You can still be a person without much sense of what it is like to be you. Apparently, this is what it is like for schizophrenics, who are quite obviously people with a mind, but it has been found by psychologists that people with such an illness have difficult with recent autobiography and also with the understanding of the mind of others. An understanding of the mind of others, which is really constitutive of an understanding of the mind, is obviously going to disturb any understanding or feeling of what is like to be a person, and, given the autobiographical difficulty, what it is like to be me. Neuroscientists are definitely interested in and researching this problem of continuity and coherence in psychological personal identity

I think that what it is like to be me has to fall with an autobiographical and physical field. Could an amnesiac have an idea of me if there is no continuity, nothing to identify from moment to moment? I don't know. The problem is that philosophers argue in accordance with logical principles and would say that you could have a subjective experience of "me" in one moment without autobiographical or physical experiences being relevant. This is what the "what is like to be me" problem is about: subjectivity: That which is private and cannot be shared. But this pure subjectivity wouldn't determine what it is like to me, because it is purely positional

The question "what is it like to be a bat" is a type problem. We cannot know what it is like to be another species, especially a bat which has very different sense experience. But the question is also intra-species in the sense just described.

Rachel Browne

back

James asked:

Is the following argument true by definition, false by definition, a comment on my direct experience, or ultimately unknowable?

1. All statements are one of the following:

— true by definition
— false by definition
— a comment on my direct experience
— ultimately unknowable.

2. "A cat is a cat" is true by definition.

3. "A cat is not a cat" is false by definition.

4. "My desk looks brown to me" is a comment on my direct experience.

5. "The planet Jupiter is larger than my hand" is ultimately unknowable, as no matter how strong the evidence is that I use to decide whether the statement is true, there always remains the logical possibility that I will later find even stronger evidence to point to the contrary.

I would say that 'what follows' (that is, the propositions 1 to 5) is not an argument at all, but merely a set of 5 assertions. The first make an assertion that all statements fall into one of the four listed classes, while 2-5 give examples of the classes (though it must be said that within statement 5, there is an argument that the purported example is of the type asserted). My guess is that you are being asked to construct an argument of your own, which will try to establish that the statement 1 is itself of one of the types it proposes.

If it fitted into the second type, it would be useless, so you can rule that out. So you are left with three possibilities: that the compound statement (1) is true in virtue of the meanings of the words that make it up, is based on your experience of statements (and hence might or might not be true), or cannot be known to be true (which makes it merely an educated guess, and could be — you might argue — therefore equivalent to the previous type).

Alternatively, you could try to find a statement that does not fit any of the four types. That would show that compound statement (1) is false.

Tim Sprod

back

Anand asked:

I am doing my research over the positive aspect of sex: a hidden sin.

So I just want to know about sexual power over mind. Why this sex is so powerful that when a man knows that unsafe sex can led to death but still can't avoid it, because when sex comes into mind then any other thinking activity stops. Why is it so?

Wow, what a question. Offhand, everything you say above looks incorrect. First, sex is not a sin. Second, "sexual power" is one of those phrases that has been beaten to death. Just what, exactly, does it mean? Third, yes, unsafe sex can lead to death; so can driving a car, crossing the street, going swimming, and any number of other activities. So should we stop all those, also? Sex being "unsafe" is not a black-and-white issue, just as most other human activities are not. Fourth, just whom are you talking about when you say that when "sex comes to mind, all other thinking activity stops"? Not me, I hope, because I'd feel both insulted and amused at your naivete. Look at it this way: when one is a young child, and one smiles, what does that smile mean, and how much control does, say, a 4-5 year old have over it? It means pleasure or happiness, and there is very little control. As one ages, smiles become, if one wishes, controlled and imbued with many possible meanings. This is the result of education and experience. Well, it's the same with sex (and anything else). The more education and experience, the more control and the more subtlety in feeling and expression. To have the experience you describe above with sex, as an adult, indicates an appalling lack of education and experience with it, in my opinion.

I don't even know where to start to correct your misconceptions. Try the library; libraries have enormous resources of facts. Try a course in sexuality at a public university. The misconceptions above are some of the primary reasons for problems with sexuality in human cultures, in my opinion. If you doubt what I'm saying, just look at the statistics: the few cultures with good sex education (the Scandinavian countries, mostly), have the lowest rates of disease (STDs) and unwanted pregnancies in the world. The more sex education, the less unwanted pregnancies, disease, and, for that matter, emotional problems with sexuality. Look it up; all sorts of studies have been done on this.

Steven Ravett Brown


If you are doing research into this you have to read Freud. A Freudian account of the power of the sexual would be that the sexual instinct is an originary and primal instinct and as a child comes to develop a sense of self, an ego, and at the same time a relation to reality, the primal instinct towards sexuality is split off from the ego and it's relation to reality. So when a person wants to have unsafe sex, they are really driven by their originary sexual instinct, and not thinking of reality and the consequences. This is still to to be driven by the mind, but by the unconscious rather than the conscious mind. The conscious mind is concerned with reality. The unconscious is rather driven by primal instincts. The coming into being of the rational thinking social person has a repressive function on primal instincts, but the sexual drive, for Freud, is extremely strong since it has its root in our very nature, in the way we were as children before the social force took hold. This difference between the unconscious primal instinct and the conscious relation of thought and reality, given the strength of the sexual instinct, explains why thinking stops when sex comes to mind.

The sexual function is cross-species. But in male, there is perhaps a desire to demonstrate potency and giving and in the female there is desire for submission and receiving (simplifying and generalising here) and safe sex interferes with this. Potency cannot ultimately be shown, and submission is not complete.

But given all this, I would say that, basically, the desire for what you describe as "unsafe sex" comes down to the desire for true intimacy without barrier. And in terms of thought rather than desire, sometimes people have it — in their conscious mind — that certain risks are worth taking for the sake of physical intimacy.

Rachel Browne

back

Libby asked:

What is a word?

The Oxford dictionary definition is as follows: 1. A sound/ sounds expressing a meaning and forming one of the basic elements of speech 2. this represented by letters or symbols.

What I really want to know is how are language and thought connected? or how philosophers have gone about looking into this matter? Can there be such a thing as 'private language' — are thoughts simply unspoken words strung together in the mind? Where do the limits/ boundaries of language lie? And do words themselves convey meaning i.e. can a proposition be broken down into its individual elements, like building blocks, or must a word be within the context of other words for it to have meaning — just as a thing cannot exist in itself, only within a state of affairs?

Finally, if X is left of Y, is the 'left of Y' arbitrary? Could one mean the same thing by saying 'right of Y'? Why does X have to be the subject, and is syntax as it is because it is or is their some logical relationship between it and reality?

First, I believe Wittgenstein decisively undermined the notion of a private language by his famous thought experiment in Philosophical Investigations. Imagine a person who, every day he experiences a certain sensation, enters a certain squiggle in his diary. But one day he experiences a sensation about which he isn't able to decide whether it is the one he marks by a squiggle or another, slightly similar sensation. How can he now check whether or not the sensation is the same as before? The only thing he can turn to is his own memory, but the whole question arose only because he became doubtful about his own memory.

Second, it seems unclear what it would mean to say that thoughts are words in the mind. If I think of the first movement of Haydn's twenty-second symphony, what are the words I'm talking to myself? They seem more like a sequence of musical notes to me. If I think of a purple parallelogram, what are the words I'm talking to myself? They seem more like a visual image of a purple parallelogram to me.

Third, it is surely not arbitrary that X is left of Y; what is arbitrary is whether or not we happen to be located at a place in three-dimensional space from where X is left of Y and not right of it.

T. P. Uschanov
Research Assistant
Department of Philosophy
University of Helsinki

back

Michael asked:

Re: entropy and probability in August 1999 edition (Answers 1)

It's standard physics I know that if — say you — pour out something it will tend to spread out, rather than roll back into the cup or spell your name. An explanation of entropy as being just an expression of the Law of Probability seems insufficient. It seems to me to be something more profound and important as the arrow of time and entropy run in the same direction. We are born, we grow old and die. Reversing my car backwards up the motorway does not refill the tank with petrol.

Suppose you have a closed container, a cube, say, with lots of little balls bouncing around in it, in all different directions. Now suppose you squeeze the container on one side. It gets smaller, the balls bounce around in a smaller space, right? Now suppose you suddenly release that side and the cube pops back to its original size and shape. Do the balls stay in the quarter of the cube they were in when you squeezed it? Why not? They spread out into the whole cube, right? Why? What is the chance, keeping the cube the same size, that they will all suddenly fly up into one corner, all at once? Pretty low, right? Why? What if you start with all the balls just lying on the bottom of the cube and you start bouncing the cube. Do the balls just bounce up and down, and stay like that, bouncing up and down? Why not? If one ball bounces a little to one side, soon all the balls will be bouncing in various directions. Why?

All we're dealing with, in the above, are laws of probability. In a closed system, you get, eventually, uniform distribution of energy and position. An open system is another thing entirely. Now, on the way to that uniform distribution, you can get all sorts of strange effects: chaotic systems, various periodic effects, and so forth. You might take a look at Prigogine on that. But ultimately you get uniformity. The organization that living creatures exhibit in their bodies is obtained at the price of increasing entropy in the form of radiated heat, mostly; and we are, locally, on earth (neglecting the rest of the galaxy, etc., makes it an open system), in an open system with the sun supplying us with energy to maintain our structures. But it's all running down, as far as we know at this point.

Now, why should reversing your car fill it with petrol, any more than walking backward would make you gain weight? Come on. We're dealing with events on a micro level here. It takes energy to reverse your car, so you supply that energy, you don't gain it. You need to do some reading in basic physics. The only systems in which you can do anything resembling beating the system here are on the lowest levels of structure, the quantum levels, and even there it's only temporary, or a stasis. Even protons have a decay rate; it's very long, but it exists. Depressing, isn't it. But that's no excuse; depressing or not, that's reality as we know it at the beginning of the 21st century.

Steven Ravett Brown

back

Kate asked:

Who was called the father of Greek science and in ancient Greek who founded scientific zoology?

I don't really recognise these terms, but I would have to say that in my opinion the ancient Greek who best corresponds to them would have to be Aristotle — certainly for the second, probably for the first.

Tim Sprod

back

Liz asked:

According to solipsism how would I determine if another person has a mind or not?

What other person?! The existence of anything other than solus ipse (the self alone) is doubtful to the putative solipsist. Also, in so far as mentality is central to personhood, then the determination that there is a mind is already built into the determination that there is another person.

Tony Flood


In no way. Solipsism is precisely the view that there are no minds other than one's own.

T. P. Uschanov
Research Assistant
Department of Philosophy
University of Helsinki


As a solipsist, I would determine whether an entity has a mind in exactly the same way as the non-solipsist, namely from its speech and behaviour. The only difference between the solipsist and the non-solipsist is that the solipsist says, "The mental states I attribute to others are not real mental states, but only ways of describing behaviour." — The challenge is to make sense of that statement.

Geoffrey Klempner


Arguably the term 'I' presupposes 'the other(s)', therefore a solitary mind could not think about itself or others.

Helene Dumitriu

back

Kambo asked:

How would you place the following aspects in the context of the philosophy of education:

1. discipline

2. democracy

3. freedom

4. punishment

Here's one take on it: animal studies have shown that negative reinforcement, i.e., punishment, is a fast way to stop animals from making only the exact behavior they are punished for. It does not stop that same behavior under other circumstances. In other words, the animal avoids being punished. Period. The same studies have shown that positive reinforcement, while more difficult and longer to make effective, generalizes to other settings and to related behaviors. In other words, animals try to find ways to be reinforced: with food, petting, etc. I'll leave you to work out the implications for disciplining children.

Here's another take on it. George Lakoff wrote a book called Moral Politics in which he talks about the Strict Father metaphor which politically conservative people run their and others lives by. The Strict Father sets rules which must be obeyed or a child will be punished. Liberals, on the other hand, live according to the Supportive Parent model, in which the parent praises a child and attempts to help them. Now what would you expect to happen, given the animal studies above?

Steven Ravett Brown

back

Upendra asked:

1. When we understand a theory of just society, understanding in totality is very difficult, I mean to say we can keep few things aside. If I am right, then could we for a moment keep aside the aspect of distributive justice?

2. The choice theorists and the communitarians tend to advocate overlapping of individual action with collective action, the latter saying that self is shaped by the community or group. This may be a harsh truth, but identity politics is always a destructive factor rather than a constructive one. (Rousseau saying about the interdependency of human beings). I want to begin with a stand that individuality is a crucial factor for this process. What would be your reaction?

On 2, I'm not sure who you mean by the 'choice theorists'. I do know about communitarians, though. I agree with them that the self is shaped by community — I can't see how that can not be true. You seem to agree — though for a reason I do not understand, you label this a 'harsh' truth. What do you mean by 'identity politics'? Why do you claim that it is always destructive?

It seems that maybe you think that, if our identity is always tied up in our community, then we are always narrow and sectarian. Certainly, if we grow up in a narrow and sectarian community, we may end up that way — but not all communities are like this. Further, you seem to think that the only defence against this is 'individuality' — which I take to be another way of saying autonomy.

This seems mistaken to me. It implies that autonomy is something we just have, and that community swamps it. I can't see how we can just have autonomy. It is something that we must develop through learning how to operate in the world — how to perceive what is there, how to reason about it, how to react emotionally to events. All these have underlying genetic predispositions, but none of them can come to fruition without growing up in a community. Hence, autonomy itself is dependent on community.

So, if you are saying that we must strive to create open, educative communities that develop and foster autonomous individuals — those who can represent themselves and negotiate with (and defend) others, in the communal discourse — then I would agree.

Tim Sprod

back

Michael asked:

(1)To check the terms:

a priori knowable prior to experience

a posteriori knowable only after experience

analytic statement statement in which the predicate is contained within the subject, e.g. all bachelors are male

synthetic statement statement in which the predicate is not so contained, e.g. all bachelors are stupid

Kant argued synthetic a priori knowledge is possible. I find his argument convincing. However, I believe I have found an example of analytic a posteriori knowledge: "all bicycles have two wheels". This is clearly analytic as the predicate is contained within the subject, since a bicycle has two wheels by definition. If it did not, it would not be a bicycle. However, if our definition of "bicycle" comes from experience, is this statement not also a posteriori? Humans establish the definition of "bicycle" because they experience many objects, with varying numbers of wheels and need a term that refers to a group of objects with only one common property — having two wheels. Experience of objects with varying numbers of wheels is what makes it necessary to establish the definition of bicycle, thus it is essential to understanding the statement. Therefore, the statement is not merely analytic, but a posteriori. Are there any faults in this reasoning?

(2)A philosopher consults an agony aunt, saying he and his assistant have tried and tried with no success to understand synthetic a priori knowledge. She replies: "Well, it looks like 'im an' you 'ell can't understand it then". Do you like this pun?

(3)Can you recommend a dialogue of Plato that deals mainly with his epistemology and metaphysics and is suitable for an A-level philosopher?

Sorry this is a long question.

An objection to your bicycle example is that "All bicycles have two wheels" is a latently synthetic statement. It can be rephrased as "People take having two wheels to be a condition of being a bicycle". Analytic a priori propositions, such as "5 = 5," are propositions that would result in contradictions if denied. If the last five people in the world died, their corpses would still add up to five — "5 = 5" is true irrespective of anyone's experience. But there would no longer be bicycles — only things that used to be called bicycles when there were people around to call them such.

T. P. Uschanov
Research Assistant
Department of Philosophy
University of Helsinki

back

Reg asked:

I have heard it claimed that the mind-body problem is a philosophical not a psychological problem. Do you agree?

What is a "psychological problem"? I assume Reg does not mean a troubled state of mind that requires therapy. If he means a problem pertaining to a scientific theory of mind, then I suppose one can argue that any such theory must rest on a philosophical theory of how mind and body relate. For well over a century now, psychology has been an independent field of empirical inquiry, not merely a subdivision of philosophy. Each intellectual framework within which psychologists interpret the empirical data of their case studies, however, each school of psychological thought, presupposes an implicit philosophical understanding of the mind-body relationship. Psychologists as such will not explicitly verify or falsify one or another such understanding. That remains a lively field of investigation for philosophers.

Tony Flood


A fascinating question. I don't actually think that it can be answered in any simple, yes/no fashion. There are aspects of that problem, for example, involving the ontological status of qualia, which are philosophical. Other aspects, concerning questions involving specific relationships between mind and body, such as where in the brain we process visual information and how we do that, are psychological.

If the "mind-body problem" is taken to be, simply, the elucidation of the relationships between mind and body, and "mind" is taken to mean "experienced phenomena" (a rather narrow definition), and "body" is taken to mean something like "the objects which realize mind" (a rather question-begging definition in this context), then at this point in time there is still no clear answer. The problem there is that it is not clear what, if any, the boundary or differentiator between experiences and objects is. Do objects have color, or do they merely reflect radiation, for which we, as we sense that radiation and process those sensations, create and assign the color? Does a hammer, to take Heidegger's example, as an object, somehow realize or embody its purpose, or does that realization necessitate the participation of acculturated humans, and if so, is that purpose any less "objective" for that necessity than, say, its shape (which, in order to be described, at least, requires the same)?

Are the questions above psychological or philosophical? They are a small subset of the questions in the context of the mind-body problem.

Steven Ravett Brown

back

Barbera asked:

Is Buddhism a philosophy or a religion?

Is Christianity a philosophy or a religion? For most philosophers before the previous century, it was both, and the same is true of Buddhism. Thinkers within the Buddhist religion have developed Buddhist philosophy. Buddhist religious beliefs are underpinned by philosophical beliefs and arguments. Notice also that, just like Christianity, there are many schools of Buddhist philosophies (which can disagree quite markedly), and there are many sects of Buddhist religion.

Tim Sprod

back

Sonal asked:

"The limits of my language are the limits of my world."

Agree or disagree?

I disagree and think that in recent decades we have been made by philosophers who think language is of primary importance to the philosophy of mind, later Wittgenstein, and Lacanian psychoanalysts to think that entry into the world of the mind starts with language. And that may be so. I'm not at all sure it is. Kleinian object relations theory allows that we have affective relations with another prior to language ability and use and modern neuroscience is compatible with object relations theory. Even if language was the entry into the world it need not constitute its limits. It is the essence of art and poetry that it offers more than is represented or said. It is the point of metaphor that some things cannot be directly said and language has to be manipulated poetically in order for us to try to communicate beyond the limits of ordinary language. Further, as early Wittgenstein said, some things cannot be put into propositions, only shown.

Object relations theory which is about affective relations with others or emotionally coloured relations is particularly interesting here, and it is also true that the unconscious does not operate along the pattern of ordinary language and yet the unconscious is part of our world, our mental life, and our dreams. So much that is emotional seems to be reduced when put into language, such as "I love you". But even if behaviour is included as language, I don't think this is true, unless our imaginative engagement with a poem is behaviouristic. I would say not.

Rachel Browne


I would tend to agree to a certain extent. There are though, I believe, extensions beyond language which are very much part of our world. The thoughts and ideas we hold and express are dependant on language, however, there are sensations which we can hold and appreciate without the use of language. As I sit here on the lawn at the edge of woodland answering this question, I can feel the radiation from the sun on my arms, head and body; I can observe the different plants, the light and shade in the wood, and, somehow, I feel that I do not need any words, any language. Yes, I can name the daisies, the rosebay, the blackberry blossom, etc.; in fact I could describe the whole scene in words to someone, but what the personhears and what I feel are two very different things. Feelings are extremely hard to put into words, in the process the reality is lost and something more artificial is passed to the listener.

Sensations, then, are personal, they belong to the person experiencing them. Sensations of pleasure, love, sadness, joy and hope, cannot possibly be expressed in words, but to suggest that they are not part of our world would be a fallacy. Perhaps the inadequacy of words to express feelings is borne out by the more tangible idea of offering gifts, a legacy of our more romantic past is still observed in the gift of flowers, flowers seem to be able to illuminate someone's feelings far better than any words, in fact there is an old saying, still used in advertising, " Say it with flowers," which indicates that, for many, there is an inbuilt awareness of the inadequacy of language in certain situations. Does putting an arm around someone to comfort them mean more to that person than the blurting out of a few words?

Yes, we need our language, probably the most important and the most amazing attribute presented to mankind, yes, it would be a strange world without it, but let us not run away with the idea that it constitutes the whole of our world.

John Brandon


This may seem like a cop-out, but the book from which that claim comes, Wittgenstein's Tractatus Logico-Philosophicus, ends with another claim: "My propositions serve as elucidations in the following way: anyone who understands me eventually recognizes them as nonsensical." Agree or disagree? If you agree, why are you asking whether we agree with something you yourself admit is nonsensical? If you disagree, then how could Wittgenstein conceivably be mistaken or lying about how he wanted his readers to take his propositions?

T. P. Uschanov
Research Assistant
Department of Philosophy
University of Helsinki


While I agree with the opinions of Browne and Brandon, I think the difference made between "feelings" and the world represented by language is not clearly drawn out. Of course Wittgenstein would not have denied that feelings and subjective impressions and experiences are part of your "world", but without a proper language they remain your private reality, you cannot share them with others, and then they remain even problematic with yourself, because you cannot "handle" them: You cannot formulate "second thoughts" if you cannot formulate "first thoughts" so to say. And in this way you really are trapped in some sort of cage which is defined by your language: You can look out of this cage and understand that reality is much more than what is in your language, but you cannot leave your cage.

Then there are two more points to be seen: Language on the one hand hides and distorts the reality you percive, but on the other hand it expands your experience. The Greek and Romans of Antiquity knew of no personal god in the way St. Paul or St. Augustine knew of HIM, and that difference was the work of language. How could the gospel have spread without language? Some feelings of a "higher being" or of "mana" are not "The Holy Bible" — and are not "The Koran" or the pali-canon or other holy scriptures either. In this way language "creates" reality and feelings which without language would not be "there". One even has asked in this vein if our experience of love was possible before Sappho and other poets gave it expression in her lyrics. Of course there have been diffuse feelings of "being in love" before, but they became much more intensive when they were "formulated" in the words and images of a great poet. Those poets created "templates" for our feelings. Compare this to the effect of great films: Most people "see" Scarlett O'Hara as "being" Vivian Leigh and Rhett Butler as "being" Clark Gable. Once more you have both: A restriction on the one hand — why Vivian Leigh and Clark Gable? There are other actors! — and at the same time an intense imagination, an impressive imaging of persons and scenes of "Gone with the wind".

So once more: Language not only limits and distorts the way we see the world but it creates our world in large part. Look up for instance Nelson Goodmans Ways of Worldmaking (1978) on this. In just this sense we "dwell in the houses of language" or in "towns of language" or in "worlds of language" — and are not only captivated in "cages of language". A house and a prison and a cage have in common, that they are "buildings", artificial constructs of men. And language is a sort of construction-set to build cages, prisons, homes, towns and worlds from it.

Hubertus Fremerey

back

Lisa asked:

Do you believe in reincarnation? If so, can you justify it. I constantly have dreams in which I am a boy. Does this mean I have memories of a past life?

No. No. Ok, fine. No.

Perhaps at some level you want to be a boy. Perhaps, instead, you want to know what it is to be a boy, and your subconscious is attempting to answer that in your dreams. Perhaps neither of those, and you like excitement and activity, and being a girl, perhaps you have been told that you cannot be too physically active. Being a boy would let you be in dreams what you have been prohibited to be in real life, then. Perhaps something else. If it bothers you, get some counseling. Otherwise, just go with the flow, I'd say, and enjoy.

Steven Ravett Brown

back

David asked:

I have been troubled by a concept or question: If people make poor decisions in their life which leads to hardships for them, why should I help or be concerned — after all, it's their own fault! A family member, the bum on the street, a coworker anyone who is not doing very well in life because of their own making. How do you come to terms with wanting to help or actually helping, but still knowing people caused their own problems and should be responsible for their hardships?

It seems to me that people who have troubles through their own fault must feel worse than those who have met misfortune through no fault of their own. They could have done otherwise, and have to suffer regret. And the worse a person feels, the more reason for concern. And then the more reason for concern, the more reason to help.

I always think about the bum in the street "there but for the Grace of God go I". Not that I believe in God, but I know that I could have — and maybe still will — act in such a way that I bring hardship upon myself.

It's strange that you should ask "how should we come to terms with this?" I don't understand this. For sure, it is more difficult if it involves a family member because the fault is closer to home and you have expectations of those close to you that correspond to your own evaluations of behaviour. But the same principle should apply.

Rachel Browne


One comes to terms with the situation that David describes by making sure that in helping one does not communicate that one's beneficiary is not responsible for his situation. Now if you are convinced that helping will only reinforce the irresponsible behavior, then you may be justified, maybe even morally obliged, to refrain from helping. I'm no Objectivist, but I'm struck by the relevance of what Ayn Rand, the apostle of selfishness, had to say about charity, and David may wish to explore further her thought on this topic: "The fact that a man has no claim on others.... does not preclude or prohibit good will among men and does not make it immoral to offer or to accept voluntary, non-sacrificial assistance.... when it is offered in response to the receiver's virtues, not in response to his flaws, weaknesses or moral failures, and not on the ground of his need as such." ("Charity," The Ayn Rand Lexicon: Objectivism from A to Z, Harry Binswanger, ed., New York: New American Library, 1986, p. 69.)

Tony Flood

back

Kevin asked:

A friend of mine holds two beliefs that I feel are contradictory. First, that life is objectively meaningless. We set goals, attain them and set other goals with no ultimate end. Second, that some careers have (subjectively) more value than others. This, she claims, is measured by the effect the chosen career has on the world e.g. a doctor is more valuable than a cashier because the doctor affects more lives.

It seems that belief 2 cancels belief 1. If life is objectively meaningless, then a career's affect on others has no bearing on value unless the lives they affect can be shown to have objective meaning. If not, what's the difference?

I think your friend can sustain her argument. By objective values, my guess is that you mean values that would exist even in the absence of humans. My view is that such an idea is meaningless (unless there are other being(s) to value these things — and then it is hard to see why they should apply to us — unless those/that being made us... but this is getting off the track). To have value, we must have valuers.

Values can, in my view, be created by a community of valuers. Within that community, we can talk about values (such as human well-being) which, while not being objective in the sense above, do transcend individuals. They are created within what Wittgenstein referred to as a way-of-life.

Tim Sprod

back

Nik asked:

I have a friend who was going for an electrical engineering degree when he wrote an excellent paper for English and was "discovered" by the philosophy department. They have convinced him to get a degree in philosophy but he is going to also get his degree in electrical engineering. Combining the two create a sort of 'scientific philosophy.' He was explaining a relatively new movement called the Human Project that explored man's ultimate goals with technology. Do we want to better the world for us or are we trying to conquer it. Do you have more information on this idea? Web sites, books, names of philosophers, or anything published at all. Thanks.

Try this for an interesting take on the subject, with rebuttals.

http://www.spiked-online.com/articles/00000006D94A.htm

Steven Ravett Brown

back

Ian asked:

Given that life begins and ends on the Earth for every person, now and into the future, i.e. the possibility of an 'after-life' for the purpose of this question is nil, how does one overcome the sense of futility and instead obtain a sense of purpose for one's own life and others, knowing that ALL are destined for the same fate?

Many philosophers (including such polar opposites of each other as Martin Heidegger and Karl Popper) have suggested that, far from rendering everything futile, the finitude of life is in fact the only thing that can give it meaning. If there was too much life available, there would be inflation the way there is when too much money is circulating in the economy. Like money, life would be completely worthless if everyone had as much of it as he wanted — you "couldn't give it away for free".

For a nice illustration of this argument, I can recommend the last chapter of Julian Barnes's novel A History of the World in 10 1/2 Chapters, in which a man becomes mortally bored in the afterlife because his every wish is always instantly granted — and he can't even be thrilled by the risk of dying in adventure or the like, as he has already died.

T. P. Uschanov
Research Assistant
Department of Philosophy
University of Helsinki


Ian believes that that it matters whether one can overcome the sense of futility and obtain a sense of purpose, given his factual presuppositions, which he regards as "given" and "known." He wants to know how to make sense of this mattering. Now to the degree that Ian thinks that this is a real question, to that degree I would want to hear his reasons for presupposing what he does about our destinies. To the extent that skepticism about his reasons can be sustained, to that extent his problem loosens its grip. The question of the meaning of human life in prospect of the eventual heat-death of the universe is a serious one, but it is up to the one posing the question to make the case for its gravity, not for others to make it for him and then to answer their own construction. They risk misinterpreting his question in order to make an answer more likely. But the philosopher may have set up the question in a way that makes an answer impossible, and so he will probably not recognize his question in their interpretation of it.

The short answer is that perhaps one doesn't "overcome" or "obtain" if what Ian presupposes is true, but perhaps he has presupposed what is false. The feeling that this short answer is unsatisfactory should goad him to explicate and defend his presuppositions.

Tony Flood


Yes, this is the position that every religion wants you to believe. But let's look at it. Suppose that you (and everyone) lived forever; we had "souls" (whatever those are) and our lifespan (never mind the body — that's just "clay") was thus infinite. Ok... now, given that, why should we act morally, compassionately, etc., while we're alive on earth? Our lifespan here is literally an infinitesimal fraction, totally insignificant, of our total lives, which are infinite, right? So what difference do our actions over any finite time span make? None whatsoever. We could be the most vile tyrants for, say, 50 million years, but in an infinite span, that is precisely as long (relative to our whole span) as any other finite length: five seconds, for example. I'm not being metaphorical here. Given an infinite life span, if any finite part of that is removed, there is still an infinite length of time left (in fact, you can remove infinite amounts, if you do it in particular ways, and have infinity left also). So why bother? You have an infinite time left in which to change things or to act morally, however long you have been evil, however much pain you have inflicted.

It is, in fact, just when we realize that we have only a finite life span that we should realize that we must act ethically in that span. That's all we have; we can't correct for it in the next few billion years. In addition, although we do not know the consequences of our actions, we have evidence that "good" actions produce "good" (I'm not going to go into what these mean here) results. So, we will be dead, and indeed everyone else now living, in, say, 100 years, so what? The odds are good that humans will still exist, and human culture, and that our actions may have effects, which we cannot foresee (except, as I say, to expect that moral actions will have good results), for an indefinite length of time. Who knows? But it's possible that some act of ours may echo, even covertly, for weeks or for aeons. We just don't know, and indeed probably (given chaos theory) cannot.

As for purpose. Here's another one the religions love to seize on. God gives us purpose, right? Otherwise we'd have none. Poor us. Well, how about this: we can create our own purposes. Where does a god get its purposes, anyway... from its god? A nice regression there. How about your trying to come up with a purpose, and living that purpose, with the expectation that even though you will die, that purpose will have given your life meaning, may help others to find meaning, and may in addition, through your actions based on it, have beneficial consequences for the future?

Steven Ravett Brown

back

Chantelle asked:

What is Freud's view on the nature and justifiability of religious beliefs?

According to Freud in The Future of an Illusion, we cannot justify religion. Religious doctrines are bodies of knowledge which cannot be tested since they are teachings handed down from ancestors, and while it might seem that we ought to believe them, and most people do, there is no reason to believe writings from the past. Whilst a particular person might come to believe through divine revelation, Freud says that all we can say about this is that the person has had a mental experience giving rise to his conviction, but it doesn't follow that everyone ought to have such an experience.

However, Freud does say here that the man who adds religious beliefs to his "knowledge" is richer, which is to say we can assume there is more to reality than can be discovered through the scientific route of testing, evidence and justification. He calls religious knowledge an illusion, but claims that illusions (as opposed to delusions) are not necessarily in contradiction to reality since it is not impossible that an illusion, which for Freud is essentially derived from or motivated by wishes, can be true.

Freud doesn't justify religion, but looks at the origin of its illusory nature in the psyche of man, and speculates that this origin lies in the helplessness of childhood and the desire for protection against danger, the unknown and fate. This exalted protective father figure, God, can protect from unknown fate.

In his later work Civilization and its Discontents, Freud took the view that religious beliefs are delusions, which is to change his stance from saying that such beliefs might be true, to claiming that they are not. This is because the instinct to religion is "infantile", an early instinct which is the expression of the desire to be protected from a hostile world. In psychoanalytical theory, man grows and develops through many stages and develops individual grown-up instincts and sublimations which can lead to a satisfying life in which the world doesn't appear hostile, and there is no wish for protection. In contrast, the consolations offered by religion don't have developmental efficacy in that religious beliefs don't offer the development of individualistic choice and adaptation to the world. So no longer would Freud claim that religion is not the type of thing that can be justified. The later view is that it cannot be justified as being beneficial to mankind.

Rachel Browne

back

Mohammad asked:

What is a 'refutation' in Popper's view?

One of Popper's books is called Conjectures and Refutations. A conjecture is a proposed theory of how some scientific thing works. A refutation is an experiment that does not turn out the way it was predicted to, on the basis of the conjecture. It falsifies the theory.

Tim Sprod


According to Popper, the theorist is not obliged to treat every experiment which fails to turn out as predicted as a 'refutation'. If the theory in question has made lots of successful predictions in the past, then an alternative, less radical course of action is to put forward auxiliary hypotheses which explain why the prediction failed in this particular instance. And that's where things get difficult. Because it is left to the judgement of the researcher in each case of a putative 'refutation' whether to trash the theory or try to save it.

Geoffrey Klempner

back

Carmen asked:

I got into an argument with someone over the question: 'What I tell you three times is true' (Lewis Carroll). Might this formula — or a more sophisticated version of it — actually determine what we believe to be true?

To me it is clear that the answer is no. I couldn't be determined to believe something just by being repeated something over and over. If one told me that aliens exist several times, I still wouldn't believe it. Yet I do accept something as true when a teacher tells me something. So I guess authority has something to do with it. Nevertheless, my friend argues that if you're told something over and over, you eventually end up believing it. Would you please comment on this?

First of all, this isn't, in my opinion, a philosophical question. Clearly, from the standpoint of logic, the number of times a falsehood is repeated is irrelevant. If you're asking whether we accept authority... yes, much more than we should, in my opinion, and training in philosophy, hopefully, counters this. But if you're asking, very generally, what inclines us toward belief in something... that's an enormous subject, too much for this forum, and in addition isn't, in my opinion, strictly a philosophical question (although Hume, Descartes, Locke, and Berkeley, to name a few, did essay answers to this).

Now to the specific issue. Depressingly and unfortunately, research (in psychology) has indeed shown that repetition inclines us to feel better towards something. And that increased positive feeling could indeed lead to belief, eventually, although a) it shouldn't, probably, and b) even if it did, our feelings almost certainly shouldn't affect our beliefs so directly. But they do. On that latter, and indeed a strong argument that feelings should affect our beliefs (but more subtly) try Damasio's book: Descartes' Error.

Steven Ravett Brown

back

George asked:

I am a subscriber to the Philosophy Pathways e-journal.

Would you be able to confirm a comment attributed to Wittgenstein and possibly cite its source? The comment is:

We search for what's hidden, dupes that we are of a dream of depth. Anything to avoid the unbearable presence of reality.

The quotation is from pages 20-21 of Terry Eagleton's novel Saints and Scholars, in which Wittgenstein is fraternizing with James Connolly and Leopold Bloom in the Ireland of 1916. But I'm sad to report that it was penned by Eagleton; there is nothing even remotely resembling it in any of Wittgenstein's published or unpublished writings and utterances.

T. P. Uschanov
Research Assistant
Department of Philosophy
University of Helsinki

back

Dora asked:

Can you please explain the view of time in the theory of special relativity? This is part of my philosophy 101 homework but as you can see I am very confused.

Think about rock sediments. Zillions of years ago, or whatever, little shells, etc., fell down through the ocean and built up layers. After that, the ocean dried up and trees grew there, died, and built up a layer of dead trees. Maybe after that there was a volcano which covered it all with a layer of lava. Then more trees, and we get dirt on top of all that.

So you cut through and look at those sediments. At the bottom, there are these little shells, compressed. Over that, there are the fossilized trees and their stuff... and so forth. So we have a record of time, sort of frozen and extended in space, through the sediments. Ok? Well, that's the view in special relativity. If you could look at any object, down through time, you'd see objects the way they were, say, an hour ago, overlaid gradually with their changes through that hour up to the present. So that total view of the object, including all those different times, would be like one object, extended through time. Get it? So if you cut that one object at some point corresponding to an hour ago, a week ago, or whatever, you'd see it, like the sediments, as it was then.

Well, the idea is that all objects really are like that... extended in time; we just can't see the "time" part of them, except bit by bit, because we're stuck in the present (which "moves" forward... never mind about the problems here...). So if all objects are like that, then everything is like that, and the whole universe is a kind of huge brick, but in 4 dimensions (since all the 3-dimensional stuff is extended in time), floating (roughly speaking), and if you just had the right perspective (a 4th dimensional one) you could peek into it anywhere and see us, birds, dogs, flying saucers, etc... sort of frozen but changing along their length (the time length).

That's pretty much it for the metaphysics of special relativity. Time is another dimension, not like space, but close enough that the above picture gives you an idea. It is not a very good metaphysics, as it stands, because of problems with mind and consciousness. There's a sci-fi book, October the 1st is Too Late by Fred Hoyle, which attempts, unsuccessfully, to deal with the metaphysics. Good book, though.

Steven Ravett Brown

back

Faruk asked:

Why do we need history of philosophy?

We need to unpack "we." Some people need to know the history of philosophy for general reference purposes. Philosophers need to know it in order to avoid wasting time when they decide to answer a question. Why assume that one is the first philosopher to raise it? Why risk wasting time reinventing the wheel? Of course, one can use the "need to do more research" as an excuse to evade the responsibility to do one's own thinking, but one has no basis for claiming that it has not been thought before — let alone scrutinized and challenged — if one has not read what others have done in that area of inquiry.

Is knowledge of the history of philosophy strictly necessary? No. It is theoretically possible for one to answer all questions that other would recognize as philosophical without one's being aware that anyone else had ever attempted to answer them. Without referring to the diachronic conversation that is philosophy's history, however, one cannot know that one's set of answers contributes to philosophy. Such an isolated thinker would be cut off from the life of that conversation, and both he and others would be the poorer. Practically speaking, such isolation is impossible. Also, as in other areas, knowledge of the history of philosophy is a matter of more or less. Unless one is interested in the history of philosophy for its own sake (whatever that means), one fills in the gaps of one's knowledge as the need arises, always grateful for interlocutors who make "old" texts live prospects for reading rather than the dead inventory of a reading list.

Tony Flood


As the Spanish-American philosopher George Santayana put it: "Those who cannot remember the past are condemned to repeat it." (Perhaps tellingly, Santayana himself has been omitted from most of the standard one-volume histories of philosophy available today.)

T. P. Uschanov
Research Assistant
Department of Philosophy
University of Helsinki

back

Allison asked:

What are the different types of philosophical inquiry?

How has the understanding of philosophy changed from the time of the Greeks to our modern understanding or common understanding of today?

These questions have been lying around for a while. I just saw them again today... I'm not going to tackle the first one.

The second one is more interesting, in a way. Let's do a little comparison. Suppose that you are a reasonably educated person, young, with a good, solid high-school (or the equivalent) education, or even the beginnings (2-3 years) of a college education. You read books, newspapers, etc., and you can think about, a bit, and talk about, a bit, topics like relativity: "Einstein showed that mass and energy are conserved" or "space is warped around stars" or something like that. Now, first, does this make you a physicist? No, of course not. Second, do you understand physics? No, not really; not until you can explain why these and many other statements are true (or not). Third, do you understand what it is to be a physicist? No. Not until you can explain how the explanations of the above statements, and others, were arrived at. In other words, understanding physics is one thing, far beyond playing with a few of its ideas. Knowing what doing physics is, is another thing, beyond understanding physics. And, finally, actually doing physics, being a physicist, is yet a third thing, beyond the other two. And then there's being a really good physicist, which is even harder.

Take an example. I mention "energy" above. What's that? Well, when you learn basic physics, you learn that to accelerate a mass, it takes a force proportional to that mass. F = ma. Clear so far? Now, if you know calculus, you can then derive E = 1/2mv2, if you integrate over time, because acceleration, which we started with, is the change in velocity over time, and energy is the integration of force over time. Or you can go the other way, start with simple motion, i.e., velocity is distance per unit time, v = d/t. You can then differentiate by time and get a = v/t, i.e., acceleration is velocity per time. Still following? So we have, from the first equation, F = mv/t, right? Multiply both sides by time (or integrate), and we have E = mv2, which is basically where we were above (I'm not going to bother with the constant here). Hey, relax, we're just starting basic Newtonian mechanics. We've got about 200 years to go yet before we get to Einstein... the rest of mechanics, optics, electromagnetics... then when it gets past "special" to "general" relativity, and on to quantum mechanics, the math gets quite seriously difficult. You still want to be a physicist, to "know" physics?

There's about 2-300 years of physics to learn, from Newtonian on up, from scratch, because it's all interdependent, and you can't understand relativity, not to mention do relativistic physics, without understanding the Newtonian physics on which it's based (sorry, but the idea that "paradigm shifts" made all earlier work irrelevant is garbage, as any physicist knows). Then, of course, you have to have the right kind of mind, to do the math, to think analytically. Not everyone has that. All in all, not easy, and far beyond playing conceptual games with "warped space", whatever that means when you read it in the newspapers. Yes, fine, you can take "non-major" courses in physics, with no or watered-down math, and simple concepts. At the end of those, if you're very bright and have a very good teacher, you might possibly know a bit of what you're missing, and that you're missing most of it. Sorry if this sounds negative, but this is just not simple stuff, not if you really want to understand it. It just may be the case, depressing as it sounds, that in order to really get the concepts, you have to start young, as in before high school, just as in order to become a good musician you have to start young. Or, if you start older, you have to be extremely dedicated.

Well, here's the shocker, people... philosophy is about 2-3 thousand years old, in the West. Physics, not to mention biology, engineering, most of math, and dozens of other fields are derived from concepts in philosophy. Now what do you think it takes to be a philosopher? Well, just look at the paragraphs above, and multiply by 10. Yes, that's about it (depending, actually, on where you want to go in philosophy... you can keep it simple if you want, and just learn the equivalent of physics... although I disapprove).

So... you want someone to tell you how "philosophy", that enormous, 3000-year collection of concepts, symbolisms, history, literature, logic, and on and on, has changed since the Greeks. Well, now you know why this question has been sitting around for so long. I could take a stab at telling you how physics has changed; I have a degree in that, and it's only a couple of centuries of work. Easy, haha. Philosophy? Forget it.

Steven Ravett Brown

back

Julie asked:

I was just going through your website and was reading all the questions and answers people ask you! So, I decided to ask you a question myself and get your expert advice.

I like photography and I have an interest for learning more philosophy. Through examining your website I wanted to take a closer look into photography and philosophy, meaning looking at photography using philosophy and analyze it a bit!

I was wondering about your thoughts on abstract objects in photography and whether or not you feel photographs can depict Plato's forms? and whether or not they can depict abstract things of any type?

I was also wondering if you felt photographs can be used to depict a photographers' subjective impressions?

I have formed many opinions concerning this topic through photography and just wanted someone else's point of view from the philosophy angle. I know these questions are not straight forward but I thought I would just see a few of your views on the subject!

I would love to hear back from you concerning this topic. Thought it might give you something to think about and wrap your mind around!

I must confess that I am no expert on the subject of photography; like many others I enjoy using my camera, which is efficient and idiot-proof. I am, however, interested in art and spend some of my spare time painting in acrylics.

There was for some time a general feeling amongst art lovers that the camera could never replace the painter. In the beginning the box camera was nothing more than a gimmick for family entertainment, also a photographer was thought of as someone who took family portraits, weddings and school groups. However, developing quietly behind this banal facade was a highly efficient medium for communication, in fact, it was fast becoming a specialised form of art which incorporated not only a means of communication but was also a provider of instant records in time and space. Art, of course, always had something to say, it was never just a case of painting a picture, it has been used extensively in the past as a vehicle of politics and propaganda. Photography has proved that it is not only just as capable of carrying forward this commitment but has, in my view, enhanced this facet of the art world with its speed and accuracy.

All forms of art have always progressed hand in hand with developing technology, probably photography, more than any other form, and by its very nature, depends on this development. Perhaps where other forms of art pioneered the path of development by their demands on technology, photography lay somewhat dormant waiting to follow the inspiration and motivation in the developing technology. In other words, art said, "This is what we want, provide us with it." Whereas, technology said to photography, "This is what we have got, learn how to use it." Though the means to the end have been different both painter and photographer have achieved their objectives.

You ask if photographs can be used to depict a photographer's subjective impressions. I believe that this can be achieved in photographs just as successfully as in any other expressive art. A photographer, like a painter, will require to think about composition, both will have to decide on how to present meaningful subject matter, both will have to exhibit creative flair, both, by the skillful use of light and shadow will show that awareness of contrast and fine detail necessary to impart the personal content which brings the work alive. In this way the character and feelings of the photographer become the soul of the subject matter. A photograph, then, can contain the same form and meaning as a painting and the photographer, just like the painter, is recognised by his/her style. You will know more about the techniques of photography than I do, but I feel sure that other factors intrinsic within those techniques will contribute to the subjective talents of the photographer. When a painter is advised to paint what he/she actually sees, in a similar way the photographer must capture what is seen. In this the photographer has a definite advantage over the painter, for where the latter requires to remember how the scene was influenced by the light at the start, which could be some hours before, the photographer captures the scene instantly, what is seen in that instant the photographer secures.

Like a painter, then, a photographer can give a lot of him/ herself to the subject matter, and there is no reason why those conversant with the work of different photographers would not be able to recognise the style and individual talent embodied in the subject matter of a masterpiece.

A claim has always been made that a good artist can improve on nature by skillful use of artistic licence, the question arises, Can a photographer achieve the same? Well, a photographer certainly could not add an extra tree to a composition, or move a cathedral as Constable did. There would also be some difficulty in trying to produce the 'impressionist' effects of Turner. You will, no doubt, hold your own views on this, but, for what it is worth, I, personally, do not see any conflict here. Photography is a different form of art; there is no doubt that rugby and cricket are both games, but to suggest that cricketers are deprived of goal posts and could not, therefore, compete with rugby players would be a spurious and stupid argument, cricketers do not need goal posts to play cricket. Just as rugby and cricket are games within their own right and one is in no way reliant on the other, so with painting and photography. That photography is an art form in its own right is now universally accepted. Of course, public interest has itself increased with the availability of easy to use cameras and higher grade films. However, above all, it is well understood that pictures speak louder than words, and it is as a valuable communication medium that photography is unsurpassed, but the value of that communication depends on the skill of the photographer to impart the feeling and emotion into the subject to make it more than just a picture.

John Brandon


I was working at the library of the Viennese School of Photography and Graphic Design when I saw your question. It indeed is "something to think about and wrap my mind around". The only problem is, I do not know much about photography.

Probably the most frequently asked philosophical question concerning photography is, whether photography is art or not. More than 150 years ago the French history painter Paul Delaroche declared, "From today painting is dead." He saw it as a powerful new medium that could replace painting in representing and documenting the world much more precisely and in much less time than could be done with brush and pigments. This of course is a statement from a very "naturalistic" point of view and perhaps the invention of photography is partly responsible for further development of arts away from mere lifelike painting.

Nowadays Fine Arts and Photography don't seem to be in an "either ... or" relation any more, but in a "both ... and ... " relation. Photography is used by many artists as either an aid (such as Chuck Close who's work would be impossible without the aid of photography) or as actual artwork. Some artists use it so they can go beyond the boundaries of painting/drawing/sculpting etc. Man Ray paints "what he can't photograph, and photographs what he can't paint."

I think "photographs can be used to depict a photographers' subjective impressions", as there are many possibilities to go beyond "click and process":

  1. Like a painter a photographer subjectively looks for the perfect scene and can arrange it before depicting it
  2. Printing can be a long and intensive creative process if you wish to make your print 'perfect', and
  3. image manipulation by hand or computer requires a lot a time, effort and patience — similar to a lot of artistic methods.

"To me it seems that a photograph of a pretty landscape simply is another natural phenomenon like a landscape — a paper landscape or a fingerprint or footprint of a landscape.", I recently read in a magazine. Can you feel the wind, can you smell the flowers or feel the real surface on this "paper landscape"?

Still, most of us, me included, are fascinated by these paper landscapes. To me, looking at good photographs is like putting myself in the photographer's position and therefore a good way of travelling without actually being there.

Simone Klein
http://www.sophiasworld.at/

back

Edward asked:

What did Albert Camus mean when he wrote or said the following quote:

The rebel can never find peace. He knows what is good and, despite himself, does evil. The value which supports him is never given to him once and for all.

I don't know this quote, but I have been using The Rebel on and off in my courses over the last 7 years and can clarify the meaning. Quotes from The Rebel are especially dependent on the context because coherence is not one of the book's great qualities. It is a bubbling cauldron of philosophy, reaction, history, manifesto, call to arms, book review and journalism. While it fails on all these counts taken singly, it succeeds in being unique and in making the reader catch his breath. In the last analysis The Rebel is a plea for the individual; and that is who the rebel is. It is Camus and — he hopes — his solitary reader.

The value which supports the rebel is never given to him once and for all, because value is not absolute, it is in the making or on the way. The original rebel was Ivan Karamazov from Dostoyevsky's novel The Brothers Karamazov. Camus' The Rebel, and the idea of it, comes from a chapter of that title in the Dostoyevsky novel. Ivan rebels against God, as did Camus in real life, for justifying suffering as a means to a greater end. To God's instrumentalism Ivan Karamazov says No. This leads the door open to a nihilism where "all is permitted", to a Nietzschean Yea-saying (although Camus defends Nietzsche from misinterpretations along these lines). Nihilism is the shape of every alternative 'end of history' and of man as far as Camus and his rebel are concerned. To nihilism the rebel says No. This is a No to man, history, reason, consciousness, art and everything else that would replace God as a 'higher end'.

Rebellion itself is the principle. This is the thesis of the book. Given this principle, the rebel can never find peace. In principle, he cannot. For the rebel every superior motive for action, or inaction, finds its limits in evil. To join the revolution leads to murder, not to join it condones the murder going on. Murder either way. Therefore, despite himself he does evil. But the rebel knows what is good. Freedom and justice are good, and they find their limits in each other qua individual life. The right of the individual to life — to freedom circumscribed by justice and justice supportive of freedom — is Camus' thought at the meridian. With this thought, it is no longer true that, 'despite himself he does evil', for the rebel, instead, finds himself condemned to live for those who, like himself, cannot live, i.e. those who suffer injustice and imprisonment by the instrumental purposes of God or of ideology. The principle of rebellion turns into what Camus calls "a strange form of love".

Matthew Del Nevo
www.sicetnon.com

back

Alex asked:

Dear Friends,

I have already studied philosophy for several years, but I really can't make up my mind to say if it is possible to transfer this knowledge to anybody, i.e. to teach somebody philosophy. Have You ever had the same problem and how do You solve it?

As you can see, those of us on this website are attempting to solve this problem in a way involving your choice of questions, our interest, and modern technology. But that's a superficial answer, in a sense; I assume you're asking something more along the lines of detailed methodology, and detailed content. Well, then, there are of course always Socrates' methods. I won't go into them; if you have indeed studied philosophy you know them. Why don't you think they work? They did for Plato, Socrates' pupil.

What is philosophy, in terms of what can be taught? Is it primarily content or primarily methodology? If content, what is that content; if methodology, how can that be described and taught? Speaking of content, surely you can think of many methods to teach that?

What about methodology, i.e., techniques of thinking? Well, how do you teach methods of clear and effective thinking? How is mathematics taught, for example? First, it's learning by doing, right? Second, one critiques technique in process. One might think of teaching technique in terms of teaching a craft; and how is that done? Now, I'm not asserting that teaching is easy, especially teaching methodology, no indeed... but I am asserting that it is doable, given willing pupils (and that's yet another issue, isn't it). However, there are some characteristics of philosophy, in my opinion, that make it particularly difficult. First, it primarily deals with highly abstract concepts, and those are not usually easy to grasp clearly. In fact, accomplishing that understanding is part of what must be taught. I believe it is de-emphasized in our culture because of that difficulty, but also, I believe, because learning critical and clear thinking leads (hopefully) to questioning the beliefs of one's teachers and of one's culture, and who wants a pupil who does that? The more repressive a culture, the less philosophy is encouraged (and I do not count textual interpretation, per se, as philosophy). Further, in any culture, philosophers are somewhat like monks... they preserve learning, they reason and think abstractly, they produce systems of ideas, but they do not directly (although I believe — and of course I'm prejudiced — they do produce valuable products) produce "goods" which people can pay for (except as teachers... and there we run into the second problem, when the pupils go home and defy their parents). What we need is a culture in which philosophy is valued, and that in turn requires an attitude toward questioning oneself and others which is usually very difficult to tolerate.

So it seems to me that teaching philosophy must go beyond conventional teaching of methodology and content into the realm of the proper means of the philosopher interacting with others in order to influence them in a way that induces a kind of questioning which is almost inevitably threatening, without being threatening. A good trick, wouldn't you say? Oh well.

Now that we've got it analyzed (joke), why don't you come up with some solutions? Historically, as far as I can see, the best solution has been to keep philosophers in the "academy", where that type of thinking is tolerated because it is isolated from the rest of the culture... a kind of intellectual ghetto, if you will. Is that the best solution? Well, it has endured around 300-400 years, in this culture... not too bad a record. Then there's this forum, and some others like it, and in addition there is the new field of philosophical therapy, which I do not expect to do very well, precisely because of the above problems.

Steven Ravett Brown

back

Shawn asked:

Explain Soren Kierkegaard's theory of "The Stages of Life, Aesthetic, Ethical and Religious".

The aesthetic stage is a life in which the person lives for sense pleasure, wine women and song. It is life geared to comfort, leisure, leading if the person is not careful (and what's to stop him?) to voluptuousness and hedonism. This is the 'good life' in the sense of lapping up the goods of the world.

The ethical stage is the moral life. It is dutiful life — the Kantian life. It is a life that adheres to reason rather than pleasure. This was also Kierkegaard's stage, as he admits. The religious life is therefore speculative from Kierkegaard's point of view. Abraham was a type of it, also St. Paul. The religious stage is one in which the person has placed themself into the hands of the living God (cf. Hebrews 10: 31). This means something beyond reason, but which is not irrational. It means one has become a 'redemptive' personality, one takes the evil doing of others as if it were one's own and tries to atone for it. All this is best seen in Dostoyevsky, who I think, without knowing Kierkegaard, knows what is meant by these stages and knows more than K does about them in fact. Crime and Punishment depicts the difficulty and discontinuity of these Stages. Their discontinuity is their most interesting aspect and what lends them to philosophy rather than psychology. Sonia in Crime and Punishment is at the Religious Stage, her person has become redemptive.

Matthew Del Nevo
www.sicetnon.com

back

Sandip asked:

What is the view of contemporary philosophers about 'moral relativism'?

Slavery was practiced in the the past which is considered immoral today (right?). Was it considered immoral those days also? If it was does it mean some of the world leaders who had slaves were immoral? If not, then the definition of morality has changed, right!? So why is there objection to moral relativism? Or should the term be 'contemporary moral absolutism vs relativism'?

Moral relativism is not the thesis that what is regarded as moral or immoral varies from place to place or from time to time. That is uncontroversially true, as your example demonstrates: Plato and Aristotle wrote extensively about ethics but both of them took the moral legitimacy of slavery for granted. Today we (most of us) take a different view.

Rather moral relativism is the thesis that morality itself, and not merely its public perception, can vary with context. So a moral relativist could argue that slavery was ok in the ancient world, but wrong today: Plato was right not to condemn slave owners, and we are right to condemn them. Opponents of moral relativism think that this is nonsense; the standards of right and wrong should apply everywhere and at all times. So if slavery is wrong for us it was wrong for Plato too, and his failure to condemn it was an error (perhaps an understandable or excusable error, but still an error). Of course, our moral standards are not necessarily right either, although we might hope that we have made some progress, as we have in other areas of knowledge.

Some contemporary philosophers have deep misgivings about all forms of objective truth, including (perhaps especially) that of moral judgements. This has led to their endorsement of moral relativism, or at least the accusation that they have done so.

You might find the following two on-line encyclopaedia entries helpful:

http://www.wikipedia.com/wiki/moral+relativism
http://www.utm.edu/research/iep/m/m-relati.htm

If you are looking for something more substantial, I would recommend Mary Midgley's Can't We Make Moral Judgements? (St. Martin's Press/ Bristol Press, 1991).

Andrew Aberdein


Let's say, for the sake of argument, that there are moral absolutes, and that slavery is absolutely immoral. Let us say, in addition, that some world leaders have in the past, and do now, regard slavery as moral. Does that mean that slavery was or is moral? No, it means that those people were and are mistaken about morality. Your argument above does not support moral relativism, it merely points out that people can be and have been wrong. In other words, their "definition" of morality is not, given that there are moral absolutes, "the" true definition of morality. So your argument is irrelevant to the issue of absolute vs. relative morality, although not, of course, to the relative understandings of morality.

Steven Ravett Brown

back

Damon asked:

What did Jaspers mean by ciphers?

He meant that everything meaning-bearing, language in its broadest conception — everything that speaks through us and because of us — is symbolic. To get this point you need to realize the a priori distinction Jaspers makes between what something is and that it is. Language functions in the normal way when referring to what something is (quidditas). But this leaves out the problem of its being, its beingness (the actus essendi or Existenz). Existenz comprehends all that is. Existenz is the transcendent horizon of phenomena, against which as well as in which things live and move and are what they are. Things (entities and phenomena) are "ciphers" of this Existenz, which we cannot talk about directly, in the manner that we can say what a thing is directly, clearly and coherently. Whatness presupposes a subject-object dichotomy, but thatness transcends the subject-object dichotomy and provides the horizon in which it can occur, its pre-condition. By talking of "ciphers" Jaspers is pointing back to a whole dimension of language that discursive thinking, logic and scientific thinking in particular forget. I hope this helps.

Matthew Del Nevo
www.sicetnon.com

back

Sean asked:

How does one become a professional Philosopher? My writings seem to revolve around the Origins — of life, of matter, of the universe — but I don't know where to go from here. I've applied to Humanities programs, but if I don't get into University now I may not be able to go for a while. Is a degree necessary for a naturally intuitive, able minded Philosopher to succeed in this field? I want to be well known one day, how do I do it?

Do you want to be "known" or do you want to be a philosopher? Are the two the same? What is a "professional" philosopher? You might think about those a bit. Here's what's necessary to be a "professional philosopher" as I understand that phrase. 1) learn philosophy extremely thoroughly and well, 2) learn to think very precisely, 3) find some original problems, or at least, problems recognized as such by a community of philosophers, 4) publish some approaches to solving, or if you're very lucky, some solutions to those problems, 5) defend those against the inevitable attacks, 6) use the above to get a job teaching philosophy, or to write popular books (see below).

1) requires a great deal of reading, usually guided (i.e., a graduate education in philosophy, usually). 2) requires a great deal of writing, criticized in detail (i.e., a graduate education in philosophy... usually). 3) requires 1 and some of 2, plus creativity... which latter seems very hard to teach. 4) once you've got the above, just look at journals and see which ones might be sympathetic to your point of view. Send papers to them. 5) more of 3 and 4.

In the meantime, if "professional" implies being paid for what you do, you have to find a job "doing" philosophy, i.e., teaching it, usually (which isn't usually part of 1-5, above). Is a degree necessary? No... if you're exceptionally brilliant, motivated, you work at it continuously, and you earn money by some other means... no. Colin Wilson might be considered a philosopher who succeeded without all that (but he's also a novelist, who has tried, initially successfully, to combine the two). I don't know of any modern professional philosopher, without a doctorate, who is not a novelist and making money that way, who has succeeded in doing nothing but writing technical works in philosophy, for the simple reason that you don't get paid for those (unless you're already well-known as a philosopher... then popularizing your work can sell it; or writing textbooks can sell them). Now, there might be someone who inherited wealth and has only written, with no degrees... but I can't think of who that might be, offhand, except Wittgenstein, perhaps, in his early years (I can't remember whether he had a doctorate... I don't believe so... later on, however, he repudiated his family's wealth, and taught). Nietzsche inherited money, as I recall. You can go to the books and look up biographies, if you want... but now, I think, it is virtually impossible to teach without a doctorate, and how else are you going to eat and do philosophy, long-term?

Steven Ravett Brown

back

Caroline asked:

I am going R.E. course work at school and I can't find much information on my third piece. The question is:

"For a Christian to die for his/ her beliefs does not make sense. It is better to live for them."

Do you agree? Give reasons for your answer, showing that you have looked at both points of view.

I don't think there is any particular 'information' that can help you. It is your own reasoning that is called for. There are various ways in which things "make sense" or do not. The everyday way in which things "make sense" accords with common sense. The philosophical way in which things "make sense" is logical. The Christian way in which things "make sense" is 'theological' — i.e. it presupposes God, an absolute value. Your question does not distinguish between these three, but you need to. Therefore, from a common sense point of view:

It does not make sense to die for your beliefs because then you are no longer around to be active on their behalf. If you die for your beliefs, you lessen them, because they have one less supporter, therefore, it does not make sense to die for them and in fact you should try not to die for them, but to live for them and make them live for you.

On the other hand, it makes sense to die for your beliefs sometimes, or people may not believe that this belief is serious or to be taken seriously. For instance, you may die for what you believe to show how seriously you hold it, and also how worthwhile such a belief must be if you are willing to die for it. Or you might die for someone else — i.e. in trying to save their life — because you believe that you can't live without them. From a logical point of view:

Beliefs which necessitate your dying on their behalf are self-contradictory, because while you believe x or y in order to live for it, to believe it "unto death" eradicates the very source of belief, which is oneself. The belief cannot live without you. Therefore the true believer will live for what they believe and live up to it.

On the other hand, it is not necessarily self-contradictory. If it is a matter of life and death, you may need to choose death in order to show how your belief works; for instance, if I believe in the 'right to die', in euthanasia, I do not die for my belief as such, but for the human right which it upholds, this makes my death right according to what I believe. Or I could die for a good cause, like the progress of mankind, because I would know, then, that my death, helped that progress, like the doctor who caught the virus his research helped to find a cure for. From a theological point of view: The question points to the central paradox of Christianity, that: "except a corn of wheat fall into the ground and die, it abideth alone; but if it die, it bringeth forth much fruit." (Jn. 12: 24). This is essentially your answer.

For a Christian to die and for a non-Christian to die are not the same thing. In general terms it is the same, but existentially (which is what counts from the Christian point of view) it is not. For a Christian to die and for a non-Christian to die are not the same thing because the Christian is assured of life after death. You need to make this distinction. For the Christian, therefore, "death" does not mean what it means ordinarily, either in common sense or logic. Death means "death to sin", that is, death to all those things in this life which lead the soul to perdition. In this way, for the Christian, to die, is a good thing, it brings virtue and thereby, the everlasting life of the soul. What, therefore, is ordinarily called 'death', for the Christian, is a departure from the bodily state and an entry into the promised life to come. For the Christian, to live for your beliefs, is to die. This is the paradox of which the Cross is the symbol. But "to die" in the foregoing proposition does not refer to an 'event' as a terminus of what is called "life". Life, in the Christian idea is only life if it is loving, that is, self-sacrificing and up-building in virtue; otherwise it is merely a form of death which though it looks like life, is not and leads to everlasting death at the end of it. To say, therefore, that for the Christian to live for her beliefs is to die, means that it is to die to sin and death, but dying to these means living for that to which — or to Whom — your beliefs point, that is, God. If you copy some of this for your essay be sure to cite your source or you may be plagiarising, which they are strict on in schools over here.

Matthew Del Nevo
www.sicetnon.com

back

Damon asked:

Did Spinoza believe in a beginning of the universe, like a big bang, or more of a universe that has no beginning or ending? Did God create the universe, or is God just the whole universe without a creation?

Spinoza believed that the universe was eternal. Since he believed that Time was a part of the universe, the universe itself could have neither beginning nor end.

Spinoza uses the Latin phrase "Deus sive Natura" which means "God or Nature (the Universe)." Spinoza believed that God and Nature were identical. In the sense that God sustains himself, God creates himself, and so, Nature or the Universe. Thus, Spinoza calls God "Causa sui," which means "cause of himself."

Ken Stern

back

Brandon asked:

I recently created an argument that I think might be an ontological disproof of god's existence, but every person I have presented it to has changed the subject, or said that they would think about it and get back to me and never did, and I would really like some critiques of it, especially if any of the premises have been proven to be false. here it is in a series of categoricals (except the second conclusion):

1. Everything that thinks, changes

2. Everything that changes, is destroyed

3. Nothing that is destroyed is eternal

4. Therefore, nothing that thinks is eternal

5. Therefore, the mind of god is (not thinking or not eternal)

This would have applications to all eternal entities if true of course, but I thought about it in relation to god first and so that is how I have been presenting it. Of course it could most easily be solved by appealing to god's immanence, but then how would god decide to make a changing universe to become immanent in? Mainly it would apply to judeo-christian theology anyway.

Thank you for any and all help you can provide me with.

Brandon — I think there are two basic flaws to your argument. 1. The first is that you call this a disproof of God's existence, but that is not in fact what your conclusion relates to. A more precise formulation of the issue you address might be this: "Is it consistent to believe in a god who is both rational and eternal?" If your argument holds water, this would naturally be a problem for orthodox Christianity, but it would not be a disproof of God per se. Although "God" in the Jewish-Christian tradition is a rational and eternal being, it is quite possible to conceive of a non-rational god, and even a non-eternal one. In fact many people do believe in "gods" who are not "rational" in our sense of the word.

2. The second flaw concerns your second statement, "Everything that changes, is destroyed". Here you are attempting to make an absolute statement based on empirical evidence — but this cannot be done. It may be true, in our experience, that everything that changes is destroyed, but our experience is limited. Empirical science deals not in absolute truths, but in hypotheses and probabilities. The very best it can achieve on any subject whatever is a statement to the effect that "judging by the currently available evidence in the currently observable universe, this would appear to be the case". Many of these hypotheses are obviously close enough to the truth for us to live by them: Turning on a light or starting a car assumes faith in a hypothesis about the behaviour of electricity which on the whole turns out to work. But there may be evidence which we have not yet uncovered, there may be parts of the universe that we have not yet observed, and there may be combinations of circumstances, even in the world we know, which have not yet arisen. There may also be aspects of reality which are impossible to observe directly. Emotional pain, say in a divorce, is not measurable or even observable, except by introspection. All we can observe is the behaviour of those affected. Does the pain not exist? Empirical data is therefore by nature incomplete, and this means that it is not possible either to prove or disprove your second statement. Our experience of the universe may have led us to a correct hypothesis, but there is no way of testing this in absolute terms. Since your whole argument rests on the truth of this statement, this means that the argument as it stands can be neither proven nor disproven.

Immanuel Kant came to the same conclusion: The existence and nature of god can neither be proved nor disproved. If God (in the Jewish-Christian conception) exists he is by definition not bound the "laws" of the physical universe, and that means that we cannot prove or disprove his existence by studying the universe. It is therefore a matter of a priori choice, not a matter or reason, whether you believe in such a God or not.

But you are welcome to disagree if you think my argument is flawed!

Paul M Waters
http://www.paul-waters.co.uk


Most of Brandon's argument goes through:

1. All T's are C's
2. All C's are D's
3. All D's are not-E's
4. Therefore, all T's are not-E's.

But he further concludes:

5. Therefore, God is either not-T or not-E.

Unfortunately, 'God' does not appear in any of the premises. From them one can conclude to a denial of God's existence only if one assumes that God is both T and E. But why should one assume that? (Aristotle's divine First Mover is eternal and thinks, but doesn't change and can't be destroyed. [Metaphysics, Book Lambda, 1072-1073.] Was Aristotle confused?) Also, Brandon leaves the reader uncertain as to the meaning of to change (substantially or accidentally?) and destroyed (annihilated or merely decomposed?). So the truth of the premises is uncertain. His argument has the virtue of brevity, but brevity at the cost of clarity is no bargain.

The generic concept of God relevant to philosophical theology (pro and con) was best captured by David Ray Griffin: a personal, purposive being who is supreme in power and perfect in goodness, who created the world and acts providentially in it, who is experienced by human beings and is the ultimate guarantee for the meaningfulness of human life, the ultimate ground of hope for the victory of good over evil, thereby alone worthy of worship. (Religion and Scientific Naturalism, page 90) The relevance of Brandon's syllogism to that concept, and more importantly to the effort of human minds to wrap themselves around the issue of God, is not clear.

Tony Flood


What do you think "god" refers to in these propositions? How can god be a term of a proposition in the first place if the conclusion is that He is unthinkable? If god is "thinkable" what is being thought? If god is "unthinkable" what is not being thought? — god? But in that case you're stating a tautology. In the first place, why not ask yourself if what is being thought is what is customarily meant by "god", and if not, why not? And is it the "god" of the philosophers, i.e an abstraction, like x, that is thinkable? Why cannot this abstraction be G-d — that is, "the God of our fathers, of Abraham, Isaac and Jacob"? What is the difference between god and God and what difference to your propositions does this difference make? Is philosophy about smart games with words, ideograms, or is it a love of wisdom in which a real philosopher — rather than sophistical playing with words — is ever searching out the face of that which he loves, and in which 'knowledge' is relational heartfelt and irreducible to straight information and class — book answers?

Matthew Del Nevo
www.sicetnon.com


I'll have a go at this. I find (1) unobjectionable. I'm not so sure about (2). What about the universe as a whole? Of course, it may be destroyed in the Big Crunch, but I don't think that it must be. (3) is almost tautological. If (2) is not (always) true, then your argument fails.

In any case, I'm not sure that matters for your argument, because I understand that 'eternal', in theological terms, is usually taken to imply being outside of time and hence unchanging. My understanding is that an eternal god is (as your argument shows) necessarily unthinking, unable to intervene in time — as you say, an immanent god. Thus, god cannot be the sort of god that decides to make a changing universe. Rather, he is the Uncaused Cause, or the Unmoved Prime Mover of the universe. Of course, this does not fit with most people's idea of what god is like, but there are many judeo-christian accounts of god's nature, many of them incompatible.

Tim Sprod

back

Arthur asked:

If we photocopy a human (not genetically but atom by atom) the new individual will have the same characters and memories (memories are chemical compounds in the brain) but still he will not have the same consciousness...why?

If we make a copy of a man (by cloning or by another way), the new creature will be a duplicate of the parent creature both in character and in other qualities. So, what is individual? — That is the question you mean. Is there any thing (character, experience, talent, outlook, style...) which can help us to differentiate one person from another. If you suppose that memory is a chemical compound in the brain, we should therefore conclude that our experience and ability to perceive any things and ideas and deny the others are the results of bio-chemical reaction in our brain too. Characters, addictions and talents are the effects of chemical activity of the human brain. Answering you question, I can say: yes, copied man will have the same consciousness, and the same way of thinking, and the same experience. Because all of these phenomena belong to our brain and there is nothing, which cannot be described by bio-chemical laws.

Moreover, love and faith, in this paradigm, are chemical effects too. If we consider that our choice of partner is determined by concrete scientific bio-chemical laws, we should agree that there is no mystery and enigma in love, as well as that there is no superior impulse which galvanizes the poets into rhyme. If you think that character and memories could be copied, so you should agree that predilections and habits could be copied too. That is why copied man will love the same things (and prefer the same partners) and believe the same gods, as the original man.

I understand that this answer cannot satisfy many inquiring minds. It is very heavy to agree that someone else can feel the same emotions, see the world by the same eyes, and write the poems by the same words, and it is impossible to agree that we are animals with identical minds, senses and souls. If so, this is the question about soul. If you believe that soul exists, you should agree that nobody could copy the soul.

Christians, Jews and Muslims believe that there is only one soul, therefore we cannot make a copy of soul. That is why the Pope declares that cloning contradicts to the ideas of ethics and religion. To believe that my soul is only mine and nobody can see my own world, nobody can feel the same emotions and nobody can write the same poem seems to be the more optimistic position. But it is the question of beliefs.

If we consider that all of us have our own (inner) worlds, we should agree that there could not be accordance between people. If we have different world outlooks, we never learn and never understand each other, and we doomed on permanent conflicts. If we have different (and endless) inner worlds, we can never understand even our beloved and very familiar friends and we can never reach accordance and consensus. We never even learn about or understand ourselves. I we consider that if all of us have endless inner worlds, we should agree that we can not describe and cognize it as well as we can not control and explain our behavior.

On the other hand, it could be argued that this independence of our soul from the power of mind is the basis of our permanent interest in communication, to meet and to love. If one see in his partner the permanent secret and permanent seduction, they can live together for a many years. Because if we can not cognize the other, we have an interest to the other, because s/he keeps to be dangle (and danger) unknown universe. We can never learn the soul of other man, but, according to Levinas, in dialogue we can see the face of the Other. We love (wo)man, because we do not know her(his) soul.

Dmitry Olshansky
philosophos.org postcard 9


What do you mean by "same"? For a few moments, at least, before the two people's experiences diverge, they will have the "same" consciousness, in the sense that their memories, feelings, sensations (if they're both looking at the same object), and so forth will be identical to each other; and they will also have the "same" body, in just that sense. As their experiences diverge, of course, they will have divergent sensations, feelings, and memories... so they will not have the same consciousnesses, in that sense of "same", after a short time interval... nor the same body. If you mean, "will there be one consciousness sharing two bodies?", i.e., one self, one "center", one unified field of consciousness spread between two bodies, why should that happen, and how could it possibly happen, since their brains are not neurally united? Since you take the position (which I agree with) that consciousness is materially generated, then it follows that two separate bodies (brains) will generate two separate consciousnesses. If those brains are identical, then the contents of those consciousnesses, i.e., their potentials, their dynamics, their biases, etc., will be identical. But that's just saying that there will exist two identical neural (and bodily, to generalize) dynamics... for a while, at least. So answering this question depends on what you mean by "same".

Steven Ravett Brown

back

Chandru asked:

I am very happy to meet you.

My mind is full of unsolved matters which I pursue with eagerness. The complete I-ness is residing in the concepts like Nature, soul, god, meaningful-meaningless states, life, death, purpose of life. If the purpose here is an unsolved one then how best we can lead our life until we arrive the truth or if we are not fond of purpose of life, what next, where are the current philosophy standing about this context. I request you kindly to provide all available information throughout the search around the existing world.

These are big questions. A note at the beginning: One interesting thing about philosophy is that the really big questions have not been definitively and decisively answered — though many philosophers have suggested answers. It is also an open question whether these questions can be answered at all. Hence the 'current' philosophical thinking is not necessarily 'better' regarding many of these issues than that of the most ancient philosophers. (This is an important difference to natural sciences).

You mention 'purpose of life'. The term 'purpose' in this context is somewhat ambiguous. It could indicate a choice you yourself make for your personal life, or it could mean that whether you agree with it or not, your life has a purpose, which is not chosen by you, but comes from elsewhere. I will now call the first kind 'personal purpose' and the second kind 'absolute purpose'. Someone who thought that there is no God and therefore no 'absolute purpose' to life, was Sartre. He thought however, that we can live a moral life by personal choice i.e. give our life a 'personal purpose'. Someone who thought that there is an 'absolute purpose' was Aristotle. He argued that every rational activity aims at something. (Example: I go to the doctor to get healthy, I want to be healthy to feel good. I go to work to earn a living. I want to earn a living in order to buy things I need. I need things in order to ... you get the drift.) To avoid infinite regress all these activities ultimately must aim at one goal — the highest 'good'. This good is happiness, because happiness is the only thing we desire for itself, and not in order to get something else. To better understand what the good for man is Aristotle argues from function: As the foot has a function to support the body and the sculptor has a function to produce statues, so man has a function. This function can most clearly be understood in the capacity unique to man and distinguishing him from other beings — the capacity for reason/rational thought. So the good for man is to perform this function and to perform it well i.e. in accordance with virtue. So the best life — how to live and do well — is the happy life, meaning a life of virtue and contemplation (exercising man's function). What do you think — is there an 'absolute purpose' to life? And if that was true, would you be obliged to go along with it?

Truth — that's another one. Is there such a thing as 'absolute truth' and if there was, could we know it? Plato for example seems to have believed in the existence of absolute truths (even in their reality, not just abstractions). However there is a lot of evidence to suggest that he did not believe such truths to be attainable in this life, but believed that the soul was immortal and contemplated the truths before and after our earthly life. Aristotle disagreed with this theory (and gave some very convincing reasons for this disagreement). Sceptics have questioned the possibility of knowledge of absolute truths: One argument goes like this: All our beliefs are based on input via the senses, sense perception. Now sometimes such beliefs turn out to be mistaken. Example: A stick partly immersed in water looks bent — so I assume it is bent and find when taking the stick out that this was just an optical illusion. Now if such mistakes sometimes happen, then I can never be sure that at this particular moment I am not under an illusion. (Descartes famously suggested the following scenarios: What if I am dreaming? What if an evil demon is tricking me all the time, creating delusions for me?). Therefore I can never claim to have knowledge in the sense of absolute truths based on sense perception. I cannot even be sure of my body, that others exist, that there is a world outside my own mind etc. Do you think there is such a thing as absolute truth? Or is everything a dream or an illusion? Are there only personal, that is subjective, truths?

Some other big questions surround the mind/body problem (are they the same or of the same type, how do they interact, is there an immortal soul etc.etc.), personal identity (is there a self, and if so, how to define it, what constitutes personal identity over time), moral values (are there any absolute moral laws, are there 'rights') etc.

Sound like you are embarking on a fascinating trip through the whole of philosophy — have fun!

Helene Dumitriu

back

Bill asked:

What is the "modal argument" in relation to dualism?

There are two meanings to the word "modal", one being that of mode or attribute and the other to do with the modalities of possibility and necessity.

Descartes' used the former, but to make a successful claim for a real or ontological distinction, he really needs the latter but this was not available to him since he didn't successfully establish substantial difference. Descartes found that he had a clear idea of himself as a thinking thing, and having established that clear and distinct ideas are grounds for knowledge, he also found he had the clear idea of body as essentially extended and so he concluded that his was essence was a non-extended thinking thing. The argument makes use of the assumption that attributes or modes distinguish substances. However, it is not impossible for one substance to have two irreducible and conceptually distinct attributes. Gilbert Ryle's category distinction, for instance, shows that a university can be an institution and a campus, the campus being extended while the institution is not. Peter Strawson has argued that a single substance can have physical and mental attributes which receive different predicates without any implication of substantial dualism.

In the other sense of the modal, it might be argued that if two things are identical, one could never exist without the other or, conversely, if two things are non-identical there are no circumstances under which they could be the same thing. In a possible world thought experiment, it is certainly conceivable that I could have a different body than the one I in fact have, but it wouldn't follow from this that I could exist without any body and whichever body I do have, I could be identical to it and not exist without it. However, it is logically for a person to survive the destruction of his body, and conceivable that there are soul-substances. Yet, on the hand, the thought experiment lacks clarity. So again, the dualist would need to look at essence, and argue that whichever body I have, the body is not me, but I have a subjective essence, or personal identity which is mental and that makes me "me".

Rachel Browne

back

Christine asked:

What does Heidegger mean when he says, "What we call a 'feeling' is neither a transitory epiphenomenon of our thinking and willing behavior nor simply an impulse that provokes such behavior nor merely a present condition we have to put up with somehow or another"? what exactly is feeling then, according to Heidegger?

Heidegger refuses to think the person — and personal feelings included — in terms of ego psychology. He is non-Cartesian. He distinguishes (in the language of Being and Time) between the 'ontical' depiction of entities (e.g. feelings) in the world and the 'ontological' Interpretation of their Being. Heidegger does not deny that feelings exist ontically, but that is not what they are. What feeling (Gefhl) is is a matter of the question of Being, which means, therefore, an existential question, that can only be approached (but never captured) by what he calls fundamental ontology, which is basically a radical restatement of phenomenology. For Heidegger, feeling qua Being is a disclosure of existence. It is disclosure of a particular kind, one that discloses mood (Stimmung). "Mood is a primordial kind of being for Dasein" writes Heidegger (H.136 in the MacQuarrie Trans.). 'Moods' are the precondition of feelings. "A mood assails us. It comes neither from 'outside' nor from 'inside' but arises out of Being-in-the-world, as a way of such Being." (ibid.). What that means is that 'mood' is a 'bare' way of being for man. Our 'mood' is constitutive of our selves and our world, it structures them. Mood discloses 'state of mind' (Befindlichkeit) as the way we are as In-der-Welt-seins (Being-in-the-world). These are the grounds of feeling. Heidegger basically has to rewrite the language in an attempt to talk about human being in a way which questions Being rather than presupposes it in these or those (Aristotelean) terms.

Matthew Del Nevo
www.sicetnon.com

back

Sandip asked:

What is the precise definition of:

Spirit
Ego
Mind
Brain Process
Soul
Consciousness?

Do we really need so many words?
What do you philosophers think?

We do need these words since they all signify different aspects of man, although the spirit and soul mean the same thing, both have religions connotations. Spirit and soul are the part of man which is supposed by some to survive the body's destruction. This is logically possible for the mind, but in this sense the proposition that the mind might survive the body is something to be considered when looking at the nature of mind and it's relation to body, whereas the proposition that spirit or soul survives the body is connected to belief.

The ego is 'I', the self, and could be identified with the mind, the mind and body, history, survival etc. There is a lot of philosophy about the self, some of which you can find on this site.

The ego is also a psychoanalytical term introduced by Freud to describe a particular conscious agency in the mind, in contrast to drives (the id) and the moral agency (superego). The ego is that part of the mind which represents reality, but this can't be equated with man's rational faculty, since there is nothing about the superego, and the ego also has the function of controlling and repressing unacceptable drives. The function of control and repression can lead to the setting up of defence mechanisms which prevent a person from pursuing goals successfully, forming stable relationships, and can lead to forms of neurosis which are treated by ego-psychologists. The term is a theoretical one and it is possible to deny the truth of psychoanalytical theory and also the existence of the ego.

Everyone believes that man has a brain and it is arguable whether this should be identified with the mind, since the mind is essentially conscious whereas the brain is a physical thing. Again, there is a lot of philosophical discussion about this and some of it can be found on this site.

Brain processes underlie conscious states, or the mind. Discussion here centres around the question whether a particular brain process determines a particular conscious or mental state. The nature of brain process is also the subject matter of neuroscience.

I haven't provided any precise definitions, because as far as I know there aren't any, which is why we manage to do philosophy.

Rachel Browne


I'm sorry having to tell you, that there is no precise definition of any of your mentioned terms. They all refer to something beyond physical existence and are subject of a branch of philosophy, the philosophy of mind. It includes philosophical psychology, philosophy of psychology and the area of metaphysics concerned with the nature of mental phenomena and how they fit into the causal structure of reality. As we cannot point at an entity "spirit", "soul" and so on, we have to describe them and we need different words to describe the different aspects of mental phenomena. When philosophers use one of these terms, they have to explain in what sense they use it. I'll give you some examples below.

The original idea of a spirit is of a disembodied agent, as an immaterial soul or a non-material intelligent power. In the seventeenth century and earlier there was a belief in spirits as gaslike substances intermediate between matter and mind. When we talk now of the spiritual we refer to neither of these but typically to the kind of emotion one might have towards God or some other factor beyond one's material life.

Ego is what 'I' stands for, the subject's essence. Plato and Descartes thought a person could exist disembodied. Locke imagined that a prince could swap bodies with a cobbler. It is hard to see how these stories could be intelligible without conceding the existence of an incorporeal ego, a subject for thinking, feeling, and willing, which makes each person who they are. In psychoanalysis, ego is the part of the mind that is closely in touch with the demands of external reality and operates rationally. It includes some motives (such as hunger and ambition), the individual's learned responses, and his (or her) conscious thought. It has to reconcile the conflicting demands of the id, the superego, and the outside world. The human soul is that which gives life to the human being. For Aristotle, the soul was simply the form of the body, which means the way our bodies behave, and cannot exist separately from it; plants and animals also had souls of their own kinds. For Plato, most Christian theologians of the first millennium AD, Descartes, and many others, the soul was the essential immaterial part of a human, temporarily united with its body. Aquinas also held this, while emphasizing that union with a body was the natural state for a soul. Most modern philosophers deny the existence of an immaterial soul. One strong argument for the existence of such a soul given in essence by Descartes is this: I am now conscious. But it is logically possible that my body should suddenly be destroyed and yet I continue to be conscious and so to exist. But a thing such as I am can only continue to exist if some part of the thing continues to exist. So I must now already have an essential non-bodily part, a "soul", if my continued existence is to be logically possible.

Most of us will say, that consciousness exists, but it resists definition. There are some criteria for saying of some organism or state that it is conscious. Consciousness involves experience or awareness. Human mental life undoubtedly has a phenomenal side, a subjective side that even the most sophisticated information-processing systems lack.

Simone Klein
http://www.sophiasworld.at/

back

Jackie asked:

If asked the following question: How do we elect, concerning values in America? would you consider this a question that should be answered in terms of the moral depravity of America i.e. Clinton's affair with Lewinski, or rather a question as to how we make our choices in life. I am currently studying philosophical ethics and am unsure how to answer this question.

I am not sure just what you are asking. Are you asking on what basis we do elect our leaders? Or on what basis we ought to elect our leaders? And is the choice between their personal behavior, on the one hand, and what you call "how we make our choices in life?"

I must begin by saying that calling the affair between Clinton and Lewinski "depraved" seems to me rather overwrought. Unless you have some special animus against that sex act (called "fellatio") itself. it is something that is done by millions of people every day without the sky having fallen down.

But to get to your question: It seems to me that we have to consider the personal behavior of those we are thinking of electing and , also, the values on which we make our own choices in life. When we consider the personal behavior of candidates running for office, we ought to consider how much we actually know about how they behave (which, except in a few headline instances, is not very much) and especially, to what extent their personal behavior is likely to affect and spill over to their public behavior where it counts so far as we are concerned. Now, so far as our own values are concerned, we have to think clearly about how the candidate is apt to further the aims and goals we would like our country or out county or our city to pursue. If, for instance, we think strongly that the minimum wage should be raised (or lowered) we should inquire closely into what the prospective candidates say about that matter.

We should vote intelligently. You already know that. But it never hurts to repeat it.

Ken Stern

back

Dawn asked:

What is Christianity? I am doing my own research because I don't want to be pushed to believe something. I just want to know what it is. I will really be thankful if you can help.

Christianity is a reform of Jewish religion — I do not say Judaism, because Judaism did not exist as such at the time of the reform. Jewish religion is historical and ethical. For Jewish religion history is one great tracing of God. God is the Origin of all that is. Ancient Jewish historiography (of the Old Testament) interprets the way things happen, both nationally, internationally and personally to the individual, in terms of God. And Jewish religion is born and raised on the experience that God is living and, as such, revealed Himself, first and foremost, to Abraham, to Moses and the prophets. To Moses he revealed a Law (the 10 Commandments). Jewish ethics is not therefore grounded in reason, rather, reason is grounded in ethics and ethics in historicity, or rather, as historicity.

Jesus revealed something about the nature of this Law and about God (a 'new' testament to Him), although retrospectively it seemed that the prophets had always been trying to work towards it, and some of the later prophets (men of extraordinary moral insight) were explicit about it, but to no avail in the life of their people. Christians believed that Jesus himself was a revelation of God. There had been some general expectation among Jewish groups of such a revelation. Followers believed Jesus was the 'Son of God' because of the spiritual authority of his life and teaching. What Jesus revealed about God is that forgiveness is the essence of the Law (of Moses), which meant sacrifice for others, and never at the cost of anything less than life. He revealed that 'God' is related to Man intimately, familiarly, and is not exterior, not jealous, not like a judge in court; His Law is not that kind of Law, but rather God is "with us". Jesus revealed that there is life with God after death for those who seek it in this world. Jesus revealed that the Law is for our freedom — our life — it should not be a burden or a restriction (which it seemed like to many Jews under the religious authorities). Instead of looking for 'loopholes' in God's law (like the Pharisees) and trying to tie them tie them up, the Law should be an inner law which is an expression of our love. Jesus naturalised and internalised the Jewish Law. He taught that the law was intended to transform the world into a place of peace and the brotherhood of all people, not just Jews, not just men, not just 'good' people, not just 'religious' people.

Matthew Del Nevo
www.sicetnon.com

back

Damon asked:

I just read the book Existentialism by John MacQuarrie. In the chapter on thinking it talked about using different types of logic. It primarily said breaking away from the tyranny of the traditional logics which I am assuming are deductive and inductive. My question is what other types of logic to existentialist adhere to?

It is not enough for philosophical knowledge to be expressed in general statements or abstract syllogisms or logical propositions as far as existentialists are concerned. In the Kierkegaard chapter of Sophie's World it is put very well. Sophie is reading:

"Eight plus four is twelve. We can be absolutely certain of this. That's an example of the sort of 'reasoned truth' that every philosopher since Descartes had talked about. But do we include it our daily prayers? Is it something we will lie pondering over when we are dying? Not at all. Truths like those can be both 'objective' and 'general' , but they are nevertheless totally immaterial to each man's existence."

What is worth praying about, or the conditions that lead us, sometimes against our will and 'better nature' to pray, or what keeps us awake at night, or what we will ponder on our death bed, these are the substance of existential logic. Furthermore, knowledge of the individual human existence before God, or before the Absolute, or before nothing depends most on deeds, not words, and on authoritative sources for the question of 'how to live?' The logic of the existentialist therefore has to do with the consequences and responsibilities, the necessities and contingencies, that follow.

Matthew Del Nevo
www.sicetnon.com

back

Stephanie asked:

I'm doing my dissertation on a comparison between art (particularly that of MC Escher) and philosophy. However, I am struggling to find valuable resources. Do you know of any texts which you feel would be important in this subject?

Escher is one of the most popular choices of artist for the covers of philosophy books, but his appearances between them are much less frequent. The principal exception is a book you must have already seen: Douglas R. Hofstadter 1979 Godel, Escher, Bach: An Eternal Golden Braid (New York: Basic Books/ London: Harvester). This is a vast and extraordinary book, and there's a lot of stuff in it which is outside the scope of your dissertation, but it is well worth a careful browse.

You might also want to take a look at: Patrick Hughes and George Brecht 1975 Vicious Circles and Infinity: An Anthology of Paradoxes (New York: Doubleday/ London: Jonathan Cape). There are more philosophically rewarding treatments of the paradoxes of self-reference, but this one was compiled by two artists, and should prove worth the effort it may take you to track it down (it's out of print). It contains a lot of carefully chosen material that can be hard to find elsewhere, including a discussion of visual and artistic paradox.

Michel Foucault's essay on Magritte, Michel Foucault 1983 This is not a pipe, translated by James Harkness (Berkeley, CA: University of California Press). could be a helpful point of comparison, but you would really have to read a lot more Foucault to appreciate it fully.

Finally, I did a search on Philosopher's Index for 'Escher'. The most plausible looking item retrieved was: Jane Duran 1993 'Escher and Parmigianino: A Study in Paradox' British Journal of Aesthetics 33(3): 239-245.

Andrew Aberdein

back

Joe asked:

Is there such a thing as a philosophy of media, or a philosophy of film which concentrates on human emotions and the need for the viewer to see these emotions? I am very interested in this topic and would be very grateful for any advice on books or internet sites in this area.

There are a number of approaches worth looking at.

'Reception Studies' is an area of film studies. It looks at the audience's (primarily emotional) reaction to film. There is an excellent and incredibly comprehensive bibliography — albeit slightly out of date — produced by Austin. (Bruce A. Austin, The Film Audience: An International Bibliography of Research (Metuchen, N.J.: Scarecrow Press, 1983.) You can wade through this extensive bibliography covering just about every aspect of the audience reaction to films. He has also written more recent articles. Also look out for Janet Staiger who has written extensively e.g. Janet Staiger, 'Film, Reception and Cultural Studies', Centennial Review vol. 36 no. 1, 1992, pp. 89-104. See also a series of books by Stokes and Maltby (Eds) published by the British Film Institute about who formed early film audiences and the emotional appeal of 'the movies'.

The subject inevitably spills over into the 'do violent films cause violence' debate (Martin Barker and Julian Petley (eds.), Ill Effects: The Media/Violence Debate (London: Routledge, 1997)) and the effect of slasher/horror films (Ron Tamborini and James Stiff, 'Predictors of Horror Film Attendance and Appeal: An Analysis of the Audience for Frightening Films', Communication Research vol. 14 no. 4, August 1987, pp. 415-436).

You could also look at Film Philosophy — particularly the excellent and very readable book of that name by Ian Jarvie. Jarvie is a sociologist by training and 'comes at' philosophy of film in a psychologically inflected way. There is also a good section on emotional response in Allen, Richard and Murray Smith Film Theory and Philosophy (Clarendon Press, 1997). See in particular the articles by Carl Plantinga 'Notes on Spectator Emotion' and by Murray Smith 'Seeing from the Inside'.

Film Studies itself has touched on this area. See the very influential compilation by David Bordwell Post Theory which has loads of stuff on e.g. empathy and film, role of film music in impacting emotions. Noel Carroll (was a film prof, then moved into philosophy then into pure aesthetics) is worth reading. See his Theorising the Moving Image (Cambridge University press, 1996) which includes his weird theory of what emotional factors are triggered by film suspense.

If you still an unfinished urges then try out film psychology as a subject where there are a number of complex studies e.g. one by Zizek based on Freudian/ Lacanian based analysis of the psychology of the cinema ('Everything you always wanted to know about Lacan'.)

You should enjoy the often contradictory analyses contained in these different approaches. You might even conclude that no-one really knows how/ why some films work at creating emotional and others don't. If you find the answer then a bright future awaits you in Hollywood!

Andrew Browne

back

Demo asked:

Who is a philosopher?


and Shaista asked:

What is a philosopher? (I need a brief definition)
Why are we not philosophers?
Why do we study philosophy?
What is philosophy?

Perhaps the real question should be who is a philosopher, an easier answer — everyone! I suppose a philosopher is one who seeks to understand life, its meaning, its orientation, its end and the processes that that we engage in our attempt to navigate our way through life. The philosopher seeks to understand where we have come from, how we got here, where we will end up. The great philosophers, Plato, Aristotle, Socrates, Augustine, Aquinas, Descartes, Kant etc., searched in their own way to explain what makes us 'tick' as human persons, whether it was in the nous-sphere (the sphere of thought) the moral sphere, the human sphere. All of them offered their insights and perceptions into these great human issues. Ultimately, some philosophers search after meaning, and how to make that meaning applicable, others would say there is no meaning, others would say we cannot know anything, others would say it is possible for us to know everything. A philosopher is anyone who seeks to reflect on life and its meaning and the consequences of that meaning for the individual, the corporate body, society, the world.

Fr. Seamus Mulholland OFM

back

Edward asked:

If god is dead, then everything is permitted. What is Sartre's response to this?

For Sartre, if God is dead, it doesn't follow that everything is permitted, although it follows, for Sartre, that there are no objective moral commands or values. In Emotion and Human Existence, where he quotes Dostoevsky, Sartre claims that man chooses values, creates and imposes them as an ideal goal transcending the material world as a result of the forlornness of his existence. Man, in his freedom defines himself, and in his desire for a God who doesn't exist, he defines himself as a being whose reality includes values. In Being and Nothingness, man is described in abstract as, at first a desiring consciousness suffering lack or privation which gives rise to an upsurge towards being and beyond being to value. In emerging into human reality we create value. If we create values as part of human reality, it cannot be that anything is permissible.

In our freedom to choose, it would seem that we can choose any action. However, in Emotion and Human Existence, it is claimed that we are not just responsible for ourselves but for all men, and also that we can never choose evil and although this claim doesn't seem to be directly substantiated in this text, Sartre can hold that to choose evil as a goal is to limit possibilities for action and so would be to live in bad faith. For any particular evil act, it is suggested in Being and Nothingness that we cannot choose to do something we regard as evil as this is to be both for and against it.

There is also the moral prohibition which arises by virtue of the fact that we exist as one amongst others, in human reality, and is exampled in Being and Nothingness as the moral awareness of the "look" of the other which can bring us to shame. We are conscious of the moral judgements. Also, we are conscious of the consciousness of others, which is another reason that we also cannot do what we like. We don't just perceive that which is at a spatial distance, but have awareness of the other without this distance, and as such morality is transcendent, but close, and we cannot ignore suffering of others.

Rachel Browne

back

David asked:

I need help with a question I am struggling to answer.

A boy falls off a dock twenty feet down and he is nearly about to drown. A man see's this going on and jumps in with every intention to save the boy, but accidentally falls on the boy and kills him. The question is how would Kant, Mill, and Aristotle react to this situation? Think in terms that — Kant refers to one's intentions, Mill refers to one's conscience, and Aristotle refers to one's appropriate virtue. Also which of the philosophers is right/wrong? and Why?

To begin with, I don't think that Kant thinks that intentions are what make an action moral. Intentions what are thought to be the probable consequences of an action, and Kant does not think that the probable consequences of an action are morally relevant. Kant talks about the motive of the action, and, in particular "The Good Will." The motive of the action must be the performance of your duty for the sake of duty, and not for any consequences that may ensue. If it was the man's duty to save the boy, then if the man did it for that motive, then the consequences do not matter, and the man performed morally.

And I don't believe that Mill thought that acting from conscience was a factor in deciding whether a certain action is moral. For Mill, unlike Kant, it is the intention to perform an action which conforms to the Principle of Utility, that makes an action moral. If the intention was to perform an action whose consequences were likely to cause "the greatest good for the greatest number of those affected including the agent" then the action was a right action. It might be argued that the would be rescuer action could be judged in this way. The fact that a tragic accident occurred would not matter.

As for Aristotle, Book Three of the Nichomachean Ethics seems to me most pertinent to your question. Here, Aristotle talks of voluntary, involuntary, and non-voluntary action. He also states that intentions are crucial for virtue: that is, the intended consequences of the action. Aristotle presents two classes of action in which a man would not be held responsible for the consequences of his action: compulsion and ignorance. A man who is forced to do something is not to blame for the consequences of his action; and neither is a man who does something in ignorance of the consequences. Accident would, I think, fall under the general category of ignorance. (If the man, for instance, stumbled on a small stone he could not reasonably be expected to avoid under the circumstances.)

So all three philosophers would have held that the man (who had the best of intentions) was not to blame for what happened. But Kant would not have allowed the action was a moral one unless it was done from the sake of duty. The Mill and Aristotle would, I think have given moral accolades to the man, while, of course, excusing him.

Ken Stern

back

Fabio asked:

Do you know Norman Malcolm? I need information about this philosopher. Thanks!

Norman Malcolm was born in 1911 and died in 1990. He worked at Cornell University in New York State for several decades before retiring in 1978, after which he moved to King's College London. He was one of the master pupils of Wittgenstein and perhaps the chief transmitter of his ideas to the United States in the 1940s and 1950s. His main work was in philosophy of mind and in the theory of knowledge, where he opposed Cartesian and scientistic views by means of Wittgensteinian conceptual analysis. What is perhaps his best-known book, Dreaming (1959), is a critique of Descartes's argument from illusion, arguing that dreams cannot be coherently be viewed as mental impressions occurring in sleep (what they actually are, he didn't claim to be able to say). As an interpreter of Wittgenstein, Malcolm advanced the currently widely disputed view that Wittgenstein's early work consists of straightforward metaphysical theses in the classic tradition of Western philosophy which his later work is largely devoted to attacking, instead of already being an early version of such an attack on the tradition.

T. P. Uschanov
Research Assistant
Department of Philosophy
University of Helsinki

back

Alan asked:

In your opinion has the post-modern trinity of Lacan/ Foucault/ Derrida been successful in their effort to destroy philosophy? What is your opinion about other post-modern thinkers such as Vattimo and Levinas and predecessors such as Baudrillard and Bataille? Finally I'm very interested in your opinion about Cioran and how you would classify him in relation to postmodernism.

First of all, nobody from this trinity tries to destroy philosophy. All of them proposed very different ways to put the question about metaphysics.

Lacan was a psychoanalyst and he did not deal with philosophy. Of course, he influenced philosophy also, because he was serious intellectual of his time. But we can find the marks of Lacan as both psychoanalyst and philosopher in philosophy, and in linguistics, and in semiotics, and in sociology and in many other spheres of humanities. So, I do not think that he had a down on philosophy. In my mind he was very serious psychoanalyst, first of all.

Of course, reading his texts we can find the image of culture as well as education as very destructive and aggressive forces. For example, reading his "Mirror Stage" or "The Four Fundamental Concepts of Psychoanalysis", we learn that culture and language brings us alienation. In his opinion, alienation is the basis point of human identification. In alienation the child receives the first experience of separation, which becomes the crucial operation of signification. [Lacan, Jaqcues The Four Fundamental Concepts of Psychoanalysis Trans. Alan Sheridan. New York: Norton, 1981 p.214].

We can find many examples in Lacan to prove that he describes culture, language and education as the form of alienation, violence and separation. In education, the child receives an external system, which is intruded by the (m)other. So, culture and language is the product of other, an external and aggressive power. To become a man, the child should decline his (her) own inner world and should follow the (m)other's edification and receive the external rules of the game: language and culture. "The alienation is not the specular alienation of the mirror stage but the alienation essential to signification and the subject's relation to language. As language becomes paramount, the alienation inherent in language also becomes paramount." [Oliver, Kelly. Witnessing. Beyond recognition Minneapolis and London: University of Minnesota Press 2001 p.188]. Language, in Lacan, is alienation and the hidden violence of culture. So, I should agree that Lacan uses the rhetoric of alienation, but we can not conclude that he ties to show all human culture as violence and evil.

In my mind, Foucault was not a postmodernist, because he died in 1984 when postmodernism had just begun. He was rather a structuralist, who tries to conceive in human history, consistent patterns of its development and the phenomena of culture: clinics, prison, and writing. He accomplished a shift in historical sciences. Before him, history was the history of main events: wars, revolutions, reforms, as well as the biographies of great persons. There were many gaps, which were not considered important: stagnation and anticlimax. Before Foucault the historian supposed that the gaps are not the matter of history, but Foucault proved that in stagnation we can find the appearance of new ideology, he shows that history really is not the history of great events, but the history of development of human's ideology. And the key periods of appearance of new forms of ideology are the gaps. In my mind, he proposed a new dimension of history, a new historical outline.

Derrida proposed the deconstruction of metaphysics, but it is not the same as destruction. Of course, all of these thinkers try to shift the subject matter and make philosophy into something new. So, that is why Derrida as well as Heidegger tries to make a bridge across metaphysics. And Derrida reminds us that his own efforts are related to Heidegger's ideology: "Among other things I wished to translate and adapt to my own ends the Heidggerian word Destruktion or Abbau. Each signified in this context an operation bearing on the structure or traditional architecture of the fundamental concepts of ontology or of Western metaphysics. But in French 'destruction' too obviously implied an annihilation or a negative reduction much closer perhaps to Nietzschean 'demolition' than to the Heideggerian interpretation or to the type of reading that I proposed." [Derrida, Jacques 'Letter to a Japanese Friend.' Derrida and Difference ed. Wood & Bernasconi, Warwick: Parousia Press 1985 p.1-2]. So, the aim of deconstruction is not to destroy philosophy and deny classical metaphysics, but it is a form of criticism and of renovation of philosophy. That is why, in my mind, Derrida proposed a new form of criticism and tried to continue the Heidegger's tradition.

These are very brief ideas on the role and efforts of Lacan, Foucault and Derrida in contemporary philosophy. My own ideas on the role of postmodern philosophy you can learn from my article from Philosophy Pathways Issue 27 which you can also find on the PhiloSophos web site here:

Dmitry Olshansky Foundations of Non-Classical Thinking.

Dmitry Olshansky
Urals State University
Yekaterinburg City
Russia

back

Shen asked:

How might a philosopher help one to be a better general?

A philosopher would say this is in part this is a factual question relating to success so one approach would be to study the methods of a great general such as Napoleon. Tolstoy did this and from his novel War and Peace I remember, for instance, that Napoleon was great because he was a strategist rather than a tactician. He took a long term view of how things would develop, rather than waiting to react to actual occurrences.

It also might be an ethical question, and the paper "Ethics and Public Office" in Mortal Questions by Thomas Nagel might give you some guidance as to the ethical issue involved. You might also consider what makes a good general in the wider context of when? In the past, when the general was on the battlefield, now when there is less need for soldiers, or in the future when fighting at ground level or in the air no longer takes place. You might look at Hannah Arendt's book On Violence to help you consider the nature of force, authority and violence.

A philosopher would hope that a general would consider all of these matters with care.

Rachel Browne

back

Emily asked:

"What I tell you three times is true" — what are arguments in favor and against this? Is it not true that if an individual hears something enough he takes it to be true?

It seems to me that you are mixing up three different questions here concerning what we can call the "Bellman's maxim" (from Carroll's "The Hunting of the Snark.")

One question is whether it follows from the fact that someone tells you something three times, or one time, or 30 times, that what he tells you is true. The answer to that is clearly, no. The truth of a statement does not in any way depend on anyone's stating it is true no matter how many times. A statement is true if and only if it states how the world is, or what the facts are. The facts are what make a statement true.

A different question is what might be called an appeal to authority. Could the fact that someone tells you something be some evidence of its truth in the sense that you would have reason to believe what that person says? The answer is clearly yes, but depending on who the person is, and on what he says. If a person (say a scientist) tells you something about what he is supposed to know something about (this has to do with his credentials on the subject matter he is pronouncing on) then you have good reason to believe that what that scientist says is true (supposing, of course, you have no reason to think he is lying or joking etc.) So if the great physicist Albert Einstein had told me (even once) Earth describes an elliptical orbit around the sun, I would believe that what he said was true. But if an untutored person stated to me (even thirty times) some abstruse chemical statement, I would not believe it. And I would be right not to believe it on the untutored person's word even if it turned out to be true.

Another question you ask is whether it is true that if someone hears something enough, he will think it true. Yes, very often, especially if the person is not very critical, he will believe something true if he hears it enough times. The Nazi Josef Goebbels used to talk about what he called "The Big Lie" and maintained that no matter how preposterous a statement is; if you broadcast it loudly and often enough, people will come to believe it. If that is so, and it may be, it is a sad psychological commentary on the credulous nature of human beings.

Ken Stern

back

Adam asked:

I know this is a general statement, but what is the point of philosophy?

"To think better in order to live better," as the French philosopher Andre Comte-Sponville puts it in his philosophical dictionary. (That's what I believe the point of philosophy should be, anyway. One can also use, and many people do use, philosophy as a tool to allow one's vices to flourish more freely and in subtler forms. But fortunately philosophy is simultaneously a tool to counter such uses of itself.)

T. P. Uschanov
Research Assistant
Department of Philosophy
University of Helsinki


Generally, the "point" of philosophy is the achievement of a coherent and experientially adequate understanding of a broad range of fundamental matters such as knowledge, existence, and value. The impetus that moved you to ask your question is what moves philosophy. It is the pure desire to know the truth. This desire competes daily with other desires, but left to itself it would not rest until satisfied by knowledge of everything about everything. The pure desire to know distinguishes you as a human questor, which you exemplified by asking your question. Whether there is a "point" to fulfilling one's nature is a question for another day.

One superficial difference between a simple question that anyone may ask (e.g., "What is knowledge?," "Is there a God?," or "What ought I do with my life?") and a philosophical question is endurance: the philosopher pursues simple questions long past the point at which the non-philosopher loses interest or gives up. But more than intellectual stamina is involved. The philosopher's interest encompasses every area of experience of which he or she is aware and demands that provisional answers hang together logically. If they are lacking either in experiential adequacy or mutual coherence, the philosopher must modify or abandon at least one of them.

Tony Flood

back

Shelly asked:

How is justice related to equality and how is equality related to the distinctive identities and other circumstances of individuals such as age, race, and disability?

One aspect of justice is fair or equal treatment of human beings. People who call for equal (political) treatment (practical maxim of equality) of human beings normally hold that all human beings, just because they are human beings have the right to equal treatment in certain areas like the right to vote, equal treatment in court but also equal opportunities e.g. regarding education and jobs, and equal distribution of necessary goods e.g. medical treatment.

As you can already see from this sentence there are a number of notions linked together (which is why justice/equality is such an interesting topic) — in the following I will just try and give you some ideas you may want to explore ...

The notion of equality of human beings i.e. the factual statement that human beings are equal (as the basis of the request that human beings should be treated equal). The problem with that, as Bernard Williams has pointed out in 'The Idea of Equality', in: Problems of Self (1973), is that if you take it literally it is too strong i.e. wrong because there are numerous counterexamples where human beings are clearly not equal, e.g. our genetic make-up differs, we differ in talents, upbringing, social circumstances, physical strength and health etc. On the other hand if you interpret the statement in the weak sense it is too weak, because it is trivial to say that the only thing which is equal is the fact that we are all human beings. Williams suggests that between these two extremes the factual statement could be supported by the following considerations:

1. All human beings feel pain. Any society that discriminates certain groups using a criterion like colour of the skin does so either arbitrarily (because the criterion is irrelevant) or simply acts wrongly i.e. disregarding the capacity of these human beings for feeling pain. In fact according to Williams the latter is the case, demonstrated by the fact that people/societies who act like that normally rationalise the discrimination additionally i.e. they do not say that colour of skin is sufficient for different treatment but they attribute some character deficiencies or lack of intelligence or other weakness to the group they are discriminating against. This shows according to Williams that they in fact know and agree that all human beings are equal and have therefore a claim to equal treatment.

2. All human beings have moral capacities: Kant has argued that all men deserve equal respect as moral agents. Williams finds a problem with this as Kant in order to remove this claim from contingencies (i.e. he does not want to allow the capacity for moral action to vary like other talents or capacities vary between men) makes it a transcendental capacity; this results however in the problem that there is a conflict between this vague notion of equal moral agents and the practice of holding men responsible for their actions according to their capacities (e.g. taking into account mental illness, moment of extreme anger etc.). Williams however finds that something is left of this notion in that we can request for every man that his point of view is considered, what it means for him to live his life (i.e. empathy, putting oneself in his shoes). One point Williams makes is that we should bear in mind that society can influence our consciousness (i.e. extreme oppression can lead to the oppressed adopting the same point of view, that they deserve such treatment. Therefore lack of suffering is in itself no guarantee that the system is fair).

Considering the notion of equal opportunities in unequal circumstances: Equality is often discussed regarding the distribution of (limited) goods: Williams argues that in cases of need such need should be the sufficient and operative criterion for distribution. Example: Sick people have the need (illness) and should therefore receive medical treatment (the good). The practice where money becomes a major factor in the allocation of medical treatment (rich people receive better or earlier treatment, the poor delayed, less good treatment or none at all) is according to Williams irrational. The situation regarding goods that are allocated based on merit is somewhat different — they may be desired by those that do not merit them or not desired by those who merit them: Example: University education. In these cases there may be a mechanism to allocate the good e.g. certain grades to be reached at the final exams qualify you to enter university. The problem with this is that the circumstances may give certain groups an unfair advantage so that opportunities are equal only in name. Consider for example that rich people can afford tuition, can send their children to better schools etc. In those cases the question arises whether these underlying circumstances should be altered to provide truly equal opportunities? Williams sees a problem regarding where to draw the line e.g. should one (if it were possible) use brain surgery, genetic modification to erase differences that give advantage to more talented/ intelligent children? Carried to the extreme the notion of equal opportunity collides (and threatens to obliterate) the notion of personal identity and also the notion of equal respect deserved despite existing differences.

Robert Nozick has criticised the idea of need giving a right to receive certain goods. He pointed out that e.g. in the case of medical treatment the doctor providing the treatment has a legitimate right to want to make a living out of his talent/skill, and that this is the important consideration in the distribution of medical treatment. Nozick thinks society should not interfere with unequal situations that have arisen as the result of legitimate actions. You can think for example of a situation where some people chose to save their money, and pay for a better education of their children, the children consequently get better jobs, they marry in the same social circle and due to good connections do even better etc. The resulting inequality is the outcome of normal and legitimate actions. Nozick holds that people are entitled to have and keep property that they have legitimately earned (notion of entitlement).

It is noteworthy that often people argue for certain rights (which in fact both Williams and Nozick do in this discussion) without explaining where these rights come from (are there natural moral laws and rights or not?).

Consider also this: Is my need to eat a cake a sufficient reason for you to give me your cake or a piece of it (Williams)? On the other hand is your having the cake legitimately a sufficient reason for me not to take it from you if I want it (Nozick)? Is it not after all a question of power to take or to keep the cake? And could one not argue that society is a finely-balanced system of power structures where for example the need of the poor for medical treatment is met not just because of the need but because all of us together have a mutual agreement where all pay tax so that such expenses can be met should we ever need them etc. Obviously we feel differently about need for cancer treatment than requests for luxury goods, so should society provide for basic needs of all? What are these basic needs? And is it ultimately not a case of what a society can afford, and therefore a question of wealth?

In a restricted sense of 'justice' consider justice in court — in democracies people are supposed to be equal before the law, but the rich and famous can afford better counsel and can take their case if required through numerous appeals, which is much more difficult for the poor. Education might also play a role in whether you realise all the options you have to make your case. Again — existing circumstances can give the advantage to certain groups as opposed to others.

Helene Dumitriu

back

Kat asked:

Is marriage doomed without major changes?

Perhaps it is people's natures rather than the institution of marriage that is at fault. These days of the decline and failure of marriage coincide with the rise of individualism and the dislike of compromising one's personal potential and ambition to the demands of another. I notice from friends who are in their forties and unmarried that they have put themselves in a position which is so self-orientated that they feel they couldn't get married and give up their freedom and homes and share. Yet they are lonely and wish they could do so.

Marriage makes more demands on a person than co-habitation. In both situations you need to share and compromise, but in the latter case you have a get-out clause if you don't like a situation: "But we're not married". Half-heartedness or a lack of total commitment is not going to help a relationship that must be close to develop. Development of such a relationship comes through mutual knowledge of a sound basis for trust in the other's commitment.

Rachel Browne

back

Jon asked:

Do you believe it is possible that evil is just another form of good? That is to say that man only commits evil in an attempt to promote what he has construed as good? Or do you believe man will go out of his way to create evil for reasons unknown?

Suppose it were true that when someone does evil, he does it thinking it is good. Why would that make the evil a form of good? It would not. It would still be evil even if someone did it thinking it good. Suppose someone tortured a little child to death, but in some twisted way thought that was a good thing to do. That would not make the torture a good thing. Thinking something is good doesn't make it good. Why should it?

People do evil for a variety of motives. Sometimes they are able to convince themselves that what they are doing is good. (Apparently many convicted criminals in prison believe that they are not "really" guilty, and persuade themselves that they had an adequate excuse or even justification for what they did.) People deceived and lie to themselves all the time. Sometimes people are just evil and twisted people and, as a result, do evil and twisted things. For some reason, some people don't think this is possible. I don't know why.

In John Milton's great Paradise Lost, Satan, who has just been tossed out of heaven by God says, "Evil, be thou my good." That is to say, I will from now on think of evil as good, and (I suppose) good as evil." But at least, Satan was honest with himself.

Ken Stern

back

Bajak asked:

How does God know things in detail before their creation even though he doesn't see them?

God does not 'see' them in the sensory sense that we do. 'Things' are conceived in the 'mind' of God and brought to be through their conformity to the Exemplar (Nous-Mind). The Exemplar is the locus for all that God could possibly conceive, and as Exemplar it contains within it the 'ideas' (the Platonic Forms) that God conceives ('sees'). Things are known in their 'detail' before their creation because God is act/ potency/ conception all at once and formally so rather that simply predicated as attributes and because the Exemplar possesses all that needs to be known about anything that could possibly be conceived or brought to be.

In the Christian speculations this is the Logos, the Word, the Son who is the perfect 'blueprint', 'model', Exemplar for all else. God 'sees' all that exists and will exist and could possibly exist already as existing in the Logos/ Son/ Exemplar. Things exist in the mind of God and are brought to be in the Exemplar through a series of emanations (a going out) and already possess every detail that can be known about them in themselves and outside themselves by others. But this is already known to God who in these emanations, gives 'things' their reality in the sensory world, or the world of reality ad extra God (outside God) because they already exist in the intelligible world (the world of Ideas) in every detail of their own exemplar.

Fr. Seamus Mulholland OFM

back

Matt asked:

According to Descartes what would be the problem with the passing down of a family heritage and history, taking into account his Method of Doubt?

What Descartes doubted was that he (or anyone) could know for certain that what we believed, based on sense-experience, was true. So, someone who accepted Descartes notion of doubt, who tried to pass down "family heritage and history," could easily say to others: "Look, I believe what I am telling you is true, and I think I have good reasons for thinking it is true, but I am not absolutely certain what I am telling you is true." With that as a preface, he could happily relate the history and the heritage. When Descartes doubted things, he did not doubt that they were true. What he doubted was that he could be absolutely certain of them.

Ken Stern

back

Matthew asked:

How do we know we exist?

Who's talking?

T. P. Uschanov
Research Assistant
Department of Philosophy
University of Helsinki


A simple answer is that you could not have asked the question (or done anything else) unless you existed. Of course, you need not have asked the question at all. And, of course, you need not have known you exist either. But many things (like animals and inanimate objects) exist, but do not know they exist. But nothing can know that (or believe that, or wonder whether) it exists without existing. So, in a way, your question answers itself.

Ken Stern

back

Isioma asked:

Please, I'd like to know about the claim on African origin of Greek philosophy?

Can you think of any bridges between naturalism and spiritualism? So many people that I associate with claim to subscribe to a vague "spiritualism" which I take to mean a veiled theistic belief.

I regard myself as a naturalist and pantheist as well. It seems to me that these two beliefs constitute a naturalist and spiritualist viewpoint and aren't contradictory. However, as a pantheist, it seems to me that some matter, such as organisms, contains more spirit, or god? than others. Do you have any thoughts on this to sort things out?

Certainly you might look into the philosophy of Baruch (or Benedict) Spinoza (1632-1677), for whom God is Nature. But I would recommend the more contemporary philosophy of panexperientialism (panpsychism, panentheism), especially that of David Ray Griffin (born 1939), who has written several books in answer to the question you raised. As a student of the thought of Alfred North Whitehead (1861-1947), he locates the bridge between nature and spirit in the "actual occasion," a subatomic and momentary "drop" of experience. Everything is either an occasion of experience or is comprised of such occasions. In every actual entity there is an alternation between physical (efficient) and mental (final) causality in the same entity. In this categorical scheme, the age-old strife between monism and dualism is laid to rest. In certain beings that are superorganizations of such occasions, the mental poles of all the relevant entities coalesce into the conscious mentality that characterizes the higher animals, including homo sapiens.

Griffin's naturalism rejects supernaturalism root and branch, and categorically outlaws miraculous interruptions of the natural order. As Whitehead put it in Process and Reality (1929), God is not an exception to the categorical scheme, but its chief exemplification. But Griffin repudiates materialism and atheism no less absolutely. There is an actual entity that influences, and is influenced by, all others. This actual entity provides every other with its initial aim, having selected it from the realm of pure possibilities — a metaphysical necessity for which materialism makes no satisfactory provision. That entity is God.

Materialism affirms that, at bottom, all fundamental entities are "all outsides and no insides," purely vacuous beings, from which premise atheism is a simple inference. But when our hardcore common sense — which we cannot help but presuppose in practice — tells us that at least we have insides (perspectives, points of view), the materialist balks and insists on explaining that fact away. Griffin suggests that we instead make it our model of reality. In his view, all the alternatives have been tried and found wanting.

The above hardly does Griffin justice so, please, track down his Unsnarling the World Knot: Consciousness, Freedom, and the Mind-Body Problem (1998); Reenchantment without Supernaturalism: A Process Philosophy of Religion (2000); and Religion and Scientific Naturalism: Overcoming the Conflicts (2000).

Tony Flood

back

Shane asked:

This is more of a request for an opinion rather than a direct question. I have recently graduated with a BSc (computer science) and while at uni took a few philosophy (ethics, epistemology & metaphysics) and biology papers (evolution). I had always considered myself to be an agnostic but now after much study and contemplation can see no other alternative other than atheism. I am interested in the personal beliefs of philosophers regarding theism. I note that Geoffrey Klempner has stated he used to be an atheist. If anyone shares this view, can they please tell me what they believe now and why?

What immediately sprang to mind on reading this question were some appropriately irreverent items of philosophical graffiti:

God is dead but won't lie down.

On a scale of death, God is mostly dead.

God is not dead merely pining for the fjords ( The Python Defense).

All of which may seem a facetious response to one of the big puzzles of our century; what is it in our species that makes us re-invent God as a Central Organising Principal (C.O.P) or makes us vulnerable to G.O.D Corp. (God On Delivery Corp) in it's many guises?

I once attended a lecture by the physicist Pokinhorn who was talking to the teachers, of whom I was one, of a school in the village of which he was a famous son. He was advocating the 'God stirs the mix' conjecture but could not find a response to the Popperian charge of executing a metaphysical sleight of hand by representing an essentially irrefutable conjecture disguised as a theory of physics. I enjoyed the debate and felt that I had won the game of philosophical tennis even though I was denied the authority of institutional truth.

At an intellectual level I cannot justify the concept of 'faith' when it is based purely on the propositional concept of truth and knowledge. Yet statistically we can see that people move in and out, out and in, stay in, stay out of religious belief. So a scientific account of the world our species inhabits must take this fact into account. It also has to consider the feeling we can have at moments of life-threatening crisis that we want there to be a finger stirring up the probabilities or throwing the dice to shake up the cards in what may seem to be a certain hand expressing extreme prejudice towards us or our loved ones.

So when you have these experiences how do you reconcile the logically driven intellectual view of faith with the emotionally driven view? My personal solution is that I do not see these positions as separately haunting the devil's horns but as indiscernibly conjoined twins dancing on the devils nose, too close to be seen in focus.

At a deep and purely speculative level I believe there are finite logico-group like mechanisms in cells that are 'writ large' in the body of complex organisms and which operate on, in and through us of which we can have knowledge through their emergent works, i.e. what we do, what we know and what we want.

More specifically, I believe that humans have a high level cognitive system the products of which are what we call 'thought' of which the objects are vehicles for disjunctively joined propositional and value content. Education, experience, situations and disposition may dilate or inhibit either of the two passengers so that thought becomes mostly propositional or mostly value based but there is still, even in polarized individuals a residual channel in which the two are still twinned. Within this system there are two distinct units, one of which takes the world as it is as its objects and the second which acts on the objects in the world wants to change or maintain our world. This second unit is also a part of our world and so is subject to its own dynamic.

Within this highly personal and idiosyncratic view, 'faith' and 'hope' can be seen as the emergence of the deep and permanent working of our organism which in normal times may be skewed towards the opposite poles of rationalism and emotivism but in situations involving personal catastrophic crisis almost certainly will default to an unpolarised position in which both propositional and value channels of thought merge, mix and churn turbulently but within which there are created islands of calm from inside which we can reconstruct our world.

From a logical point of view I think our world is perceived within the flow control logic of a three valued classification system consisting of satisfaction, non-satisfaction and indifference the deep constituents of which are propositional and value channels carried in an expectation nucleus. In a crisis or when searching, our thoughts wander across all three states but we can default to an attitude of indifference towards a situation or consciously choose this position above satisfaction, dissatisfaction or frustration. Though most of us I believe default to the latter and become satisfied with non-satisfaction, a self-locking state of affairs.

As you can probably tell there is a theory about to jump up and bite you. While it is mostly developed I haven't as yet worked it through enough situations to offer you a well founded critical method that allows you to think about and retain your beliefs while systematically doubting them while at the same time offering you a vehicle sound enough to replace them.

But I am working on it.

Neil Buckland

back

Sgt Greene asked:

Regarding the question that was posed to Ray Kurzweil, concerning "an entity capable of suffering deserves moral consideration."

My question to you sir, if the above statement is true, is it fair to say that the killing of entities such as animals that support nutrients to the human race is wrong? If it is wrong, what is the basis for it to be wrong, in the areas of philosophy, values and beliefs sir?

Gunnery Sergeant Greene
United States Marine Corps

Singer's key argument that we ought to treat animals as morally important rests on the following argument:

1. An interest is an interest whoever's it may be.

2. Animals have interests.

3. We ought to consider the interests of all those affected by an action.

4. We should act in a way that is most likely to maximise the interests of those affected.

5. Therefore we are not justified in treating human interests as more important than animal interests.

6. Suffering allows animals to fit into this framework since, "A mouse does have an interest in not being tormented, because it will suffer if it is" (Singer Animal Equality).

7. Therefore, it is not objectively wrong to kill an animal for food since there may be circumstances where the interests of the person(s) killing the animal for food outweigh the interests of the animal. Unfortunately for meat eaters there are ways of obtaining all our dietary requirements without the use of animals, therefore not to kill for food is better than to kill.

The basis of this sort of wrongness is in Utilitarian ethics which looks at the consequences of acts as opposed to the acts in themselves; Peter Singer is perhaps the most important utilitarian around.

I think that he is wrong for two reasons, first I think that utilitarian ethics is wrong and therefore don't accept arguments derived from it, I'll not go into this here. Within his framework I still think he is wrong, he moves from the proposition that animals can suffer to the proposition that they have interests, which is wrong. I offer as a counterexample a sadomasochist who actively seeks out suffering. Perhaps this is not what singer means, but here I believe he makes the mistake of confusing:

i. It is in the best interests of x that y

with:

ii. x has an interest that y.

This distinction can be illustrated by comparing:

iii. It is in the best interest of Mike that he ceases smoking

iv. Mike has an interest that he ceases smoking.

It is clear that iii doesn't entail iv. Now given this distinction, the way to find out what interests an animal can have is not through attributing to them what we consider to be in their best interests, but through what they themselves consider to be in their interests. Unfortunately I don't think that animals are the kind of things that have interests since to have an interest that something you must be a rational animal, a requirement I think animals fail to meet. I'll not argue the point here but for reference see:

Davidson, Donald "Rational Animals" in Actions and Events, Le Pore, Ernest (ed), pp. 473-481

Mike Lee


Well this is a dilemma which has puzzled people for thousands of years. Is it moral to kill animals for food? And various groups and societies have come up with various answers to this question. The classical Buddhist answer is that any human and animal (and I'm not actually sure how much the Buddhists separate humans from other animals) killing is wrong, and that we should all be vegetarians. Not unreasonable, especially these days, when you can eat a perfectly fine diet with a wide variety of meat substitutes, protein supplements, etc. Hindus have much the same answer, for similar reasons: if you kill an animal, you may be killing a human being in the process of working off their bad karma in an animal body. So you've interrupted someone, in a series of rebirths, potentially on the path to enlightenment. Christians don't seem to have qualms in this matter because of the Catholic Church's position (and I don't actually know whether Catholics still hold this; nor indeed which of the huge number of Christian sects hold it and which do not; but this was, at least, the Medieval position) that animals don't have souls, so you're not killing a being that can suffer in any deep sense.

So what we need to do, first, is to assume that animals can suffer. Is this so unreasonable an assumption? We share something like 98% of our DNA with chimpanzees, and quite a bit with the other apes. So neurally, they basically have the same structures we do. We can demonstrate that stimulation of various nerves, brain centers, etc., results in human suffering... can one then claim that stimulation of virtually identical brain structures in apes does not? I don't think so. The structures are the same, the resulting behavior is the same... what more do you want? That's about all you get from other people, right? As we go through the animals, then, we find that mammals, at least, have, again, virtually the same structures that we do... much smaller, with some alterations, etc... but the lower-level structures, responsible for emotions, remain very analogous to ours. I think that it's safe to say that they suffer pain, to some extent, and feel pleasure, to some extent. I'll stop with mammals and avoid the fowl/fish issue here for the sake of brevity.

So then the questions become: a) is killing animals immoral, b) is animal suffering immoral. These aren't the same, since it is possible to kill with virtually no suffering: by drugs, for example. But we can start by posing similar questions about human beings. I'm making, for the sake of this discussion, a distinction between humans and animals which is pretty hard to justify in any hard and fast manner, but given the strong prohibitions we have against eating humans, and the weak ones against eating animals, I think that in this context it's justified. Is killing humans moral? Is human suffering moral? Well, of course, the answers are not absolute. You're a soldier, and so you regard killing in war as moral (or so I assume). So there are circumstances, for you at least, in which human killing is moral, and, since suffering in war is inevitable, also circumstances in which human suffering is moral. I'll take your position to be correct, for the sake of this discussion (and clearly there are people who would disagree... but we'll ignore arguments claiming that any killing of humans is immoral, or that killing is an absolute last resort). But you (I will also assume) also hold that in many, indeed most, circumstances, killing humans is immoral. What justifies killing, then? The threat of being killed, right? If others are threatening you, your society (a group of people associated with you in various ways), etc., with death, then you regard killing them either in retaliation for past killings or to prevent future killings which you regard as highly likely, as moral.

Am I right? The threat or reasonable potential of death for you and/or people you regard as your society justifies killing those who you have reason to believe will carry out that potential threat. I'll take that as your position, since we can't discuss it directly to clarify it further. Ok? Now, when is human suffering moral? Clearly, in the above circumstances it must be, since suffering is inevitable in those circumstances. Are there other circumstances in which human suffering is moral? Well, if someone must suffer in order to avoid greater suffering later, then we usually take that to be moral; and if some few suffer in order to prevent either the killing or the suffering of many, then we usually assume that it is moral also. I am quite consciously ignoring the obvious problems with determining what the "amount" of suffering is, how to compare various peoples' suffering, and so forth.

So basically, the common position (and really, I haven't justified this in any deep way; I'm just taking more or less normal positions on killing and suffering as correct, for the sake of discussion) is that killing in order to prevent more killing is moral; suffering in order to prevent either more suffering or killing is moral. A sort of additive morality, which has lots of problems when we actually try to figure it out in the real world... but we need something to go on, right?

Now that's all with humans. What about animals? Well, I have taken the position, above, that animals can suffer. If you don't hold that, then we just can't go further: then you think that animals are little machines; they may behave as if they suffer, but that's just behavior. If you do, then just have at it; chopping up a dog is just like throwing a dish at your TV set, right? Descartes held that position... but I don't, and I'm not going to in this discussion, for reasons given above.

Since we have an additive position on morality and suffering, we take the same for animals. First, animals can suffer to different extents, similarly to humans (human/animal distinction again assumed). Now the big question: is animal suffering as bad, pain for pain, as human suffering? Whoops. How do we answer that? Let's take a little teeny animal that everyone acknowledges is really stupid: a porcupine (hedgehog, to Brits), say. Or we could take a cow... whatever you think is really the bottom of the mammals, insofar as intelligence, awareness, capacity for feeling, etc. (no offense to animal lovers here... I'm just trying to get to the far end of the spectrum from us, inasmuch as that's possible). So when we cut a porcupine, does it (the porcupine) hurt? Yes, by the reasoning above. Is its pain as bad, morally speaking, as the pain when we cut another person? No... you say...? Why not? Well, if it's not "as conscious" as us, then its pain is not as intense, or at least, its suffering is not as intense. Ok... not unreasonable, given other differences we see more clearly, like intelligence, neural structures, and so forth. I will assume this for the sake of our discussion. But really, how do we judge this? And also, notice that if we make this judgment, we are forced to make it within humans also. Not a very comfortable conclusion, right? But how do we escape it? On the other hand, putting everything, all animals, all humans, on the same level, insofar as feelings go also seems wrong. Is it really justifiable to claim that all mammals feel equally, given that there are clear differences in cognitive, linguistic, neural, etc., capacities? I will assume not, and that is the position normally held... but you can see the problems here.

But second, it really seems that we have to look at the consequences of feelings as well. That is, killing a human being (and/or human suffering) has, potentially at least, greater consequences for the world than the killing (or the suffering) of a porcupine, even at that latter's most influential. This is a strange and interesting consequence of looking at feelings, isn't it... that in order to evaluate the value of a feeling, we must go beyond the feeling to it's consequences. But really, we do that for all actions, don't we, and a feeling is at least a kind of internal action. So let's go with that, again for the sake of discussion. Well, if we go this way, then everything falls into place, in a simplistic kind of way. Animal suffering is bad (immoral), yes, but not as bad as human suffering. Killing animals is bad, but not as bad as killing humans. I've got to say that I personally am not unhappy with this conclusion; it does seem in line with normal moral reasoning, and indeed with reasoning as it applies to humans exclusively as well (but we have seen some of the problems with it, above). That is, we do, when we're making hard judgments about killing and suffering with respect to people, judge in just the above manner: it's hard and painful, but we have to try, on the basis of both the depth of feeling and the consequences of feelings (and other factors, obviously), to grade or rank the morality of various alternative actions. And we do that grading or ranking, I believe, in something like the above manner.

Now... yes, as I say, the above distillation and compression of morality into one easy-to-digest paragraph is pretty ridiculously simplistic... but I'm trying to answer a question here, and I've already written a huge amount for this forum. Enough.

So, then, to answer your question. Is killing animals to eat wrong? Not if you have to, to eat, since humans have more value than animals. Less, if the animals don't suffer. What if you don't have to, to eat? Well, you know, I just don't know the answer to that, and here's why: you go to a market, and there's dead animals. Will your not eating them help them? No. Will it stop other animals from being killed in the future? Maybe, depending, I would think, pretty much on where the market is, how much your not eating them is noticed, how much economic effect it has, etc. Should you avoid eating animals on the chance that it might stop them being killed? Probably yes, given a) that it might have that effect, and b) that your not eating animals will not cause human suffering. First, do you have alternatives available? Second, there are people who live by hunting, meat processing, and farming, etc., and they can't just walk away from that, even if they want to; they're part of huge social systems, not to mention having families to support. Not an easy situation, then, with easy answers. But the trend, I think, the way we should try to move things generally, should be towards not killing animals and towards not having them suffer unnecessarily.

Steven Ravett Brown


I think that the view "an entity capable of suffering deserves moral consideration," does not imply (as you seem to think) that animals should not be used for food. Some might think so, but to say that entities which can suffer should have moral consideration might imply only that they should not be treated cruelly, nor given pain for no reason. So, a person might hold that animals may be killed for food, or even for other purposes, as long as the killing is done with the least amount of pain practically possible.

Hope this helps.

Ken Stern

back

John asked:

How do you think a hard determinist would react to the argument that the death penalty deters crime?

I have never thought that a hard determinist would have a problem with this. Assuming for the sake of argument that it is true that the death penalty does, in fact, deter murder, then a hard determinist would say something like this:

A murderer is put to death. This is a physical event, and it has physical effects. Some of those physical effects (very complex ones) cause reporters to write about the death. Other physical events cause a reader to read about this death. This reading (itself a complex physical event) causes certain changes in the brain of the reader. These changes in turn alter the future behaviours of the reader. Some behaviours become more likely, others less likely. One of those behaviours is the plotting of a murder — itself yet another complex physical event that requires certain brain states to come about. Thus, the execution can be seen to affect the frequency of future murders, without any reference to desires, emotions etc (which are merely misleading summary terms for complex physical events which are all that really matter). All you have to add is the idea that any given physical state causes (by the laws of physics) one and only one future physical state, and you have hard determinism.

Tim Sprod


I believe the question reveals a common confusion about determinism. The question is often asked: If all human action is determined, then how can we punish criminals, since they couldn't do otherwise? But if all human action is determined, the action of punishing criminals is determined just as much as the action of committing crimes! The determinism debate is haunted by the spectre of a criminal saying to a judge that he cannot be convicted, because all freedom is destroyed by determinism, and all he really could do was therefore to commit crimes. But such a criminal is in no position to complain if the judge replies to him that because all freedom is destroyed by determinism, all he, the judge, really can do is therefore to find him guilty and sentence him.

And if hard determinism is true, then everything about the death penalty is determined just like everything about everything else: for instance, it is determined that some countries have abolished it (at least for the time being), and it is determined that some other states have not (at least yet).

But if the question is whether there is empirical evidence that the death penalty is statistically more efficient in preventing crime than lengthy imprisonment, the answer is that there are 200 years of research in the social sciences on that very question, and the answer is a resounding "no". That's one reason why I, for one, am a hard determinist who opposes the death penalty, and I hope that my opposition will eventually contribute to determining things so that they will lead to its worldwide abandonment.

T. P. Uschanov
Research Assistant
Department of Philosophy
University of Helsinki

back

Alexandra asked:

I would like to know about "Comficius"

I think you must be referring to Confucius, a Chinese master, whose name in Chinese is K'ung fu-tzu. His ideas are only known from dialogues and sayings recorded by his disciples. He regarded moral values as the basis of social and political order, and is particularly thought of as promoting the ancient hierarchical values of Chinese society on this basis.

Hope this is what you were looking for, you should be able to find out more on the internet now you have the correct spelling of the name.

Katharine Hunt

back

Jeff asked:

What is memory? How important is it to my identity? Why is my short term memory getting worse as I age? Why am I starting to remember random things from my childhood that I haven't thought about since they happened? Along the same lines (I think), what is the current thinking on the phenomenon of deja vu?

An enormous question which has puzzled both philosophy and science for thousands of years. From a materialist point of view scientists, although admitting to being baffled by certain aspects, believe that they are now nearer to solving the problem, however, they have still a long way to go to convince most philosophers. In philosophy memory is a very important facet of philosophy of mind.

Both philosophy and science accept that the mind is daily bombarded with an enormous amount of information: this information is subjected to a very critical filtering process, links are made with already stored information and some of the new information is added to the store, the rest is rejected. However, some scientists claim that nothing is actually rejected and any piece of information could be recalled under the right conditions. For example, it is claimed that vital facts have been recalled by witnesses in legal arguments when subjected to hypnosis.

Because we can know ourselves only because we can remember, we are also involved with theory of knowledge (epistemology). When we investigate knowledge of ourselves, knowledge of our lives past and present, we are directly involved with memory. We are daily involved with two sorts of memory, short term (working) memory, and long term memory. Although it is common practice to identify the two types of memory as separate activities they are constantly interacting, and are totally interdependent. An impaired working memory means that we cannot learn anything new, and hence cannot pass new information into the long term memory. An impaired long term memory, on the other hand, means that the working memory is unable to retrieve information from the long term storage, and hence we would be suffering from dementia or alzheimer.

It is very much a fact of life that our minds are constructed to form habits. Careful analysis of everyday activity reveals the fact that we are very much dependent on habit. Further analysis discloses that habit and long term memory have a lot in common. Constant repetition eventually develops into habit, we progress from having to think about things to doing them automatically, seemingly without thinking. Consider the examples of learning to ride a bicycle, to drive a car, to play an instrument, knowing which way to turn when you leave your house to go on an errand or to work, talking to a passenger and driving, as though two separate minds are at work, etc.. The learning process is difficult, the working memory cannot retain much data, and only then for a short time, hence we have to repeat things over and over again until by some process or other a memory trapping mechanism transfers the data to long term memory. If this did not happen we would daily be in serious trouble, every event would have to be re-thought. Every important and necessary event in our normal everyday activity has been committed to memory; because we are so used to performing everything automatically we fail to recognise that we are relying on memory.

The memory is very efficient and once information is established within it, unless we are beset by some disease or accident to the brain we do not forget. Some claim that there is no such thing as a bad memory and that more people suffer from not being able to forget things that haunt them than do from not being able to remember. Also, how many people are unable to immediately repeat the alphabet when called on to do so?How many people would fail to be able to count to a thousand or more immediately?Say the nursery rhymes of their childhood? etc. Children of pre-war years and later who learned their lessons by rote were far more efficient in mathematics, spelling, poetry, literature, etc.. The decline came with the advent of calculators and other memory saving gadgets. There is no doubt that constant repetition, i.e. constant pressure on short term memory, eventually leads to transfer to long term memory. Also, as I used to tell my students, interest is the bedrock of memory. On the first day of the new academic year I would point to the door and advise that anyone not really interested in the subject ought to leave, as they were wasting both their own time and mine.

Having gone some way to answering your question on identity by saying that we can know ourselves only because we can remember, consideration of what has been said about long term memory should complete the picture. Memory ensures that we can locate ourselves in space and time. The bigger question is, What is the self that does the locating? If the self is not the memory but something that uses the memory, then your question about identity takes on a new meaning. Hume gave a great deal of thought to this problem and found that he could never identify a self observing events of memory, he was somehow just aware of memories.

Difficulties arise not with memory itself but the mechanism of recall; when we make mistakes the memory itself is not at fault if the wrong information is recalled. Also, the memory is not at fault if we find it difficult to recall something. As we age somehow or other, no one is quite sure how, our faculty of recall becomes less efficient. The memories are still there but we find it more difficult to locate them. Usually it is a slowing down condition, we need more time to recall events or names etc. It is noticeable when an elderly person meets someone in the street whom they know well but cannot instantly recall the name. This is not sign of a disease but a process of ageing. Dementia and alzheimer are diseases suffered by a relatively few people and should not be confused with the natural ageing process. Many people retain very efficient memories into very old age. The more we use the mind and exercise the memory the more likely we are to retain our faculties to the end of our lives, however long that may be.

I have no idea of your age, but remembering random things from childhood is not unusual as we get older. I am not claiming that this is true in your case, but very often as a person gets older nostalgia creeps in, they feel less to belong less to a modern world subject to values which seem less appealing than those that were instilled into them when they were younger. Also, children are more curious and learn more rapidly and more efficiently, memories strike home with greater impact and stay there, suggesting again that interest is the bedrock of memory.Your experiences are proving the point that we do not lose the memories established in long term memory, and that we are here involved with the recall system. Unless we are concentrating on something the recall facility becomes free ranging, and often in relaxation will wander into the long term memory and release random events into consciousness. Older people call this day-dreaming.

With regard to deja vu science has never relinquished its notion that this is a fleeting disorientation of the mind which happens to almost everyone during a lifetime. Scientists will never accept a mystic explanation. However, some philosophers, and others with an orientation towards mystic events might consider deja vu to have something to do with the continued survival of mind and the possibility of reincarnation. Most scientists are bent on proving that mind, memory and mystic events are to do with neuro — electricity, proteins, hormones and genes.

John Brandon

back

Chera asked:

My prof. mentioned a woman once wrote Kant a letter about suicide What's the story on that?

The woman was Maria von Herbert, an Austrian aristocrat and enthusiastic follower of Kant. She sought his counsel after a break-up from her fiance had left her feeling suicidal. She did kill herself in the end, but more than a decade after Kant's reply to her.

For the full story, and a discussion of its philosophical implications, see Rae Langton's paper 'Maria von Herbert's Challenge to Kant', an abridgement of her 'Duty and Desolation', Philosophy 67 1992, reprinted in Peter Singer (ed.), Oxford Readings in Ethics (Oxford University Press, 1994). It is available online at:

http://homepages.ed.ac.uk/ajbird/teaching/Introduction_html/KantMaria.html

Andrew Aberdein


You can get the series of letters written to Kant by Maria von Herbert, and his replies, in Sommers, C., & Sommers, F. (1985). Vice & virtue in everyday life; introductory readings in ethics. Fort Worth, Texas: Harcourt Brace & Company. Pp. 175-191. Fascinating, and I don't think Kant acquitted himself very well.

Steven Ravett Brown

back

Carmen asked:

Something is really bugging me and I have been arguing with two of my teachers. My english and philosophy teachers claim that we can have false knowledge. From what I have read, the definition of knowledge as "justified, true belief" has truth as a condition of knowledge. Then how can one know something false? If something that we knew turned out to be false, can it be said that we have known it? And do we still know it?

My philosophy teacher tells me that all that I have read regarding the fact that we can know only that which is true is crap. I can only accept his point of view if the definition of knowledge didn't include the word "true", therefore rendering justification enough of a condition for knowledge. After all do we really know what truth is? Please clarify this for me. Are my teachers right?

Your question makes a lot of sense. Is it possible to have knowledge about something that is not true? Theoretically, truth is part of the definition of knowledge. It seems senseless "to know" a falsity.

There are two ways to approach the question. The first is related to language; the second is related to temporality.

Let's start with language. A reflection on the use of "to know" is necessary. Suppose the following situation: someone complains about having a headache. You ask him a question: are you sure you have a headache? (Assuming he might be simply pretending to have a headache). His answer (a little offended perhaps) is: "I know I have a headache!" It makes sense, doesn't it? He used the verb "to know" to emphasize a trivial fact — his headache.

Now does the phrase "I know I have pain" always makes sense? Let's see: suppose someone simply says "I know I have a headache" You might ask a question: "How come? In what kind of situation could you say that you don't know whether or not you have a headache?" See, Carmen, the use of "to know" is quite specific. It seems reasonably normal to say "I know I have pain" but to say otherwise "I don't know if I have pain" sounds weird. In fact, the use of "to know" in both cases proves to be an inappropriate use of language. We do it all the time; we use words or concepts in situations in which they don't fit exactly. In the case of knowledge, we can only say that we know something if also makes sense to say that we don't know something. It is appropriate, for instance, to say "I know it will rain tomorrow" because it is also appropriate to say "I don't know if it will rain tomorrow". To be short: knowledge is not related to the truth, but to the possibility of truth. Thus, it is possible to know something that is not true: forecasters are experts in it!

Another way to approach the question is from the temporality point of view. Some centuries ago, scientists affirmed that Earth was the center of the Universe. They "knew" it was true. Experiments proved that fact. It was scientific knowledge. Then someone realized that things were not quite so. It was discovered that Earth is not the center of the Universe. Moreover it moves around the Sun, not the other way around. It was a great scientific revolution, wasn't it? What now? Did the scientists that "knew" the Earth was the centre of the Universe know something?

The answer is yes. They knew, but they were wrong. That is quite different from saying that they didn't know. Their knowledge in due time proved to be mistaken. What was true in a certain period of time became false in another one. It may sound strange, but it's quite simple after all: what you know today is what is true today.

A more trivial example: Your name is Carmen. You know it as well as your family, your friends, and so do I. All of a sudden you come across a document proving your name is actually, say, Carmena. Does it mean you didn't know your own name all the time? Of course not. You merely had a wrong knowledge, in due time corrected.

If you combine the two approaches of knowledge, it becomes clear that is possible to know something that is not true. Such is philosophy.

Mauricio Micheletti


I do not agree with your teachers; I think that their positions are derived from post-modernist writings, which thankfully are beginning to be severely questioned. Those writers take truth to be culturally and socially determined at best, and at worst completely undeterminable. Given that, one must of course revise a definition of "knowledge" so that truth as anything universal does not enter into it. Fortunately, as I say, there are many scientists today who are causing philosophers to reappraise that position. No matter where you are, in what culture, or whatever you may believe, when you flip a light switch the light comes on (and if it doesn't, you can easily find out why). There's a lot of physics behind that simple action, and it holds universally. This is just one example of phenomena which are rather hard to explain away as culturally biased truths. One may proclaim that physics is based on uncertainty, on "constructs" like virtual particles, etc... but those people are still faced with the problem of backing up a claim of cultural bias for science when machines like the electric light, the toaster, the automobile, the computer, and for that matter, the atomic reactor, a machine directly dependent on just such low-level "constructs", just keep plugging along, no matter what culture they're in, or whether the people around them understand them or believe they should or shouldn't work.

The postmodern anti-truth bias of course has its own problems in that very area: how can an assertion of non-universality be true, if there is no truth? But aside from that rather obvious paradox, claims that the lack of rigor in scientific methodology or theories, or the fact that such theories are incomplete and subject to revision, implies either that they are not true or that there is no truth to be found, is quite simply a misunderstanding of the scientific process and its dependence on induction and on extended processes of verification, both quite necessary, but disconcerting to those who want easy answers and absolute certainties.

One very simple argument against the postmodernists is to walk over to the nearest light switch, flip it a couple of times, and raise your eyebrows. If they still don't get it, well.... "Flippancy" aside, you might check out these modern and enlightened philosophers of science, who have extended arguments supporting the same points as mine: Kitcher, P. (1993). The advancement of science; science without legend, objectivity without illusions. New York, NY: Oxford University Press. Giere, R. N. (1999). Science without laws. Chicago, IL: The University of Chicago Press.

Steven Ravett Brown


To start with, let's clear up an ambiguity: There is a distinction between knowing that a proposition is false, and knowing a false proposition.

In this case, what you are doing is knowing that it is true that a particular proposition is false. For instance, I know that it is false that the earth is flat, Which is to say, I know it is true that it is false that the earth is flat. So, if knowing what is false only means knowing that a particular proposition is a false proposition, that's fine.

But if your teacher really means that you can know about a true proposition that it is false, then that teachers is just contradicting himself. After all, when I say, "I know that London is the capital of the United Kingdom, I am just saying elliptically (or for short) I know it is true that London is the capital of the United Kingdom.

It is hard to argue against your teacher unless he/she gives an argument for his view or, as least, gives an example or two of "false knowledge." Sometimes people confuse the fact that I may think or believe I know something, when it turns out that the something I believed I knew turned out to be false. In that case, of course, I never knew it in the first place, but only thought that I knew it. As soon as I discovered that I was mistaken, I would have to concede I did not know, and withdraw my claim to know. That is one of the big differences between knowing and believing. When I believe something, even if someone shows I am wrong, I can still maintain that I did (at the time) believe it, although I don't believe it anymore. But, unless I was insincere when I said I believed it, then even if I believed what was false, I still believed it. The matter is entirely different with knowing. When I said (sincerely) that I knew, then I did think I knew. But when I find our that I was mistaken, I can no longer maintain I knew anyway, even if I was wrong. I have to concede I did not know in the first place. Very different from belief, don't you agree. And, of course, it is said that for thousands of years people "knew" the earth was flat. But notice the inverted commas around "know" in the above. Those inverted commas mean that the term "know" in being used in a deviant sense. That those people did not really know the earth was flat. How could they have if it was round? The fact, if it is one, that a great many people believe something is true does not mean it is true. History shows that there have been numerous cases when many people, perhaps a whole society, have thought they knew what was in fact false.

So, I think your teacher is simply wrong, and maybe he is wrong because he confuses:

1. Knowing that something is false, with knowing something false
2. Believing one knows something and really knowing it
3. A whole people or society thinking they know something, and their really knowing that thing.

Ken Stern

back

Miguel asked:

I am reading Plato's Republic, my task is to do a paper on my favorite topic of the book. What can be a good topic to develop my paper on one part of the Republic?

You should choose whichever topic is your favourite topic of the book! As part of your answer you ought to be able to explain why this is your favourite topic. But here are a few ideas that might help you choose.

You should choose a topic that you think you understand quite well. You should choose a topic that sparks off ideas of your own — this will help you to be original in your writing. It often helps to choose something you partly agree with and partly disagree with, as this gives you lots to talk about. But it can also be fun to choose a topic where you strongly disagree with Plato's views; this will give you the opportunity to develop a good critical argument. It is often easier to write about something if you feel strongly about it.

Katharine Hunt

back

Pamela asked:

What impact can philosophy have on politics and peace in the world today?

Philosophy can only have an impact if people are prepared to listen and to debate. Unfortunately, it is unlikely that those who wield great power in the world today would be disposed to listen to, or to debate, philosophical views pertinent to political attitudes or to world peace. It is highly unlikely that we would ever come across a political party, or government for that matter, indulging itself in a debate on the political views of Plato, Locke, Hume, and others, despite the fact that a basic conception arising from these sources underlies the idea of democratic government. This idea is based on the belief that politics constitute a two way agreement; a government assumes the right to lay obligations on its subjects, traditionally construed as a moral right:on the other hand, subjects are considered to have an obligation to comply with what the law requires, this too is regarded as a moral obligation. Locke puts forward the general view that government can have no other foundation than consent. Hume agrees with this, but at the same time observes that it fails to correspond to the realities of life in political societies. Political obligations are taken to consist of all those obligations we have as a result of laws to which we are subject. However, the focus of much controversy is how one person or group of persons could come to have such a right to the obedience of others.

Hume was right in his day and would still be right in this modern age, the reality of the political set-up is a long way from the naive theory put forward by Locke. There is still an underlying idea that somehow politicians are elected to to carry out the will of the people, certainly in Britain this is the notion implied during elections, however we always note a different outcome after the election, when the victorious party sets about to do virtually as it wishes.

The basic sense of politics is completely lost in the party system, where electors are deluded into the idea that they are making a real democratic choice, Sadly, if the elected party is failing the only real choice available for the electors is to reinstate the party they previously kicked out. Is it any wonder that interest in politics is waning rapidly. No work is ever done on changing the system, certainly the politicians themselves seem not to wish to do anything about it; this is because their loyalty is to the party rather than to the public who they are supposed to represent. this is certainly an area in which philosophy could have an impact.

Televising parliament has been a revelation for the general public and has gone a long way to creating disenchantment. Many times people are amazed to find that issues they consider of major importance are being debated by a handful of disinterested M P's of which half of them have nodded off. Often they find that their own representative has not even bothered to turn up. However, they will note that issues which are vital to the well-being of the party are extremely well attended. This forces the opposition to turn up in strength to try to make life difficult for their opponents. Is it any wonder that most people see politics as nothing more than a silly game which is costing the nation a fortune. Here again philosophy could have a massive influence.

Power seeking is still a major objective in politics. However, this situation has now become more serious owing to the fact that governments are not the only holders of power and influence in the modern world. Global capitalism is now causing great concern, and the access to governments by huge multinationals is very disturbing. Big business is progressively taking priority over human issues, and governments are not only turning a blind eye to what is going on but some often seem to be in collusion. We find very little being done to stop deforestation, global warming, over-fishing, whaling, climate pollution, etc..The excuse being that interference in these issues will "upset the economy." Philosophy could certainly have a lot to say in this area, and could certainly provide a close analysis and interpretation of what capitalists mean by "the economy."

With regard to peace in the world, politics rather than providing a solution is usually, along with religious extremism, the instigator of hostility. Moral philosophy and philosophy of religion could certainly provide alternative approaches, but again, who would listen? The answers will not come from philosophical intrusion but from increasing public awareness of what is going on in the world, and eventually something akin to a global revolution.

John Brandon

back

Bex asked:

What is Paul Churchland's argument for eliminative materialism? What is 'eliminative materialism'?

What is the contribution or potential contribution, of pragmatism to the philosophy of science? What exactly is 'the philosophy of science'?

What part does empiricism play in Deleuze's philosophy?

Eliminative materialism doesn't reduce the mental to the physical, so you can be a realist about the consciousness as well as an eliminative materialist. Rather it is the view that psychology is to be eliminated in favour of a more scientific account of human behaviours, which doesn't mean we necessarily have to give up psychological talk, just that it is false. Churchland has a four part argument for eliminative materialism. Firstly, that we can't reduce psychological states to underlying neurological states because different types of systems can underlie functional states hints that psychological descriptions are false. Secondly, it is obvious that though we have used folk psychological description of behaviour in terms of belief and desire for thousands of years, we haven't advanced very far and have little understanding of memory and sleep, for instance, and thirdly, it doesn't explain abnormal cases of human behaviour. Finally, folk psychology isn't really a developing system of explanation, and remains primitive and although it seems deeply ingrained as part of our conceptual scheme, there is no reason why it can't be given in the same way as "caloric" and "phlogiston" have been. Furthermore, other theories such as an identity theory seek matches between intentional states and neurological states without developing probabilistically in terms of likely success, so it is plausible to take the position that there is no such thing.

The impossibility of inter-theoretic reduction may well be true. However, it is argued in favour of folk psychology and against Churchland that we are making psychological advances and that depth psychology which applies to abnormal human behaviours is an extension of folk psychology.

I'm afraid I don't know much about the philosophy of science but, according to my dictionary, it is the critical examination of the methods and results of the sciences. The pragmatist holds that a scientific theory is true if it helps explain the relations between our experiences, so it is anti-realist. The pragmatists Dewey and Peirce stressed the social nature of science rather than whether or not scientific theories adequately describe the way things are. The main contribution is that a scientific theory is accepted rather than true which justifies scientific induction and has allowed scientific methods to expand into non-physical realms such as psychology. Pragmatism is an approach to theorising that allows Deleuze, in analysing a cinematic image to say at one point "Knowing whether an image is subjective or objective no longer matters"

On the Deleuze question, British Empiricists hold that we acquire knowledge of the world from experience, which is not the sort of empiricist Deleuze claims to be. He calls himself a transcendental empiricist and introduces a level of immanence, or being as Life, which would be necessary for knowledge and so knowledge could not be simply built from Humean impressions and relations. Thought, for Deleuze creates the truth, shaping the way we see things, both empirically and aesthetically. Images come dominated by thought and separation and the empirical image has no priority over metaphorical understanding in this sense. So this is not British Empiricism, since while Deleuze says that the sensible comes first (as British Empiricists understand the basis of knowledge), for Deleuze, this is not simple Humean sense ideas or abstract Lockean ideas, since for Deleuze the sensible comes as something already actualised or differentiated by thought, determined by both movement (our bodies) and sense experience.

French empiricism, unlike British, is a form of naturalism, rather than an account of how we acquire knowledge and though it gives priority to sensory experience over the rational but Deleuze allows for a variety of enriched experiences which cannot be reduced to ordinary empirical experience which is why he is a radical empiricist. Searching the internet I find there is a book called Multiplicity and Becoming: The Pluralist Empiricism of Gilles Deleuze, Studies in European Thought XV which you might want to look at if you need background in European thought and empiricism.

Looking into how Deleuze is classified, I find that in the philosophy of the desiring subject, Deleuze is called a naturalist, yet in his theory of literature, he is called an idealist.

In his theory on literature, Deleuze has been called a "structural idealist" (see Literary and Theory and Poetry ed. David Murray) because of his creation of a de-territorialized "cultural space", and in his cinematic work he takes the medium to have it's own expressive space, differing from the empirical, and given his claim that the cinema contributes to the way we view the world, again Deleuze might be described as a non-empiricist but forces which give rise to cultural or a filmic space, give rise to a structured experience, even if the force is not directly from the objective world. There are different types of force giving to rise to our thoughts and ideas. The empiricism lies in the priority of the sensory or experiential, but this can be produced by any medium, forces creating different (but each equally real) systems of reference.

Rachel Browne

back

Steven asked:

If God created logic, is a rational understanding of God possible?

The riddle "can an omnipotent God create a rock so big he can't lift it" implies that there is some type of contradiction in the concept of an omnipotent being. However, as the creator of logic, God would not be bound by logic. If God can transcend logic, then the contradiction above evaporates.

Is it possible to draw conclusions about a God that can transcend logic? For example, if assertions from religious scriptures about God are used as premises, would conclusions drawn from these premises be meaningful? Is it possible to form a valid logical argument relating to God if God can transcend logic?

If God created logic, then there was no logic before that creation, which therefore could not itself have conformed to the principle of contradiction. If God created logic, then the pre-logic God was absolutely incoherent and chaotic (in which case, why should we trust that inference?). If God created logic, then God would not be "bound by logic" but rather in bondage to illogic, which is how "transcends logic" sounds to my ears. Why speculate as to whether a "rational" understanding of such a God would be "possible"? What would "possible" mean here, anyway? Logically possible? What is logical possibility in a world in which logic is "created"?

Also, there is no such thing as the concept of an omnipotent being: some concepts of "omnipotent being" are internally consistent, some inconsistent. Resorting to the notion of a God who transcends logic is a high price to pay to solve a riddle. Why not just modify the notion? "If God can transcend logic, then the contradiction above evaporates" — and so does "If . . . then . . ." (Why would we want contradictions to evaporate? Because we're "wired" to?)

Charles Hartshorne suggested, persuasively in my opinion, that logic and metaphysics ultimately do not differ. The principle that denies meaning to a contradiction is a principle of reality, not just of how our minds are structured. What he had to say bears directly on the theistic issue that Steven raised. Thanks in advance for indulging me:

"[W]e may divide knowledge as follows: mathematics, dealing with various 'possible worlds,' or better, various possible logical structures; natural and social science dealing with the one actual world; metaphysics, dealing with what is common and necessary to all possible states of affairs and all possible truths, including adjudication of the question whether 'there is no world at all' represents a conceivable truth or is mere nonsense or contradiction. Now God is conceived as the actual creator of the actual world and the potential creator of possible worlds . . .; hence divinity is not a mere fact or fiction of the actual world, but is either nonsense, in relation to all possible states of affairs, or a necessary reality, in the same relation, that is, the idea is metaphysical...

"Whether and how we can distinguish between metaphysics and logic is more difficult to say. I am not sure that they do differ. It seems easy to show that logicians today disagree on what are plainly metaphysical questions (referring to what is common to all possibility): such as, Is all truth eternal? Is there an a priori principle of causal connectedness? Is 'some world exists' true not merely in fact, but necessarily, or in any possible case? In this book I am trying to set forth the logic of basic theological concepts; but perhaps these are the same as the theistic implications of basic logical concepts. If only a few logicians could be induced to look into the matter! On one point, at least, I believe metaphysics can agree with contemporary logic: metaphysical truths, if valid, must, since they are to be necessary, be 'analytic,' if that means, 'certified by meaning alone.' I am confident that the theistic question will be rationally settled when .. . it becomes really clear to educated persons what are the possible consistent meanings . . . of 'supreme being,' 'absolute,' 'perfect,' 'necessary being,' and the like. To hasten that time is the main object of this study" (Charles Hartshorne The Divine Relativity (1948).

Tony Flood

back

Sarah asked:

I'm not a philosophy student, but I read this site with interest.

I was particularly intrigued with the notion that Atheists (I am one myself), see what God does as a greater 'Good' for humanity. The idea appeals to me. However, in the religions that I am aware of and included in the idea of Good, is the unacceptability of wantonness and hedonism.

For example, all cultures use a drug of some sort. Human beings, of all sexual orientation have shown traits of promiscuity from the beginning of time. Why do we seek to eradicate this aspect of ourselves; no religion in the western world has succeeded to my knowledge. Does philosophy seek to think and reconcile this seemingly normal human behavior?

The philosopher who might most interest you, then, would be Friedrich Nietzsche. He characterises most religions as a product of 'herd mentality', which emphasises self denial and self inflicted pain. He argues for a morality built on the 'will-to-power' — what he calls a 'master morality'.

Now, I would guess that you would be unlikely to go along with all that Nietzsche says, because among the things he attacks are pity and feeling for others. Yet he does celebrate the 'life-affirming' virtues, which would include the drug and sex related pleasures you mention. If you want to read Nietzsche (and he is not an easy read, though he is very entertaining), then a good place to start might be "The Genealogy of Morals".

Tim Sprod

back

Gabriel asked:

I am having trouble with the perception of illusory percepts such as induction gratings.

I'm afraid that I don't understand what "having trouble" means. You can't see the illusions? You don't know if others really see them? You don't understand why you see them? You want to know the epistemological/ ontological implications of perceptual illusions? I'll assume it's the last two, or something like that. Ok, first, go to Google and look up "illusions" on the web. There are an enormous number of sites dealing with optical illusions.

Now, what you're asking, maybe, is that if we see the world, what do we see when we see illusions? I'll give you a quick explanation which is based on a cognitively-oriented viewpoint. We don't, entirely, see the world. What we see is partly (we hope) what's out there, and partly constructed from inside, from processes which organize and filter what's out there. Now, it's possible to screw those processes up, in various ways. Get too close to their operating limits, combine them in ways we haven't really evolved to handle, have them deal with situations that they weren't evolved to deal with. When you do that, you get a situation in which a process which we have very little control over (if any) is operating on some sensation(s) in an inappropriate way, and you get various illusions. Ok? Their epistemological status, then, is that they're illusions: false ideas, untrue perceptions. Their ontological status is, usually, that they don't exist except in the sense that an individual idea exists. But that latter is a bit strange, really. That is, say that we grant some sort of existence to the numbers, i.e., the ideas corresponding to "1", "2", etc., or to colors, i.e., the sensations corresponding to "red", "blue", etc. And we do this on the basis of their universality and constancy, i.e., we all see or conceive of them virtually identically, in identical situations, more or less, and so forth. In that case, it seems to me that illusions should have the same ontological status; they're just as universal and constant, evokable in the same situations by the same stimuli, just like colors or numbers. But since I'm not an idealist, I don't think that colors or numbers exist either, so it's not a problem for me.

Steven Ravett Brown

back

Jose asked:

I would like to know how to defend the idea that philosophy is part of our daily life and people can't get away from employing logic for example and aesthetics?

IF you ask a question AND I give an answer, THEN you may be satisfied with my answer OR not. IF you don't like my answer, THEN I'm glad AND so are you. IF you don't like my answer, THEN I'm not glad and so aren't you. You may disapprove my answer OR not. I may accept your disapproval OR not. And so on... You can see that we are using Logic all the time. When a politician presents us his ideas, our first reaction is: "But it doesn't make sense!" And "make sense" means actually being in accordance with Logic. Whether a football coach or a software programmer, everybody needs Logic to accomplish his tasks. (Computer itself is nothing more than a Logic machine).

Aesthetics plays a role as important in our lives as Logic. You may not be a museum freak and regard abstract art as rubbish. No trouble with that. Now, what kind of car or house you've dreamed of? A car could be comfortable, powerful, speedy and even so not to meet your expectations. A house could be comfortable, spacious, well located and even so you couldn't like it. These are rational standards. But when we choose our cars, houses, clothes, etc, there is an emotional component aroused by our aesthetic judgment. If your dream is a Ferrari or a Porsche, a mansion in Florida or in the Riviera, that's because you think those things to be aesthetically superior. Or, to make it plain, it's very beautiful. Wasn't beauty an important factor for human beings, there wouldn't be so many types of, say, mixers. Every mixer in the market performs the same functions, but even so the industry provides many models so as we can make our aesthetic choice. Aesthetics, as well as Logic, is part of our day-by-day activities. The same as other parts of philosophy, like ethics, politics, etc. Philosophy is present is our daily lives even if we don't realize. That's why we have to study it.

Mauricio Micheletti

back

Karolyn asked:

Do you know who said: "If you want to improve, be content to be thought foolish and stupid"?

I think it's Pithatis but I am not sure (I am not sure I even spelled that right!).

The quote is from The Encheiridion of Epictetus sometimes referred to as Epictetus' Encheiridion by Simplicius or Of Tranquility and The Means Thereto, which you can see at:

http://www.publicbookshelf.com/public_html/Outline_of_Great_Books_Volume_I/epictetus_bea.html

The full sentence reads "...be content to be thought foolish and stupid with regard to externals and inessentials" which seems to change the meaning, but later it is said "meekly bear, then, a person who reviles you" and that a proficient person "does not care if he appears stupid"

Rachel Browne

back

Laura asked:

What is the relationship between Plato and permanence and change and how does it apply to the Simile of the Line?

According to Plato, what we see, hear, taste, feel, or smell is subject to constant change and therefore expressed in mere opinion, while true knowledge is of what is stable and unchanging. For example, we can see water freezing, knowing it's still being water, an individual changing a lot over the years, still being the same person, and so on. To have true knowledge is to have an infallible knowledge of the real, and the real can be grasped only in a clear, universal definition, not be opinion. We have to ask about the "essence" of things.

In The Republic, Plato sets forth the simile of the line by which he divides all knowledge into the realm of opinion and the realm of true knowledge. While opinion relates to particulars (for example, an individual horse), knowledge relates to universals (the essence of horses, the "horseness", that is applicable in all cases, the norm of the particular horses). For Plato, opinions can be shaken by criticism or by conflicting evidence, while true knowledge cannot. In the Republic, he seeks to illustrate his meaning by distinguishing four grades of cognition, each with its own class of objects. The lowest grade is that of mere guesswork (eikasia), which has as its objects the images of dreams or the reflections in water. A higher state of cognition is that of belief pistis), where one has learned to distinguish physical things from their mere shadows. Here a person has a conviction about the experience of the world as known through the senses. It is only when we move higher, to understanding (dianoia), that we have knowledge— when we move, so to speak, from a particular horse to the essence, "horseness," that which makes all horses alike as horses but different from human beings and other animals. There is, however, one more step needed to ascend to the supreme first principle (noesis). Each step in the ascent to knowledge moves to a higher level of abstraction, farther and farther from the particular and more and more toward the universal: from the shadow of a horse to a specific horse to horseness to the basic and fundamental principles characteristic of all biological life.

Simone Klein
http://www.sophiasworld.at/

back

Kamesha asked:

How can you distinguish the conclusion from the premises of an argument?

Explain why arguments with fallacies can still be persuasive.

An argument consists of at least two propositions, one of which is the argument's conclusion. The other proposition (or propositions) should entail the conclusion: one must be able to deduce the conclusionfrom the other proposition(s), called a premise (or premises). In a sound argument, ifevery member of the set of premises is true, then conclusion must also be true.

The "arrow" of implication is not always reversible. For example:

There are five animals in that telephone booth.

There are five elephants in that telephone booth.

I can deduce the first proposition from the second, but not the second from the first.

Any proposition can be deduced from itself. ("The dog barked," for example, can be deduced from "The dog barked," but so what?) The following is a bit more interesting:

Someone is a husband.

Someone is a wife.

Either proposition can serve as the conclusion an argument for which the other is the sole premise, but the two are not identical. "Husband" does not mean "wife," but if someone is a husband, then someone (else) is a wife, and vice versa.

Fallacies can persuade because (a) persuasiveness can depend on the state of mind of the persuaded one rather than on the argument's logical status; and, (b) not all fallacies are obvious. To spot a fallacy sometimes requires understanding that a logical operator ("Possibly . . .," "Necessarily . . .") can be distributed in subtly different ways with dramatically different results. For instance:

God knows today that I will choose to eat an omelet for breakfast tomorrow. Therefore [i.e., it necessarily follows that], I will choose to eat an omelet for breakfast tomorrow.

If the first of these two propositions is true (leaving aside the question of its verification), then necessarily the second is true. Generally, if S knows p, then necessarily p is true, because one cannot know what is false. Unfortunately, there are philosophers on both sides of the God question who have misinterpreted this logical necessity as causal necessity and hence a denial of freedom:

Therefore, I will necessarily choose to eat an omelet tomorrow; meaning, Therefore, I am necessitated to choose to eat an omelet tomorrow; meaning, Therefore, I am not free to choose to eat a bowl of cereal with bananas for breakfast tomorrow.

But knowing cannot turn free choices into determined effects. To know is not to cause! Put that way, of course, the fallacy has no power to persuade. But it is rarely put that way. Instead, the necessity by which a premise determines a conclusion is distributed over the non necessary fact to which the premise refers.

Tony Flood

back

Sean asked:

I was wondering if you could provide me with a list of the philosophical issues/ questions/ debates of the time. I was hoping to take a shot at some of them...

This is pretty much an impossible task! There are more professional philosophers working now than at any previous time in history, and they range across a bewildering variety of problems. I suggest reading through the answers on this site — you will get a very good introduction to many of the current issues.

Tim Sprod

back

Laura asked:

What do you call a question that has no answer?

Or perhaps: a question that cannot be answered?

Perhaps you are thinking of the term "pseudo-question." A pseudo-question is one which has the form of a question, but may suppose something that is false or makes no sense.

An example would be the famous lawyer's question of the accused: "Have you stopped beating your wife?" when the accused does not have a wife, or perhaps does have a wife, but has never beaten her. Either the answer "yes" or "no" would be obviously inappropriate. This kind of question is said to commit "the fallacy of many questions."

Another type of pseudo-question would be the child's question, "How high is up?" The problem with that "question" is that it supposes that "up" is the name of a place which it is not, rather than a direction, which it is.

Another example of a pseudo-question (which is adapted from Wittgenstein) is: "What time is it now on the Sun?" This "question" has no answer because there is no way (at least at present) of calculating the time on the Sun.

Pseudo-questions, as I have said before, all have the defect of supposing something true which is not true.

Of course, there are (and always have been) questions which may have answers we do not know and, perhaps cannot discover. "Is there a God?" may be one of those. Or the question, "Is the number of stars in the Universe odd or even?" is a question which, since there is some number of stars in the Universe, there must be a correct answer, but it may be that no one will ever know the answer. Certainly, no one now knows the answer.

Ken Stern

back

Andrew asked:

What are the differences and similarities between Value judgement and factual judgement?

There is an excellent chapter on value judgements in "Learning to Philosophize" by E.R. Emmet, but I think it might be out of print, so I will give you a summary:

Factual Judgement

I have in front of me two mini-skirts, and I say that this one is longer than that one. It would be generally agreed that whether this is true or not is a matter of fact. It can be decided by putting them beside one another — assuming the difference is obvious to the naked eye. As long as everyone understands what 'longer than' means,we will all agree. If the difference was not visible to the naked eye, and had to be decided by accurate measuring, everyone would still accept that it is a matter of fact which one is longer. A factual statement is able to be publicly tested, can be verified or falsified by experience. It is often called objective.

Value Judgement

In the furniture store, I try lying on two futons, and I say, "This futon is less comfortable than that one." I am saying that I personally find one futon less comfortable than the other, with perhaps also the assumption that most people will agree with me. Statements such as this, which express or describe the attitude of the speaker, are often called subjective.

Many kinds of judgement lie between these two clear-cut extremes, and it can then be difficult to decide whether they should be regarded as matters of fact or matters of taste. They include many instances where we mark or rate something in comparison to something else, or say that something is better than something else. Emmet gives the example of a teacher marking essays — we want to say it is more than just a matter of opinion what grades they are given, but it has an element of subjectivity too.

Consider this question posed by Emmet in the book:

Consider and discuss the criteria you would use if you had to judge the following contests. To what extent would they be matters of fact, and to what extent matters of taste? Would you be likely to have to compare the amount by which A was better than B in one respect with the amount by which B was better than A in another?

i. A Drill Competition
ii. A Flower Show
iii. A Music Competition
iv. A Beauty Contest

Katharine Hunt

back

Phil asked:

Is science the new religion?

Ostensibly it might seem so to many, but no genuinely religious person would accept it to be so. There is also a strong philosophical denial of the possibility. In fact many scientists themselves feel that the more progress they make the less secure their materialistic views become.

The philosophical challenge lies in the fact that scientists are not creators but discoverers; they discover things already in existence: therefore, science does not possess the qualification necessary for a religion. When they seem to be creative they are using already established natural laws and conditions, and are restricted within set parameters.

After seemingly establishing a sound materialistic basis for science following the advent of Newton, the scientific world was thrown into some confusion by the theories of Einstein, the ability to make secure predictions about the universe suddenly disappeared in the ramifications of the quantum theory; matter itself reduced to light and became known as rest mass energy. Some physicists began talking about the universe as a great thought rather than a great machine.

We live in a world that is still fundamentally religious; like science, religion continues to seek a meaning to life. Ironically religion does not succumb to scientific discovery but rather is able to use the facts provided to reinforce its own beliefs. Strangely, there is an intermediate ground of UFO's, meditation, the occult, scientology, etc., to which religion gives a mystic dimension, whilst science, though sceptical, feels bound to investigate what is considered to be natural phenomena.

When we consider that science has been responsible for producing the most horrific weapons of mass destruction known to man, the idea of it being a religion is contradictory to everything we understand about religion. On the other hand, we have to look for true religion beyond its institutionalized facade, its political attachments and its constant participation in violent conflict. Because the various sects and organizations fail to live up to the standards of morality, peace, love and virtue, this does not hide the fact that these are the underlying facets of true religion. Science does not have such a foundation, it can therefore never be considered a religion. Perhaps in this modern secular age some people may regard science as a substitute for religion; a materialist age may require a materialist religion, science is probably the nearest approach to this. Another point worth noting is that the received knowledge in developed countries indicates that science has done much more for the well being of the population than has religion, for example, drugs and advances in medical treatment generally has had greater effect than has faith healing and hoped for miracles. Most people are also unaware of where the border line is between science and technology, hence all good progress is put down to science.

Probably science will only replace religion when it can be proved beyond all possible doubt that God has nothing to do with physics, biology and chemistry; and that possibility still seems a long way off.

John Brandon


I must seriously call into question the previous answer.

First, I don't understand the claim that "no genuinely religious person would accept" that science is a religion. This claim seems always to leave a way out from refutations by saying "Well, that person might be religious and think science is a religion, but of course he's not genuinely religious" — what is known as the "no true Scotsman fallacy".

Second, it is said that "many scientists themselves feel that the more progress they make the less secure their materialistic views become". This seems to imply that religion necessarily has to do with the supernatural. In Buddhism, for instance, there is no demand of allegiance to any supernatural being. And there are even Christian philosophers, like Don Cupitt, who claim that the Bible is not a document of a supernatural being's interaction with the world, but a metaphorical story. One can turn one's feeling of freedom from the supernatural into an idol, just like one can turn one's feeling of allegiance to the supernatural into an idol.

Third, it is pointed out that "scientists are not creators but discoverers". But the comparison of science to religion normally proposes to compare scientists to priests and not gods. What takes the place of gods in the science religion is not any being, but the whole of reality as viewed scientifically, with scientists as merely the priests entrusted with the task of interpreting it.

Fourth, there have been actual attempts to found an institutionalized religion which worships the achievements of science and scientific facts and laws. In fact, Auguste Comte, the founding father of positivism, founded one such in the 1840s, the Church of Positivism, which is still functioning in Brazil (and even has a web site).

Fifth, it is said that "science will only replace religion when it can be proved beyond all possible doubt that God has nothing to do with physics, biology and chemistry". It seems to me that if something like that were to happen, it would contrariwise take away any need for science to replace religion.

T. P. Uschanov
Research Assistant
Department of Philosophy
University of Helsinki

back

Peter asked:

I am currently wrestling with the question of whether any theory can be falsified beyond doubt. Basically the Popper/ Lakatos vs. Quine/ Duhem (and implicitly Kuhn) divide. I desperately need some secondary sources, if indeed they exist, which address this issue. My own feeling is that there exists an objective truth which is real rather than ideal but which we will never arrive at. However, if nothing is falsifiable then how do we distinguish between science and belief? I don't expect an answer, just hope someone can help push me towards some helpful material. I've read Popper, Kuhn, Lakatos etc. — this is an advanced search!

My recommendations would be:

Kitcher, P. (1993). The advancement of science; science without legend, objectivity without illusions. New York, NY: Oxford University Press.
Kitcher, P. (1992). 'The naturalists return'. The Philosophical Review, 101 (1), 53-114.
Giere, R. N. (1999). Science without laws. Chicago, IL: The University of Chicago Press.

Kitcher was a student of Kuhn, and I'm a big fan of his. However, there are, as you may know, mixed reviews of his latest book.

Steven Ravett Brown

back

Stephan asked:

Is there a conflict between morality and rationality? Can all moral phenomena be accommodated within utility theory?

Two questions, to which I would answer 'no' and 'no'.

In the first case, we need to be careful about what we mean by 'rationality'. Immanuel Kant would say that rationality provides us directly with the Moral Law, so there is no conflict. I disagree with Kant on this, finding his account of rationality very austere and hyper-logical, ignoring the multi-faceted nature of (what I would prefer to call) reasonableness, as it operates in the real world. But if we have an account of reasonableness which is rich enough, taking in context, emotion, embodiment and creativity, then I think that we can see morality as an attempt to negotiate how to live lives that are good for us and for all others around us. This is pretty much Aristotle's view, with perhaps a twist given to it from Jurgen Habermas' discourse ethics. Aristotle talked of practical wisdom, which I think is pretty similar to what I have called reasonableness.

I don't think that utilitarianism can cover all morality. There are several problems with it. Here's one: utilitarianism would allow as moral the killing in cold blood of an innocence person if that death would save enough others. For me, the fact that utilitarianism clashes with our deepest moral intuitions is enough to rule it out.

Tim Sprod

back

Shamim asked:

How can we understand that our beliefs are true or not? If we wanted to help somebody in his beliefs what should we do?

Shamim also asked:

How can we improve our willpower, determination?

I'm not sure that I should even be answering these, and they've been hanging around for a long time. Basically, these questions are just too general to answer meaningfully in anything short of several courses in philosophy and psychology. But I'll give you some really simplistic answers. 1) Study science and philosophy of science. Don't study religion. 2) Encourage them to question their beliefs and to study science. 3) By exercising it. That's the only way. Start by doing something you know you need to do for, say, 20 minutes a day. Over a period of months, increase that time gradually.

Steven Ravett Brown

back

Robert asked:

Could someone explain to me as to what extend Wittgenstein's statements in Philosophical Investigations about "family resemblances" (and therefore the impossibility of constructing philosophical generalisations regarding language, meaning and a lot more) is not a philosophical statement? (Since according to para.128 there aren't any.)

The idea of "Family Resemblances" can be found in Philosophical Grammar, in The Blue and Brown Books and inPhilosophical Investigations, especially from paragraphs 65 to 88. Such ideas can't be regarded as philosophical considerations because they are not proposed as a conceptualization but as a description of the language. They could appear in a "Manual of Language", but not in a "Philosophical Compendium on Problems of Language", for the "family resemblances" are just a verification. Such is the language, according to Wittgenstein. Descriptions are not philosophical.

Actually, what is at stake here is just the fight to what is considered philosophical. If someone asks "what is a game?", we are tempted to make generalizations, for instance, "a game is a pastime". For a professional football player, this would be hardly true. "A game includes the possibility of victory or defeat" would be hardly true for a child that bounces a ball. And so on...What Wittgenstein demonstrates is only that there are certain similarities between what we call games, and that these similarities cannot be taken as a definition of games. I emphasize the word demonstrates. To point to something doesn't mean to propose a definition, therefore, it could hardly be considered as a philosophical task.

Mauricio Micheletti

back

Melissa asked:

While there are some fundamental flaws in the use of the words "ask a philosopher", which implies internally that we are all not "philosophers" in our own right, I do hope that this question is going to someone with an authority of sorts.

My question is not of a philosophical essay interpretation, or of a philosophical discussion of the questions that make philosophy the science of questions, but rather to ask for guidance on a more individualized personal matter.

I have been bitten by the "love of Wisdom" studies and will, in a few weeks, be graduating with an associates (completion of General Education).

My goal is to pursue a career either in being a philosophy professor, a lobbyist, or law student (hopefully all three in a reverse order).

So simply, my question is this, what would be the best path to attaining these goals? What schools may be better equipped for this focus?

I understand that these questions may very well go against the intended nature of this website, but hopefully I have neither offended nor taken time unfairly.

Actually, I think there is a sense in which we're not all philosophers.

So you like wisdom literature, but I feel that if this is so the question seems ironic.

But ignoring that and looking at the facts, I would suggest that you do a philosophy degree and then study law. Surely you can find out the good schools to go to? Even if you live in isolation from the media and have no friends, there must be a careers office.

In England, once you have a degree, you can do the Common Professional Examination which takes a year and is acknowledged to be equivalent to a law degree. Then you can do a year's training to be a solicitor or a barrister, and I'd suggest the latter for lobbying. You might live in the US, but there is presumably something equivalent. Once your career is established, you will be able to take fewer legal cases, get involved in lobbying and at the same time you can take a Philosophy Masters and then a PhD part-time. There you go.

Rachel Browne

back

Mode, Azim, Ramy, and Niya asked:

We are students from Maldives (where philosophical resources are scarcely available and philosophical enthusiasm hardly found and philosophical expressions highly restricted). we would like to know whether a god as held by major religions of the world exists and and to know whether what the logical positivists say about the meaninglessness of such questions as the existence of a god is really true?

To tell you that "a god as held by major religions of the world exists" or doesn't exist would be as helpful as telling you that the mind is a substance distinct from the body— or telling you that it isn't. There are brilliant thinkers on every side of every philosophical question, including the question of the existence of God, even the question of how to think about God. These are matters of worldview, not of fact, and one comes to judgment about them by thinking, not consulting experts. "Is there a God?" is not like "Is there, in 2002, a King of France?"

The movement called Logical Positivism was the expression of a methodological bias that quickly put itself out of business about 70 years ago. It tried to banish metaphysics from philosophy by identifying meaningful, i.e., intelligible, discourse with talk about what can be verified empirically. By "empirically," however, it meant through sense perception. In so doing, Logical Positivists implicitly affirmed that reality was ultimately a certain way and therefore could not include certain facts (like the theistic God, for example). Unfortunately for them, sense perception is no more able to verify the implicit metaphysics of logical positivism than it can the explicit metaphysics of the most ambitious idealist. A less empirical approach to philosophy could hardly be imagined.

Tony Flood


I most certainly can't answer whether god exists! If by 'the god of the major religions' you mean the one god of Judaism, Christianity and Islam, then I think that the argument from evil is hard to argue against (it says that the existence of evil is not compatible with an all-good, all-knowing and all-powerful god). However, other major religions do not have this problem, and there are ways around the argument from evil, if one denies any one of the three properties of being all-good, all-knowing and all-powerful.

The logical positivists' Principle of Verifiability says that the only statements that are meaningful are those for which we can show how they would be verified. Basically, this means that only logically true, and scientific, statements have meaning. The statement 'god exists' (they say) does not fit into either category and hence is meaningless. The main problem with this view is that the Principle of Verifiability itself is neither logically true, nor a scientific statement, and hence must be meaningless by its own lights.

Tim Sprod

back

Azim also asked:

I would like to know with the incorporation of latest philosophical reasoning which position (theism, agnosticism, atheism,etc) as regards a god is logically sound or which a rational mind will go for!

The problem with asking for 'the latest philosophical reasoning' on any particular question is that the latest thinking will include people who support a number of different answers to the question. It is very different from the latest (say) scientific thinking on an issue, which will often be narrowing down to a particular answer. So, there are people who would argue quite persuasively for each of the positions you mention. You would need to read them yourself and think about which one sounds right to you.

All the answers that these various people give would be logically valid, because any decent philosopher ought to be able to construct an argument which does not break logical rules. But to be logically sound, it must use this logic on true premises, and people disagree about which statements are true.

Similarly, I don't think that we can judge rationality by what conclusions we come to. Rationality is more about the process of dealing with thoughts, beliefs and values. What outcome we come to in our deliberations depends in part on the premises and values from which we start. I cannot see that rationality always forces us to a single conclusion. In some (quite a few) questions it does, but in many others it can only help us separate some better from worse answers, but not to decide which is the 'best'. It is a bit like judging music — while we can separate out good from not-so-good music, who would ever say we can find the best? That's what I think the god question is like. [I guess this is an argument for agnosticism, isn't it!]

Tim Sprod

back

Andy asked:

I have recently read some of John Locke's writings on the nature of the Universe. He states that the basis of knowledge is sensation and reflection upon sensation. I am wondering if he meant senses in purely the scientific sense (as in the five) or if he believed in the existence of a sixth sense per se that would utilize "Divine Revelation". I'm probably very vague but humor me.

In Chapters XVIII and XIX Book IV, of his Essay Concerning Human Understanding titled "Of Enthusiasm" Locke discusses faith, reason, and revelation. In those chapters, Locke argues for the supremacy of reason on the ground that although revelation, since it is a deliverance from God, cannot be false, it is up to reason to decide whether a deliverance is or is not a revelation, since revelation cannot be self-certifying. So that if there is an apparent revelation which is contrary to reason, we can be sure that it is not a real revelation. "...no Proposition" (writes Locke) "can be received for Divine Revelation, or obtain the consent due to all such, if it be contradictory to our clear intuitive knowledge. Because this would be to subvert the Principles, and Foundations of all Knowledge, Evidence, and Assent whatever...." IV. XIII, 5,10.

And in his section on "Enthusiasm" (what we, nowadays, would call "fanaticism," whether religious or political or both) which Locke, in a delightful trope describes as "founded neither on Reason nor Divine Revelation, but rising from the Conceits of a warmed or over-weening Brain, works yet, where it once gets a footing, more powerfully, on the Persuasions and Actions of Men, than either of the other two, or both together..."

Locke speaks to all of us who read about the goings-on in the Middle-East.

Ken Stern

back

Katherine asked:

I have a philosophy exam in Moral Reasoning, and I'm having some problems with a certain question I should be studying. The question is, "Can an act of Parliament make people better?"

I'll say what I think the intent of the question is, then you will have to work out your answer (although I think the question should start with 'should' not 'can' — otherwise it is an empirical and not a philosophical question). Let's take an example. Assume that we agree that gambling is morally bad — and bad for you — and not gambling makes you morally better. Should there be a law against gambling?

One view would be that it is not the place of the law to try to improve people morally. If people want to gamble, that is their decision. Others say that it is the state's (or community's) duty to protect people and look after them by stopping them doing wrong. (See also my answer on Answers page 13, no. 22).

Some argue that, even if a law trying to enforce morality is not used (much), because it is hard to enforce, or because the behaviour is so widespread that police turn a blind eye, it is still a good thing to have on the books because it expresses community distaste for the behaviour and thus may deter some people from doing it.

You might also think about whether your answer would be different depending on how it was that the law was trying to make people better. Is it OK for the law to try to help people turn away from murdering others? Or to stop them not giving money to charities? Where is the dividing line between cases where the law is justified in imposing moral behaviour and when it is not?

Tim Sprod

back

Abigail asked:

I am an A-level student currently writing an essay on Plato's main ideas about religion and how they are influential in religious philosophy. I am stuck on how to bring the ideas of kosmos noetos and aeithetos into the argument as I cannot seem to find any information on these two ideas anywhere on the internet. please could you point me in the right direction as to where I should be looking.

Look here:

http://www.perseus.tufts.edu/cgi-bin/perscoll?collection=Perseus:collection:PersInfo&type=interactive+resource

Steven Ravett Brown

back

Jeannie asked:

Of all the ethical issues, which one is most important and why?

My reaction is that I don't much like the assumption behind the question. That assumption is that we can quantify the importance of ethical issues in such a way that we can say one in particular is more important than another. I don't think that ethical issues are quantifiable in that way. A utilitarian might say they can ("Which ethical issue has the potential to create the greatest amount of harm and the least amount of happiness?"). I disagree. I would prefer to follow Aristotle, who says "The whole accounts of matters of conduct must be given in outline and not precisely... accounts we demand must be in accordance with the subject matter; matters concerned with conduct and questions of what is good for us have no fixity... the agents themselves must in each case consider what is appropriate to the occasion". That is, we cannot be precise and quantitative about ethical matters.

My guess is the question is asking you to pick on an ethical issue that matters a lot to you, and to explain why you think it so important.

Tim Sprod


Favourite candidates in my view would have to be two questions — are there (natural) moral laws? — and: — do we have free will? In other words the issues of whether there is a way to determine other than subjectively what we should do, and whether we have the free decision as to whether we actually do it.

The importance of whether there are moral laws or not lies in that people normally tell others what to do without explaining where these maxims come from i.e. what they are based upon. Now if we share the same belief system we might accept argument from authority, habit etc. — but if we have different belief systems in order to convince others to act ethically or morally we would have to convince them (by reasoning) of some universally applicable principles existing beyond the belief-systems. (Such principles could be even evolutionary advantage ...)

Free will is important because without it — how could you act morally? If you are wholly determined by your genetics and upbringing and personal history etc. — are you a free agent? Does it make sense to assign blame or praise for moral actions?

Helene Dumitriu